NUR 207 final

Réussis tes devoirs et examens dès maintenant avec Quizwiz!

1. In a patient with a disease that affects the myelin sheath of nerves, such as multiple sclerosis, the glial cells affected are the a. microglia. b. astrocytes. c. ependymal cells. d. oligodendrocytes.

1. d

10. The nurse is caring for a patient with peripheral neuropathy who is scheduled for EMG studies tomorrow morning. The nurse should a. ensure the patient has an empty bladder. b. instruct the patient there is no risk of electric shock. c. ensure the patient has no metallic jewelry or metal fragments. d. instruct the patient that pain may be experienced during the study.

10. Correct answer: b Rationale: Electromyography (EMG) is used to assess electrical activity associated with nerves and skeletal muscles. Activity is recorded by insertion of needle electrodes to detect muscle and peripheral nerve disease. The nurse should inform the patient that pain and discomfort are associated with insertion of needles. There is no risk of electric shock with this procedure.

A nurse is caring for a patient with acute respiratory distress syndrome (ARDS) who is receiving mechanical ventilation using synchronized intermittent mandatory ventilation (SIMV). The settings include fraction of inspired oxygen (FIO2) of 80%, tidal volume of 450, rate of 16/minute, and positive end-expiratory pressure (PEEP) of 5 cm. Which assessment finding is most important for the nurse to report to the health care provider? a. O2 saturation of 99% b. Heart rate 106 beats/minute c. Crackles audible at lung bases d. Respiratory rate 22 breaths/minute

ANS: A The FIO2 of 80% increases the risk for O2 toxicity. Because the patient's O2 saturation is 99%, a decrease in FIO2 is indicated to avoid toxicity. The other patient data would be typical for a patient with ARDS and would not be the most important data to report to the health care provider. DIF: Cognitive Level: Analyze (analysis)

The nurse is caring for a patient who arrived in the emergency department with acute respiratory distress. Which assessment finding by the nurse requires the most rapid action? a. The patient's PaO2 is 45 mm Hg. b. The patient's PaCO2 is 33 mm Hg. c. The patient's respirations are shallow. d. The patient's respiratory rate is 32 breaths/min.

ANS: A The PaO2 indicates severe hypoxemia and respiratory failure. Rapid action is needed to prevent further deterioration of the patient. Although the shallow breathing, rapid respiratory rate, and low PaCO2 also need to be addressed, the most urgent problem is the patient's poor oxygenation. DIF: Cognitive Level: Analyze (analysis)

A patient who has type 1 diabetes plans to swim laps for an hour daily at 1:00 PM. The clinic nurse will plan to teach the patient to a. check glucose level before, during, and after swimming. b. delay eating the noon meal until after the swimming class. c. increase the morning dose of neutral protamine Hagedorn (NPH) insulin. d. time the morning insulin injection so that the peak occurs while swimming.

ANS: A The change in exercise will affect blood glucose, and the patient will need to monitor glucose carefully to determine the need for changes in diet and insulin administration. Because exercise tends to decrease blood glucose, patients are advised to eat before exercising. Increasing the morning NPH or timing the insulin to peak during exercise may lead to hypoglycemia, especially with the increased exercise. DIF: Cognitive Level: Apply (application) REF: 1132 TOP: Nursing Process: Planning MSC: NCLEX: Physiological Integrity

Which assessment finding obtained by the nurse when caring for a patient with a right radial arterial line indicates a need for the nurse to take action? a. The right hand feels cooler than the left hand. b. The mean arterial pressure (MAP) is 77 mm Hg. c. The system is delivering 3 mL of flush solution per hour. d. The flush bag and tubing were last changed 2 days previously.

ANS: A The change in temperature of the right hand suggests that blood flow to the right hand is impaired. The flush system needs to be changed every 96 hours. A mean arterial pressure (MAP) of 75 mm Hg is normal. Flush systems for hemodynamic monitoring are set up to deliver 3 to 6 mL/hr of flush solution. DIF: Cognitive Level: Apply (application)

During change-of-shift report, the nurse is told that a patient has been admitted with dehydration and hypotension after having vomiting and diarrhea for 4 days. Which finding is most important for the nurse to report to the health care provider? a. New onset of confusion c. Heart rate 112 beats/min b. Decreased bowel sounds d. Pale, cool, and dry extremities

ANS: A The changes in mental status are indicative that the patient is in the progressive stage of shock and that rapid intervention is needed to prevent further deterioration. The other information is consistent with compensatory shock. DIF: Cognitive Level: Analyze (analysis)

The nurse is assessing a 55-yr-old female patient with type 2 diabetes who has a body mass index (BMI) of 31 kg/m2 .Which goal in the plan of care is most important for this patient? a. The patient will reach a glycosylated hemoglobin level of less than 7%. b. The patient will follow a diet and exercise plan that results in weight loss. c. The patient will choose a diet that distributes calories throughout the day. d. The patient will state the reasons for eliminating simple sugars in the diet.

ANS: A The complications of diabetes are related to elevated blood glucose and the most important patient outcome is the reduction of glucose to near-normal levels. A BMI of 31 kg/m2 or above is considered obese, so the other outcomes are appropriate but are not as high in priority. DIF: Cognitive Level: Analyze (analysis) REF: 1124 OBJ: Special Questions: Prioritization TOP: Nursing Process: Planning MSC: NCLEX: Physiological Integrity

What glomerular filtration rate (GFR) would the nurse estimate for a 30-yr-old patient with a creatinine clearance result of 60 mL/min? a. 60 mL/min c. 120 mL/min b. 90 mL/min d. 180 mL/min

ANS: A The creatinine clearance approximates the GFR. The other responses are not accurate. DIF: Cognitive Level: Understand (comprehension) REF: 1025 TOP: Nursing Process: Assessment MSC: NCLEX: Physiological Integrity

Which data collected by the nurse caring for a patient who has cardiogenic shock indicate that the patient may be developing multiple organ dysfunction syndrome (MODS)? a. The patient's serum creatinine level is elevated. b. The patient complains of intermittent chest pressure. c. The patient's extremities are cool and pulses are weak. d. The patient has bilateral crackles throughout lung fields.

ANS: A The elevated serum creatinine level indicates that the patient has renal failure as well as heart failure. The crackles, chest pressure, and cool extremities are all symptoms consistent with the patient's diagnosis of cardiogenic shock. DIF: Cognitive Level: Apply (application)

A patient tells the nurse about using acetaminophen (Tylenol) several times every day for recurrent bilateral headaches. Which action will the nurse plan to take first? a. Discuss the need to stop taking the acetaminophen. b. Suggest the use of biofeedback for headache control. c. Describe the use of botulism toxin (Botox) for headaches. d. Teach the patient about magnetic resonance imaging (MRI).

ANS: A The headache description suggests that the patient is experiencing medication overuse headache. The initial action will be withdrawal of the medication. The other actions may be needed if the headaches persist. DIF: Cognitive Level: Analyze (analysis) REF: 1372 OBJ: Special Questions: Prioritization TOP: Nursing Process: Planning MSC: NCLEX: Physiological Integrity

The nurse is caring for a patient who has a head injury. Which finding, when reported to the health care provider, should the nurse expect will result in new prescribed interventions? a. Pale yellow urine output of 1200 mL over the past 2 hours. b. Ventriculostomy drained 40 mL of fluid in the past 2 hours. c. Intracranial pressure spikes to 16 mm Hg when patient is turned. d. LICOX brain tissue oxygenation catheter shows PbtO2 of 38 mm Hg.

ANS: A The high urine output indicates that diabetes insipidus may be developing, and interventions to prevent dehydration need to be rapidly implemented. The other data do not indicate a need for any change in therapy. DIF: Cognitive Level: Apply (application)

A patient with respiratory failure has arterial pressure-based cardiac output (APCO) monitoring and is receiving mechanical ventilation with peak end-expiratory pressure (PEEP) of 12 cm H2O. Which information indicates that a change in the ventilator settings may be required? a. The arterial pressure is 90/46. b. The stroke volume is increased. c. The heart rate is 58 beats/minute. d. The stroke volume variation is 12%.

ANS: A The hypotension suggests that the high intrathoracic pressure caused by the PEEP may be decreasing venous return and (potentially) cardiac output. The other assessment data would not be a direct result of PEEP and mechanical ventilation. DIF: Cognitive Level: Apply (application)

When evaluating a patient with a central venous catheter, the nurse observes that the insertion site is red and tender to touch and the patient's temperature is 101.8° F. What should the nurse plan to do? a. Discontinue the catheter and culture the tip. b. Use the catheter only for fluid administration. c. Change the flush system and monitor the site. d. Check the site more frequently for any swelling.

ANS: A The information indicates that the patient has a local and systemic infection caused by the catheter, and the catheter should be discontinued to avoid further complications such as endocarditis. Changing the flush system, continued monitoring, or using the line for fluids will not help prevent or treat the infection. DIF: Cognitive Level: Apply (application)

A patient who is orally intubated and receiving mechanical ventilation is anxious and is "fighting" the ventilator. Which action should the nurse take next? a. Verbally coach the patient to breathe with the ventilator. b. Sedate the patient with the ordered PRN lorazepam (Ativan). c. Manually ventilate the patient with a bag-valve-mask device. d. Increase the rate for the ordered propofol (Diprivan) infusion.

ANS: A The initial response by the nurse should be to try to decrease the patient's anxiety by coaching the patient about how to coordinate respirations with the ventilator. The other actions may also be helpful if the verbal coaching is ineffective in reducing the patient's anxiety. DIF: Cognitive Level: Analyze (analysis)

A patient reports feeling numbness and tingling of the left arm before experiencing a seizure. The nurse determines that this history is consistent with what type of seizure? a. Focal c. Absence b. Atonic d. Myoclonic

ANS: A The initial symptoms of a focal seizure involve clinical manifestations that are localized to a particular part of the body or brain. Symptoms of an absence seizure are staring and a brief loss of consciousness. In an atonic seizure, the patient loses muscle tone and (typically) falls to the ground. Myoclonic seizures are characterized by a sudden jerk of the body or extremities. DIF: Cognitive Level: Understand (comprehension) REF: 1376 TOP: Nursing Process: Assessment MSC: NCLEX: Physiological Integrity

A patient who had arthroscopic surgery of the right knee 7 days ago is admitted with a red, swollen, hot knee. Which assessment finding by the nurse should be reported to the health care provider immediately? a. The blood pressure is 86/50 mm Hg. b. The patient says the knee pain is severe. c. The white blood cell count is 11,500/μL. d. The patient is taking ibuprofen (Motrin).

ANS: A The low blood pressure suggests the patient may be developing septicemia as a complication of septic arthritis. Immediate blood cultures and initiation of antibiotic therapy are indicated. The other information is typical of septic arthritis and should also be reported to the health care provider, but it does not indicate any immediately life-threatening problems. DIF: Cognitive Level: Analyze (analysis) REF: 1535 OBJ: Special Questions: Prioritization TOP: Nursing Process: Assessment MSC: NCLEX: Physiological Integrity

A patient with diabetic ketoacidosis is brought to the emergency department. Which prescribed action should the nurse implement first? a. Infuse 1 L of normal saline per hour. b. Give sodium bicarbonate 50 mEq IV push. c. Administer regular insulin 10 U by IV push. d. Start a regular insulin infusion at 0.1 units/kg/hr.

ANS: A The most urgent patient problem is the hypovolemia associated with diabetic ketoacidosis (DKA), and the priority is to infuse IV fluids. The other actions can be done after the infusion of normal saline is initiated. DIF: Cognitive Level: Analyze (analysis) REF: 1144 OBJ: Special Questions: Prioritization TOP: Nursing Process: Implementation MSC: NCLEX: Physiological Integrity

The nurse is caring for a patient who is intubated and receiving positive pressure ventilation to treat acute respiratory distress syndrome (ARDS). Which finding is most important to report to the health care provider? a. Red-brown drainage from nasogastric tube b. Blood urea nitrogen (BUN) level 32 mg/dL c. Scattered coarse crackles heard throughout lungs d. Arterial blood gases: pH of 7.31, PaCO2 of 50, and PaO2 of 68

ANS: A The nasogastric drainage indicates possible gastrointestinal bleeding or stress ulcer and should be reported. The pH and PaCO2 are slightly abnormal, but current guidelines advocating for permissive hypercapnia indicate that these would not indicate an immediate need for a change in therapy. The BUN is slightly elevated but does not indicate an immediate need for action. Adventitious breath sounds are commonly heard in patients with ARDS. DIF: Cognitive Level: Analyze (analysis)

A patient is being evaluated for a possible spinal cord tumor. Which finding by the nurse requires the most immediate action? a. The patient has new-onset weakness of both legs. b. The patient complains of chronic severe back pain. c. The patient starts to cry and says, "I feel hopeless." d. The patient expresses anxiety about having surgery.

ANS: A The new symptoms indicate spinal cord compression, an emergency that requires rapid treatment to avoid permanent loss of function. The other patient assessments also need nursing action but do not require intervention as rapidly as the new-onset weakness. DIF: Cognitive Level: Analyze (analysis)

A patient with a history of polycystic kidney disease is admitted to the surgical unit after having shoulder surgery. Which of the routine postoperative orders is most important for the nurse to discuss with the health care provider? a. Give ketorolac 10 mg PO PRN for pain. b. Infuse 5% dextrose in normal saline at 75 mL/hr. c. Order regular diet after patient is awake and alert. d. Draw blood urea nitrogen (BUN) and creatinine in 2 hours.

ANS: A The nonsteroidal antiinflammatory drugs (NSAIDs) should be avoided in patients with decreased renal function because nephrotoxicity is a potential adverse effect. The other orders do not need any clarification or change.

Which medication taken at home by a patient with decreased renal function will be of most concern to the nurse? a. ibuprofen (Motrin) c. folic acid (vitamin B9) b. warfarin (Coumadin) d. penicillin (Bicillin C-R)

ANS: A The nonsteroidal antiinflammatory medications (NSAIDs) are nephrotoxic and should be avoided in patients with impaired renal function. The nurse should also ask about reasons the patient is taking the other medications, but the medication of most concern is the ibuprofen. DIF: Cognitive Level: Analyze (analysis) REF: 1020 TOP: Nursing Process: Assessment MSC: NCLEX: Physiological Integrity

A patient who is hospitalized with pneumonia is disoriented and confused 3 days after admission. Which information indicates that the patient is experiencing delirium rather than dementia? a. The patient was oriented and alert when admitted. b. The patient's speech is fragmented and incoherent. c. The patient is oriented to person but disoriented to place and time. d. The patient has a history of increasing confusion over several years.

ANS: A The onset of delirium occurs acutely. The degree of disorientation does not differentiate between delirium and dementia. Increasing confusion for several years is consistent with dementia. Fragmented and incoherent speech may occur with either delirium or dementia. DIF: Cognitive Level: Understand (comprehension) REF: 1400 TOP: Nursing Process: Assessment MSC: NCLEX: Physiological Integrity

After change-of-shift report, which patient should the nurse assess first? a. Patient with a urethral stricture who has not voided for 12 hours b. Patient who has cloudy urine after orthotopic bladder reconstruction c. Patient with polycystic kidney disease whose blood pressure is 186/98 mm Hg d. Patient who voided bright red urine immediately after returning from lithotripsy

ANS: A The patient information suggests acute urinary retention, which is a medical emergency. The nurse will need to assess the patient and consider whether to insert a retention catheter. The other patients will also be assessed, but their findings are consistent with their diagnoses and do not require immediate assessment or possible intervention.

Which information will the nurse include in teaching a female patient who has peripheral arterial disease, type 2 diabetes, and sensory neuropathy of the feet and legs? a. Choose flat-soled leather shoes. b. Set heating pads on a low temperature. c. Use callus remover for corns or calluses. d. Soak feet in warm water for an hour each day.

ANS: A The patient is taught to avoid high heels and that leather shoes are preferred. The feet should be washed, but not soaked, in warm water daily. Heating pad use should be avoided. Commercial callus and corn removers should be avoided. The patient should see a specialist to treat these problems. DIF: Cognitive Level: Apply (application) REF: 1151 TOP: Nursing Process: Implementation MSC: NCLEX: Physiological Integrity

A patient is admitted with possible botulism poisoning after eating home-canned green beans. Which intervention ordered by the health care provider will the nurse question? a. Encourage oral fluids to 3 L/day. b. Document neurologic symptoms. c. Position patient lying on the side. d. Observe respiratory status closely.

ANS: A The patient should be maintained on NPO status because neuromuscular weakness increases risk for aspiration. Side-lying position is not contraindicated. Assessment of neurologic and respiratory status is appropriate. DIF: Cognitive Level: Apply (application)

A nurse is caring for an obese patient with right lower lobe pneumonia. Which position will be best to improve gas exchange? a. On the left side b. On the right side c. In the tripod position d. In the high-Fowler's position

ANS: A The patient should be positioned with the "good" lung in the dependent position to improve the match between ventilation and perfusion. The obese patient's abdomen will limit respiratory excursion when sitting in the high-Fowler's or tripod positions. DIF: Cognitive Level: Apply (application)

A patient has a ruptured cerebral aneurysm and subarachnoid hemorrhage. Which intervention will the nurse include in the plan of care? a. Apply intermittent pneumatic compression stockings. b. Assist to dangle on edge of bed and assess for dizziness. c. Encourage patient to cough and deep breathe every 4 hours. d. Insert an oropharyngeal airway to prevent airway obstruction.

ANS: A The patient with a subarachnoid hemorrhage usually has minimal activity to prevent cerebral vasospasm or further bleeding and is at risk for venous thromboembolism. Activities such as coughing and sitting up that might increase intracranial pressure or decrease cerebral blood flow are avoided. Because there is no indication that the patient is unconscious, an oropharyngeal airway is inappropriate. DIF: Cognitive Level: Apply (application) REF: 1359 TOP: Nursing Process: Planning MSC: NCLEX: Physiological Integrity

Which action will the nurse include in the plan of care for a patient who is experiencing pain from trigeminal neuralgia? a. Assess fluid and dietary intake. b. Apply ice packs for 20 minutes. c. Teach facial relaxation techniques. d. Spend time talking with the patient.

ANS: A The patient with an acute episode of trigeminal neuralgia may be unwilling to eat or drink, so assessment of nutritional and hydration status is important. Because stimulation by touch is the precipitating factor for pain, relaxation of the facial muscles will not improve symptoms. Application of ice is likely to precipitate pain. The patient will not want to engage in conversation, which may precipitate attacks. DIF: Cognitive Level: Apply (application)

A patient with possible viral meningitis is admitted to the nursing unit after lumbar puncture was performed in the emergency department. Which action prescribed by the health care provider should the nurse question? a. Restrict oral fluids to 1000 mL/day. b. Elevate the head of the bed 20 degrees. c. Administer ceftriaxone (Rocephin) 1 g IV every 12 hours. d. Give ibuprofen (Motrin) 400 mg every 6 hours as needed for headache.

ANS: A The patient with meningitis has increased fluid needs, so oral fluids should be encouraged. The other actions are appropriate. Slight elevation of the head of the bed will decrease headache without causing leakage of cerebrospinal fluid from the lumbar puncture site. Antibiotics should be administered until bacterial meningitis is ruled out by the cerebrospinal fluid analysis. DIF: Cognitive Level: Apply (application)

The nurse is caring for a patient admitted with diabetes insipidus (DI). Which information is most important to report to the health care provider? a. The patient is confused and lethargic. b. The patient reports a recent head injury. c. The patient has a urine output of 400 mL/hr. d. The patient's urine specific gravity is 1.003.

ANS: A The patient's confusion and lethargy may indicate hypernatremia and should be addressed quickly. In addition, patients with DI compensate for fluid losses by drinking copious amounts of fluids, but a patient who is lethargic will be unable to drink enough fluids and will become hypovolemic. A high urine output, low urine specific gravity, and history of a recent head injury are consistent with diabetes insipidus, but they do not require immediate nursing action to avoid life-threatening complications. DIF: Cognitive Level: Analyze (analysis) REF: 1161 OBJ: Special Questions: Prioritization TOP: Nursing Process: Assessment MSC: NCLEX: Physiological Integrity

A patient with Bell's palsy refuses to eat while others are present because of embarrassment about drooling. The best response by the nurse is to a. respect the patient's feelings and arrange for privacy at mealtimes. b. teach the patient to chew food on the unaffected side of the mouth. c. offer the patient liquid nutritional supplements at frequent intervals. d. discuss the patient's concerns with visitors who arrive at mealtimes.

ANS: A The patient's desire for privacy should be respected to encourage adequate nutrition and reduce patient embarrassment. Liquid supplements may help maintain nutrition but will reduce the patient's enjoyment of the taste of food. It would be inappropriate for the nurse to discuss the patient's embarrassment with visitors unless the patient wishes to share this information. Chewing on the unaffected side of the mouth will enhance nutrition and enjoyment of food but will not decrease the drooling. DIF: Cognitive Level: Analyze (analysis)

A male patient who has possible cerebral edema has a serum sodium level of 116 mEq/L (116 mmol/L) and a decreasing level of consciousness (LOC). He is now complaining of a headache. Which prescribed interventions should the nurse implement first? a. Administer IV 5% hypertonic saline. b. Draw blood for arterial blood gases (ABGs). c. Send patient for computed tomography (CT). d. Administer acetaminophen (Tylenol) 650 mg orally.

ANS: A The patient's low sodium indicates that hyponatremia may be causing the cerebral edema. The nurse's first action should be to correct the low sodium level. Acetaminophen (Tylenol) will have minimal effect on the headache because it is caused by cerebral edema and increased intracranial pressure (ICP). Drawing ABGs and obtaining a CT scan may provide some useful information, but the low sodium level may lead to seizures unless it is addressed quickly. DIF: Cognitive Level: Analyze (analysis)

A triage nurse in a busy emergency department (ED) assesses a patient who complains of 7/10 abdominal pain and states, "I had a temperature of 103.9° F (39.9° C) at home." The nurse's first action should be to a. assess the patient's current vital signs. b. give acetaminophen (Tylenol) per agency protocol. c. ask the patient to provide a clean-catch urine for urinalysis. d. tell the patient that it will be 1 to 2 hours before seeing a health care provider.

ANS: A The patient's pain and statement about an elevated temperature indicate that the nurse should obtain vital signs before deciding how rapidly the patient should be seen by the health care provider. A urinalysis may be appropriate, but this would be done after the vital signs are taken. The nurse will not give acetaminophen before confirming a current temperature elevation. DIF: Cognitive Level: Analyze (analysis)

The following interventions are part of the emergency department (ED) protocol for a patient who has been admitted with multiple bee stings to the hands. Which action should the nurse take first? a. Remove the patient's rings. b. Apply ice packs to both hands. c. Apply calamine lotion to itching areas. d. Give diphenhydramine (Benadryl) 50 mg PO.

ANS: A The patient's rings should be removed first because it might not be possible to remove them if swelling develops. The other orders should also be implemented as rapidly as possible after the nurse has removed the jewelry. DIF: Cognitive Level: Analyze (analysis)

A 25-yr-old female patient with systemic lupus erythematosus (SLE) who has a facial rash and alopecia tells the nurse, "I never leave my house because I hate the way I look." The nurse will plan interventions with the patient to address the nursing diagnosis of a. social isolation. c. impaired skin integrity. b. activity intolerance. d. impaired social interaction.

ANS: A The patient's statement about not going anywhere because of hating the way he or she looks expresses social isolation because of embarrassment about the effects of the SLE. Activity intolerance is a possible problem for patients with SLE, but the information about this patient does not support this. The rash with SLE is nonpruritic. There is no evidence of lack of social skills for this patient. DIF: Cognitive Level: Apply (application) REF: 1542 TOP: Nursing Process: Planning MSC: NCLEX: Psychosocial Integrity

When caring for a patient with a left arm arteriovenous fistula, which action will the nurse include in the plan of care to maintain the patency of the fistula? a. Auscultate for a bruit at the fistula site. b. Assess the quality of the left radial pulse. c. Compare blood pressures in the left and right arms. d. Irrigate the fistula site with saline every 8 to 12 hours.

ANS: A The presence of a thrill and bruit indicates adequate blood flow through the fistula. Pulse rate and quality are not good indicators of fistula patency. Blood pressures should never be obtained on the arm with a fistula. Irrigation of the fistula might damage the fistula, and typically only dialysis staff would access the fistula. DIF: Cognitive Level: Understand (comprehension) REF: 1087 TOP: Nursing Process: Planning MSC: NCLEX: Physiological

Which finding from analysis of fluid from a patient's right knee arthrocentesis will be of concern to the nurse? a. Cloudy fluid c. Pale yellow fluid b. Scant thin fluid d. Straw-colored fluid

ANS: A The presence of purulent (cloudy) fluid suggests a possible joint infection. Normal synovial fluid is scant in amount and pale yellow/straw-colored. DIF: Cognitive Level: Apply (application) REF: 1457 TOP: Nursing Process: Assessment MSC: NCLEX: Physiological Integrity

14. Which finding from a patient's right knee arthrocentesis will be of concern to the nurse? a. Cloudy fluid b. Scant thin fluid c. Pale yellow fluid d. Straw-colored fluid

ANS: A The presence of purulent fluid suggests a possible joint infection. Normal synovial fluid is scant in amount and pale yellow/straw-colored.

An unresponsive patient is admitted to the emergency department (ED) after falling through the ice while ice skating. Which assessment will the nurse obtain first? a. Pulse c. Breath sounds b. Heart rhythm d. Body temperature

ANS: A The priority assessment in an unresponsive patient relates to CAB (circulation, airway, breathing) so a pulse check should be performed first. While assessing the pulse, the nurse should look for signs of breathing. The other data will also be collected rapidly but are not as essential as determining if there is a pulse. DIF: Cognitive Level: Apply (application)

An unresponsive 79-yr-old patient is admitted to the emergency department (ED) during a summer heat wave. The patient's core temperature is 105.4° F (40.8° C), blood pressure (BP) is 88/50 mm Hg, and pulse is 112 beats/min. The nurse will plan to a. apply wet sheets and a fan to the patient. b. provide O2 at 2 L/min with a nasal cannula. c. start lactated Ringer's solution at 1000 mL/hr. d. give acetaminophen (Tylenol) rectal suppository.

ANS: A The priority intervention is to cool the patient. Antipyretics are not effective in decreasing temperature in heat stroke and 100% O2 should be given, which requires a high flow rate through a non-rebreather mask. An older patient would be at risk for developing complications such as pulmonary edema if given fluids at 1000 mL/hr. DIF: Cognitive Level: Apply (application)

When prone positioning is used for a patient with acute respiratory distress syndrome (ARDS), which information obtained by the nurse indicates that the positioning is effective? a. The patient's PaO2 is 89 mm Hg, and the SaO2 is 91%. b. Endotracheal suctioning results in clear mucous return. c. Sputum and blood cultures show no growth after 48 hours. d. The skin on the patient's back is intact and without redness.

ANS: A The purpose of prone positioning is to improve the patient's oxygenation as indicated by the PaO2 and SaO2. The other information will be collected but does not indicate whether prone positioning has been effective. DIF: Cognitive Level: Apply (application)

A patient receives aspart (NovoLog) insulin at 8:00 AM. At which time would the nurse anticipate the highest risk for hypoglycemia? a. 10:00 AM b. 12:00 AM c. 2:00 PM d. 4:0 PM

ANS: A The rapid-acting insulins peak in 1 to 3 hours. The patient is not at a high risk for hypoglycemia at the other listed times, although hypoglycemia may occur. DIF: Cognitive Level: Understand (comprehension) REF: 1132 TOP: Nursing Process: Evaluation MSC: NCLEX: Physiological Integrity

While admitting a 42-yr-old patient with a possible brain injury after a car accident to the emergency department (ED), the nurse obtains the following information. Which finding is most important to report to the health care provider? a. The patient takes warfarin (Coumadin) daily. b. The patient's blood pressure is 162/94 mm Hg. c. The patient is unable to remember the accident. d. The patient complains of a severe dull headache.

ANS: A The use of anticoagulants increases the risk for intracranial hemorrhage and should be immediately reported. The other information would not be unusual in a patient with a head injury who had just arrived in the ED. DIF: Cognitive Level: Analyze (analysis)

When admitting an acutely confused patient with a head injury, which action should the nurse take? a. Ask family members about the patient's health history. b. Ask leading questions to assist in obtaining health data. c. Wait until the patient is better oriented to ask questions. d. Obtain only the physiologic neurologic assessment data.

ANS: A When admitting a patient who is likely to be a poor historian, the nurse should obtain health history information from others who have knowledge about the patient's health. Waiting until the patient is oriented or obtaining only physiologic data will result in incomplete assessment data, which could adversely affect decision making about treatment. Asking leading questions may result in inaccurate or incomplete information. DIF: Cognitive Level: Apply (application) REF: 1301 TOP: Nursing Process: Assessment MSC: NCLEX: Physiological Integrity

Which nursing action is correct when performing the straight-leg raising test for an ambulatory patient with back pain? a. Lift the patient's leg to a 60-degree angle from the bed. b. Place the patient in the prone position on the exam table. c. Ask the patient to dangle both legs over the edge of the exam table. d. Instruct the patient to elevate the legs and tense the abdominal muscles.

ANS: A When performing the straight leg-raising test, the patient is in the supine position and the nurse passively lifts the patient's legs to a 60-degree angle. The other actions would not be correct for this test. DIF: Cognitive Level: Understand (comprehension) REF: 1455 TOP: Nursing Process: Assessment MSC: NCLEX: Physiological Integrity

9. Which nursing action is correct when performing the straight-leg raising test for an ambulatory patient with back pain? a. Raise the patient's legs to a 60-degree angle from the bed. b. Place the patient initially in the prone position on the exam table. c. Have the patient dangle both legs over the edge of the exam table. d. Instruct the patient to elevate the legs and tense the abdominal muscles.

ANS: A When performing the straight leg-raising test, the patient is in the supine position and the nurse passively lifts the patient's legs to a 60-degree angle. The other actions would not be correct for this test.

After the return of spontaneous circulation following the resuscitation of a patient who had a cardiac arrest, therapeutic hypothermia is ordered. Which action will the nurse include in the plan of care? a. Initiate cooling per protocol. b. Avoid the use of sedative drugs. c. Check mental status every 15 minutes. d. Rewarm if temperature is below 91° F (32.8° C).

ANS: A When therapeutic hypothermia is used postresuscitation, external cooling devices or cold normal saline infusions are used to rapidly lower body temperature to 89.6° F to 93.2° F (32° C to 34° C). Because hypothermia will decrease brain activity, assessing mental status every 15 minutes is not done at this stage. Sedative drugs are given during therapeutic hypothermia. DIF: Cognitive Level: Apply (application)

During change-of-shift report, the nurse learns that a patient with a head injury has decorticate posturing to noxious stimulation. Which positioning shown in the accompanying figure will the nurse expect to observe? a. (1) hands curled up to chest. b.(2) hands flexed and externally rotated to side of torso. c.(3) one hand on chest and one hand on the side of the torso. d.(4)Torso prosturing

ANS: A With decorticate posturing, the patient exhibits internal rotation and adduction of the arms with flexion of the elbows, wrists, and fingers. The other illustrations are of decerebrate, mixed decorticate and decerebrate posturing, and opisthotonic posturing. DIF: Cognitive Level: Understand (comprehension)

After receiving change-of-shift report on the following four patients, which patient should the nurse see first? a. A 60-yr-old patient with right-sided weakness who has an infusion of tPA prescribed b. A 50-yr-old patient who has atrial fibrillation and a new order for warfarin (Coumadin) c. A 30-yr-old patient with a subarachnoid hemorrhage 2 days ago who has nimodipine scheduled d. A 40-yr-old patient who experienced a transient ischemic attack yesterday who has a dose of aspirin due

ANS: A tPA needs to be infused within the first few hours after stroke symptoms start in order to be effective in minimizing brain injury. The other medications should also be given as quickly as possible, but timing of the medications is not as critical. DIF: Cognitive Level: Analyze (analysis) REF: 1355 OBJ: Special Questions: Prioritization | Special Questions: Multiple Patients TOP: Nursing Process: Implementation MSC: NCLEX: Safe and Effective Care Environment

Which interventions will the nurse plan for a comatose patient who is to begin therapeutic hypothermia (select all that apply)? a. Assist with endotracheal intubation. b. Insert an indwelling urinary catheter. c. Begin continuous cardiac monitoring. d. Obtain an order to restrain the patient. e. Prepare to give sympathomimetic drugs.

ANS: A, B, C Cooling can produce dysrhythmias, so the patient's heart rhythm should be continuously monitored and dysrhythmias treated if necessary. Bladder catheterization and endotracheal intubation are needed during cooling. Sympathomimetic drugs tend to stimulate the heart and increase the risk for fatal dysrhythmias such as ventricular fibrillation. Patients receiving therapeutic hypothermia are comatose or do not follow commands so restraints are not indicated. DIF: Cognitive Level: Apply (application)

Which preventive actions by the nurse will help limit the development of systemic inflammatory response syndrome (SIRS) in patients admitted to the hospital (select all that apply)? a. Ambulate postoperative patients as soon as possible after surgery. b. Use aseptic technique when manipulating invasive lines or devices. c. Remove indwelling urinary catheters as soon as possible after surgery. d. Administer prescribed antibiotics within 1 hour for patients with possible sepsis. e. Advocate for parenteral nutrition for patients who cannot take in adequate calories.

ANS: A, B, C, D Because sepsis is the most frequent etiology for SIRS, measures to avoid infection such as removing indwelling urinary catheters as soon as possible, use of aseptic technique, and early ambulation should be included in the plan of care. Adequate nutrition is important in preventing SIRS. Enteral, rather than parenteral, nutrition is preferred when patients are unable to take oral feedings because enteral nutrition helps maintain the integrity of the intestine, thus decreasing infection risk. Antibiotics should be given within 1 hour after being prescribed to decrease the risk of sepsis progressing to SIRS. DIF: Cognitive Level: Apply (application)

A patient with Parkinson's disease is admitted to the hospital for treatment of pneumonia. Which nursing interventions will be included in the plan of care (select all that apply)? a. Provide an elevated toilet seat. b. Cut patient's food into small pieces. c. Serve high-protein foods at each meal. d. Place an armchair at the patient's bedside. e. Observe for sudden exacerbation of symptoms.

ANS: A, B, D Because the patient with Parkinson's disease has difficulty chewing, food should be cut into small pieces. An armchair should be used when the patient is seated so that the patient can use the arms to assist with getting up from the chair. An elevated toilet seat will facilitate getting on and off the toilet. High-protein foods will decrease the effectiveness of L-dopa. Parkinson's disease is a steadily progressive disease without acute exacerbations. DIF: Cognitive Level: Apply (application) REF: 1391 TOP: Nursing Process: Planning MSC: NCLEX: Physiological Integrity

A patient with suspected neurogenic shock after a diving accident has arrived in the emergency department. A cervical collar is in place. Which actions should the nurse take (select all that apply)? a. Prepare to administer atropine IV. b. Obtain baseline body temperature. c. Infuse large volumes of lactated Ringer's solution. d. Provide high-flow O2 (100%) by nonrebreather mask. e. Prepare for emergent intubation and mechanical ventilation.

ANS: A, B, D, E All of the actions are appropriate except to give large volumes of lactated Ringer's solution. The patient with neurogenic shock usually has a normal blood volume, and it is important not to volume overload the patient. In addition, lactated Ringer's solution is used cautiously in all shock situations because an ischemic liver cannot convert lactate to bicarbonate. DIF: Cognitive Level: Apply (application)

During assessment of the patient with fibromyalgia, the nurse would expect the patient to report which of the following (select all that apply)? a. Sleep disturbances b. Multiple tender points c. Cardiac palpitations and dizziness d. Multijoint inflammation and swelling e. Widespread bilateral, burning musculoskeletal pain

ANS: A, B, E These symptoms are commonly described by patients with fibromyalgia. Cardiac involvement and joint inflammation are not typical of fibromyalgia. DIF: Cognitive Level: Understand (comprehension) REF: 1546 TOP: Nursing Process: Assessment MSC: NCLEX: Physiological Integrity

Which information will be included when the nurse is teaching self-management to a patient who is receiving peritoneal dialysis (select all that apply)? a. Avoid commercial salt substitutes. b. Restrict fluid intake to 1000 mL daily. c. Take phosphate binders with each meal. d. Choose high-protein foods for most meals. e. Have several servings of dairy products daily.

ANS: A, C, D Patients who are receiving peritoneal dialysis should have a high-protein diet. Phosphate binders are taken with meals to help control serum phosphate and calcium levels. Commercial salt substitutes are high in potassium and should be avoided. Fluid intake is not limited unless weight and blood pressure are not controlled. Dairy products are high in phosphate and usually are limited. DIF: Cognitive Level: Apply (application) REF: 1087 TOP: Nursing Process: Planning MSC: NCLEX: Physiological Integrity

A patient who has been treated for status epilepticus in the emergency department will be transferred to the medical nursing unit. Which equipment should the nurse have available in the patient's assigned room (select all that apply)? a. Side-rail pads d. Suction tubing b. Tongue blade e. Urinary catheter c. Oxygen mask f. Nasogastric tube

ANS: A, C, D The patient is at risk for further seizures, and O2 and suctioning may be needed after any seizures to clear the airway and maximize oxygenation. The bed's side rails should be padded to minimize the risk for patient injury during a seizure. Use of tongue blades during a seizure is contraindicated. Insertion of a nasogastric (NG) tube is not indicated because the airway problem is not caused by vomiting or abdominal distention. A urinary catheter is not required unless there is urinary retention. DIF: Cognitive Level: Apply (application) REF: 1381 TOP: Nursing Process: Planning MSC: NCLEX: Physiological Integrity

When caring for a patient who experienced a T2 spinal cord transection 24 hours ago, which collaborative and nursing actions will the nurse include in the plan of care (select all that apply)? a. Urinary catheter care b. Nasogastric (NG) tube feeding c. Continuous cardiac monitoring d. Administration of H2 receptor blockers e. Maintenance of a warm room temperature

ANS: A, C, D, E The patient is at risk for bradycardia and poikilothermia caused by sympathetic nervous system dysfunction and should have continuous cardiac monitoring and maintenance of a relatively warm room temperature. To avoid bladder distention, a urinary retention catheter is used during this acute phase. Stress ulcers are a common complication, but can be avoided through the use of the H2 receptor blockers such as famotidine. Gastrointestinal motility is decreased initially, and NG suctioning is indicated. DIF: Cognitive Level: Apply (application)

Which statements will the nurse include when teaching a patient who is scheduled for oral glucose tolerance testing in the outpatient clinic (select all that apply)? a. "You will need to avoid smoking before the test." b. "Exercise should be avoided until the testing is complete." c. "Several blood samples will be obtained during the testing." d. "You should follow a low-calorie diet the day before the test." e. "The test requires that you fast for at least 8 hours before testing."

ANS: A, C, E Smoking may affect the results of oral glucose tolerance tests. Blood samples are obtained at baseline and at 30, 60, and 120 minutes. Accuracy requires that the patient be fasting before the test. The patient should consume at least 1500 calories/day for 3 days before the test. The patient should be ambulatory and active for accurate test results. DIF: Cognitive Level: Apply (application) REF: 1118 TOP: Nursing Process: Planning MSC: NCLEX: Physiological Integrity

Which actions will the nurse include in the plan of care for a patient with metastatic bone cancer of the left femur (select all that apply)? a. Monitor serum calcium. b. Teach about the need for strict bed rest. c. Discontinue use of sustained-release opioids. d. Support the left leg when repositioning the patient. e. Support family and patient as they discuss the prognosis.

ANS: A, D, E The nurse will monitor for hypercalcemia caused by bone decalcification. Support of the leg helps reduce the risk for pathologic fractures. Although the patient may be reluctant to exercise, activity is important to maintain function and avoid complications associated with immobility. Adequate pain medication, including sustained-release and rapid-acting opioids, is needed for the severe pain often associated with bone cancer. The prognosis for metastatic bone cancer is poor so the patient and family need to be supported as they deal with the reality of the situation. DIF: Cognitive Level: Apply (application) REF: 1501 TOP: Nursing Process: Planning MSC: NCLEX: Physiological Integrity

4. Which information in a 67-year-old woman's health history will alert the nurse to the need for a more focused assessment of the musculoskeletal system? a.The patient sprained her ankle at age 13. b.The patient's mother became shorter with aging. c.The patient takes ibuprofen (Advil) for occasional headaches. d.The patient's father died of complications of miliary tuberculosis.

ANS: B A family history of height loss with aging may indicate osteoporosis, and the nurse should perform a more thorough assessment of the patient's current height and other risk factors for osteoporosis. A sprained ankle during adolescence does not place the patient at increased current risk for musculoskeletal problems. A family history of tuberculosis is not a risk factor. Occasional nonsteroidal antiinflammatory drug (NSAID) use does not indicate any increased musculoskeletal risk.

The nurse is planning postoperative care for a patient who is being admitted to the surgical unit from the recovery room after transsphenoidal resection of a pituitary tumor. Which nursing action should be included? a. Palpate extremities for edema. b. Measure urine volume every hour. c. Check hematocrit every 2 hours for 8 hours. d. Monitor continuous pulse oximetry for 24 hours.

ANS: B After pituitary surgery, the patient is at risk for diabetes insipidus caused by cerebral edema. Monitoring of urine output and urine specific gravity is essential. Hemorrhage is not a common problem. There is no need to check the hematocrit hourly. The patient is at risk for dehydration, not volume overload. The patient is not at high risk for problems with oxygenation, and continuous pulse oximetry is not needed. DIF: Cognitive Level: Apply (application) REF: 1159 TOP: Nursing Process: Planning MSC: NCLEX: Physiological Integrity

A 42-yr-old patient admitted with acute kidney injury due to dehydration has oliguria, anemia, and hyperkalemia. Which prescribed action should the nurse take first? a. Insert a urinary retention catheter. b. Place the patient on a cardiac monitor. c. Administer epoetin alfa (Epogen, Procrit). d. Give sodium polystyrene sulfonate (Kayexalate).

ANS: B Because hyperkalemia can cause fatal cardiac dysrhythmias, the initial action should be to monitor the cardiac rhythm. Kayexalate and Epogen will take time to correct the hyperkalemia and anemia. The catheter allows monitoring of the urine output but does not correct the cause of the renal failure. DIF: Cognitive Level: Analyze (analysis) REF: 1073 OBJ: Special Questions: Prioritization TOP: Nursing Process: Implementation MSC: NCLEX: Physiological Integrity

A 30-yr-old patient has a new diagnosis of type 2 diabetes. The nurse will discuss the need to schedule a dilated eye examination a. every 2 years. c. when the patient is 39 years old. b. as soon as possible. d. within the first year after diagnosis.

ANS: B Because many patients have some diabetic retinopathy when they are first diagnosed with type 2 diabetes, a dilated eye examination is recommended at the time of diagnosis and annually thereafter. Patients with type 1 diabetes should have dilated eye examinations starting 5 years after they are diagnosed and then annually. DIF: Cognitive Level: Apply (application) REF: 1149 TOP: Nursing Process: Planning MSC: NCLEX: Physiological Integrity

2. A patient with left knee pain is diagnosed with bursitis. The nurse will explain that bursitis is an inflammation of a.the synovial membrane that lines the joint. b.a small, fluid-filled sac found at some joints. c.the fibrocartilage that acts as a shock absorber in the knee joint. d.any connective tissue that is found supporting the joints of the body.

ANS: B Bursae are fluid-filled sacs that cushion joints and bony prominences. Fibrocartilage is a solid tissue that cushions some joints. Bursae are a specific type of connective tissue. The synovial membrane lines many joints but is not a bursa.

A female patient with chronic kidney disease (CKD) is receiving peritoneal dialysis with 2-L inflows. Which information should the nurse report promptly to the health care provider? a. The patient has an outflow volume of 1800 mL. b. The patient's peritoneal effluent appears cloudy. c. The patient's abdomen appears bloated after the inflow. d. The patient has abdominal pain during the inflow phase.

ANS: B Cloudy-appearing peritoneal effluent is a sign of peritonitis and should be reported immediately so that treatment with antibiotics can be started. The other problems can be addressed through nursing interventions such as slowing the inflow and repositioning the patient. DIF: Cognitive Level: Apply (application) REF: 1087 TOP: Nursing Process: Assessment MSC: NCLEX: Physiological Integrity

A patient gives the admitting nurse health information before a scheduled intravenous pyelogram (IVP). Which item requires the nurse to intervene before the procedure? a. The patient has not had food or drink for 8 hours. b. The patient lists allergies to shellfish and penicillin. c. The patient complains of costovertebral angle (CVA) tenderness. d. The patient used a bisacodyl (Dulcolax) tablet the previous night.

ANS: B Iodine-based contrast dye is used during IVP and for many CT scans. The nurse will need to notify the health care provider before the procedures so that the patient can receive medications such as antihistamines or corticosteroids before the procedures are started. The other information is also important to note and document but does not have immediate implications for the patient's care during the procedures. DIF: Cognitive Level: Apply (application) REF: 1027 TOP: Nursing Process: Assessment MSC: NCLEX: Physiological Integrity

A patient who had radical neck surgery to remove a malignant tumor developed hypoparathyroidism. The nurse should plan to teach the patient about a. bisphosphonates to reduce bone demineralization. b. calcium supplements to normalize serum calcium levels. c. increasing fluid intake to decrease risk for nephrolithiasis. d. including whole grains in the diet to prevent constipation.

ANS: B Oral calcium supplements are used to maintain the serum calcium in normal range and prevent the complications of hypocalcemia. Whole grain foods decrease calcium absorption and will not be recommended. Bisphosphonates will lower serum calcium levels further by preventing calcium from being reabsorbed from bone. Kidney stones are not a complication of hypoparathyroidism and low calcium levels. DIF: Cognitive Level: Apply (application) REF: 1174 TOP: Nursing Process: Planning MSC: NCLEX: Physiological Integrity

An assessment finding for a 55-yr-old patient that alerts the nurse to the presence of osteoporosis is a. bowed legs. b. a loss of height. c. the report of frequent falls. d. an aversion to dairy products.

ANS: B Osteoporosis occurring in the vertebrae produces a gradual loss of height. Bowed legs are associated with osteomalacia. Low intake of dairy products is a risk factor for osteoporosis, but it does not indicate osteoporosis is present. Frequent falls increase the risk for fractures but are not an indicator of osteoporosis. DIF: Cognitive Level: Understand (comprehension) REF: 1511 TOP: Nursing Process: Assessment MSC: NCLEX: Physiological Integrity

The nurse completing a physical assessment for a newly admitted patient is unable to feel either kidney on palpation. Which action should the nurse take? a. Obtain a urine specimen to check for hematuria. b. Document the information on the assessment form. c. Ask the patient about any history of recent sore throat. d. Ask the health care provider about scheduling a renal ultrasound.

ANS: B The kidneys are protected by the abdominal organs, ribs, and muscles of the back and may not be palpable under normal circumstances, so no action except to document the assessment information is needed. Asking about a recent sore throat, checking for hematuria, or obtaining a renal ultrasound may be appropriate when assessing for renal problems for some patients, but there is nothing in the question stem to indicate that they are appropriate for this patient. DIF: Cognitive Level: Apply (application) REF: 1023 TOP: Nursing Process: Assessment MSC: NCLEX: Physiological Integrity

Which laboratory value reported to the nurse by the unlicensed assistive personnel (UAP) indicates an urgent need for the nurse's assessment of the patient? a. Bedtime glucose of 140 mg/dL b. Noon blood glucose of 52 mg/dL c. Fasting blood glucose of 130 mg/dL d. 2-hr postprandial glucose of 220 mg/dL

ANS: B The nurse should assess the patient with a blood glucose level of 52 mg/dL for symptoms of hypoglycemia and give the patient a carbohydrate-containing beverage such as orange juice. The other values are within an acceptable range or not immediately dangerous for a patient with diabetes. DIF: Cognitive Level: Apply (application) REF: 1152 TOP: Nursing Process: Assessment MSC: NCLEX: Physiological Integrity

1. A 42-year-old male patient complains of shoulder pain when the nurse moves his arm behind the back. Which question should the nurse ask? a."Are you able to feed yourself without difficulty?" b."Do you have difficulty when you are putting on a shirt?" c."Are you able to sleep through the night without waking?" d."Do you ever have trouble lowering yourself to the toilet?"

ANS: B The patient's pain will make it more difficult to accomplish tasks like putting on a shirt or jacket. This pain should not affect the patient's ability to feed himself or use the toilet because these tasks do not involve moving the arm behind the patient. The arm will not usually be positioned behind the patient during sleeping.

A hospitalized patient complains of a bilateral headache (4/10 on the pain scale) that radiates from the base of the skull. Which prescribed PRN medications should the nurse administer initially? a. Lorazepam (Ativan) c. Morphine sulfate (MS Contin) b. Acetaminophen (Tylenol) d. Butalbital and aspirin (Fiorinal)

ANS: B The patient's symptoms are consistent with a tension headache, and initial therapy usually involves a nonopioid analgesic such as acetaminophen, which is sometimes combined with a sedative or muscle relaxant. Lorazepam may be used in conjunction with acetaminophen but would not be appropriate as the initial monotherapy. Morphine sulfate and butalbital and aspirin would be more appropriate for a headache that did not respond to a nonopioid analgesic. DIF: Cognitive Level: Analyze (analysis) REF: 1372 OBJ: Special Questions: Prioritization TOP: Nursing Process: Implementation MSC: NCLEX: Physiological Integrity

A patient hospitalized with polymyositis has joint pain; erythematous facial rash; eyelid edema; and a weak, hoarse voice. The safety priority for the patient is addressing the a. acute pain. c. disturbed visual perception. b. risk for aspiration. d. risk for impaired skin integrity.

ANS: B The patient's vocal weakness and hoarseness indicate weakness of the pharyngeal muscles and a high risk for aspiration. The other concerns are also appropriate but are not as high a priority as the maintenance of the patient's airway. DIF: Cognitive Level: Analyze (analysis) REF: 1545 OBJ: Special Questions: Prioritization TOP: Nursing Process: Planning MSC: NCLEX: Physiological Integrity

During the nurse's physical examination of a young adult, the patient's thyroid gland cannot be felt. The most appropriate action by the nurse is to a. palpate the patient's neck more deeply. b. document that the thyroid was nonpalpable. c. notify the health care provider immediately. d. teach the patient about thyroid hormone testing.

ANS: B The thyroid is frequently nonpalpable. The nurse should simply document the finding. There is no need to notify the health care provider immediately about a normal finding. There is no indication for thyroid-stimulating hormone (TSH) testing unless there is evidence of thyroid dysfunction. Deep palpation of the neck is not appropriate. DIF: Cognitive Level: Apply (application) REF: 1112 TOP: Nursing Process: Assessment MSC: NCLEX: Physiological Integrity

When obtaining a health history and physical assessment for a 36-yr-old female patient with possible multiple sclerosis (MS), the nurse should a. assess for the presence of chest pain. b. inquire about urinary tract problems. c. inspect the skin for rashes or discoloration. d. ask the patient about any increase in libido.

ANS: B Urinary tract problems with incontinence or retention are common symptoms of MS. Chest pain and skin rashes are not symptoms of MS. A decrease in libido is common with MS. DIF: Cognitive Level: Apply (application) REF: 1384 TOP: Nursing Process: Assessment MSC: NCLEX: Physiological Integrity

A patient who was admitted with myxedema coma and diagnosed with hypothyroidism is improving. Discharge is expected to occur in 2 days. Which teaching strategy is likely to result in effective patient self-management at home? a. Delay teaching until closer to discharge date. b. Provide written reminders of information taught. c. Offer multiple options for management of therapies. d. Ensure privacy for teaching by asking the family to leave.

ANS: B Written instructions will be helpful to the patient because initially the hypothyroid patient may be unable to remember to take medications and other aspects of self-care. Because the treatment regimen is somewhat complex, teaching should be initiated well before discharge. Family members or friends should be included in teaching because the hypothyroid patient is likely to forget some aspects of the treatment plan. A simpler regimen will be easier to understand until the patient is euthyroid. DIF: Cognitive Level: Apply (application) REF: 1170 TOP: Nursing Process: Planning MSC: NCLEX: Physiological Integrity

A 39-yr-old patient with a suspected herniated intervertebral disc is scheduled for a myelogram. Which information communicated by the nurse to the health care provider before the procedure would change the procedural plans? a. The patient is anxious about the test results. b. The patient reports a previous allergy to shellfish. c. The patient has back pain when lying flat for more than 4 hours. d. The patient drank apple juice 4 hours before the scheduled procedure.

ANS: B A contrast medium containing iodine is injected into the subarachnoid space during a myelogram. The patient's allergy would contraindicate the use of this medium. The health care provider may need to modify the orders to prevent back pain, but this can be done after the procedure. Clear liquids are usually considered safe up to 4 hours before a diagnostic or surgical procedure. The patient's anxiety should be addressed, but procedural plans would not need to be changed. DIF: Cognitive Level: Apply (application) REF: 1310 TOP: Nursing Process: Assessment MSC: NCLEX: Physiological Integrity

The nurse is caring for a 45-yr-old male patient during a water deprivation test. Which finding is most important for the nurse to communicate to the health care provider? a. The patient complains of intense thirst. b. The patient has a 5-lb (2.3-kg) weight loss. c. The patient's urine osmolality does not increase. d. The patient feels dizzy when sitting on the edge of the bed.

ANS: B A drop in the weight of more than 2 kg indicates severe dehydration, and the test should be discontinued. The other assessment data are not unusual with this test. DIF: Cognitive Level: Analyze (analysis) REF: 1115 OBJ: Special Questions: Prioritization TOP: Nursing Process: Assessment MSC: NCLEX: Physiological Integrity

Which information in a 67-yr-old woman's health history will alert the nurse to the need for a more focused assessment of the musculoskeletal system? a. The patient sprained her ankle at age 13. b. The patient's mother became shorter with aging. c. The patient takes ibuprofen for occasional headaches. d. The patient's father died of complications of miliary tuberculosis.

ANS: B A family history of height loss with aging may indicate osteoporosis, and the nurse should perform a more thorough assessment of the patient's current height and other risk factors for osteoporosis. A sprained ankle during adolescence does not place the patient at increased current risk for musculoskeletal problems. A family history of tuberculosis is not a risk factor. Occasional nonsteroidal antiinflammatory drug (NSAID) use does not indicate any increased musculoskeletal risk. DIF: Cognitive Level: Apply (application) REF: 1452 TOP: Nursing Process: Assessment MSC: NCLEX: Health Promotion and Maintenance

Which information will the nurse include when teaching a patient with newly diagnosed systemic exertion intolerance disease (SEID) about self-management? a. Symptoms usually progress as patients become older. b. A gradual increase in daily exercise may help decrease fatigue. c. Avoid use of over-the-counter antihistamines or decongestants. d. A low-residue, low-fiber diet will reduce any abdominal distention.

ANS: B A graduated exercise program is recommended to avoid fatigue while encouraging ongoing activity. Because many patients with SEID syndrome have allergies, antihistamines and decongestants are used to treat allergy symptoms. A high-fiber diet is recommended. SEID usually does not progress. DIF: Cognitive Level: Apply (application) REF: 1548 TOP: Nursing Process: Planning MSC: NCLEX: Physiological Integrity

The day shift nurse at the long-term care facility learns that a patient with dementia experienced sundowning late in the afternoon on the previous two days. Which action should the nurse take? a. Have the patient take a mid-morning nap. b. Keep window blinds open during the day. c. Provide hourly orientation to time and place. d. Move the patient to a quiet room in the afternoon.

ANS: B A likely cause of sundowning is a disruption in circadian rhythms, and keeping the patient active and in daylight will help reestablish a more normal circadian pattern. Moving the patient to a different room might increase confusion. Taking a nap will interfere with nighttime sleep. Hourly orientation will not be helpful in a patient with dementia. DIF: Cognitive Level: Apply (application) REF: 1411 TOP: Nursing Process: Implementation MSC: NCLEX: Psychosocial Integrity

After surgery for an abdominal aortic aneurysm, a patient's central venous pressure (CVP) monitor indicates low pressures. Which action should the nurse take? a. Administer IV diuretic medications. b. Increase the IV fluid infusion per protocol. c. Increase the infusion rate of IV vasodilators. d. Elevate the head of the patient's bed to 45 degrees.

ANS: B A low CVP indicates hypovolemia and a need for an increase in the infusion rate. Diuretic administration will contribute to hypovolemia and elevation of the head or increasing vasodilators may decrease cerebral perfusion. DIF: Cognitive Level: Apply (application)

Which action will the emergency department nurse anticipate for a patient diagnosed with a concussion who did not lose consciousness? a. Coordinate the transfer of the patient to the operating room. b. Provide discharge instructions about monitoring neurologic status. c. Transport the patient to radiology for magnetic resonance imaging (MRI). d. Arrange to admit the patient to the neurologic unit for 24 hours of observation.

ANS: B A patient with a minor head trauma is usually discharged with instructions about neurologic monitoring and the need to return if neurologic status deteriorates. MRI, hospital admission, and surgery are not usually indicated in a patient with a concussion. DIF: Cognitive Level: Apply (application)

Which problem can the nurse expect for a patient who has a positive Romberg test result? a. Pain b. Falls c. Aphasia d. Confusion

ANS: B A positive Romberg test result indicates that the patient has difficulty maintaining balance when standing with the eyes closed. The Romberg does not test for orientation, thermoregulation, or discomfort. DIF: Cognitive Level: Apply (application) REF: 1307 TOP: Nursing Process: Planning MSC: NCLEX: Physiological Integrity

Which nursing action is essential for a patient immediately after a renal biopsy? a. Insert a urinary catheter and test urine for microscopic hematuria. b. Apply a pressure dressing and keep the patient on the affected side. c. Check blood glucose to assess for hyperglycemia or hypoglycemia. d. Monitor blood urea nitrogen (BUN) and creatinine to assess renal function.

ANS: B A pressure dressing is applied, and the patient is kept on the affected side for 30 to 60 minutes to put pressure on the biopsy side and decrease the risk for bleeding. The blood glucose and BUN/creatinine will not be affected by the biopsy. Although monitoring for hematuria is needed, there is no need for catheterization. DIF: Cognitive Level: Apply (application) REF: 1020 TOP: Nursing Process: Implementation MSC: NCLEX: Physiological Integrity

A patient who has amyotrophic lateral sclerosis (ALS) is hospitalized with pneumonia. Which nursing action will be included in the plan of care? a. Observe for agitation and paranoia. b. Assist with active range of motion (ROM). c. Give muscle relaxants as needed to reduce spasms. d. Use simple words and phrases to explain procedures.

ANS: B ALS causes progressive muscle weakness, but assisting the patient to perform active ROM will help maintain strength as long as possible. Psychotic manifestations such as agitation and paranoia are not associated with ALS. Cognitive function is not affected by ALS, and the patient's ability to understand procedures will not be impaired. Muscle relaxants will further increase muscle weakness and depress respirations. DIF: Cognitive Level: Apply (application) REF: 1395 TOP: Nursing Process: Planning MSC: NCLEX: Physiological Integrity

A 40-yr-old patient with suspected acromegaly is seen at the clinic. To assist in making the diagnosis, which question should the nurse ask? a. "Have you had a recent head injury?" b. "Do you have to wear larger shoes now?" c. "Is there a family history of acromegaly?" d. "Are you experiencing tremors or anxiety?"

ANS: B Acromegaly causes an enlargement of the hands and feet. Head injury and family history are not risk factors for acromegaly. Tremors and anxiety are not clinical manifestations of acromegaly. DIF: Cognitive Level: Apply (application) REF: 1157 TOP: Nursing Process: Assessment MSC: NCLEX: Physiological Integrity

A 29-yr-old woman with systemic lupus erythematosus has been prescribed 2 weeks of high-dose prednisone therapy. Which information about the prednisone is most important for the nurse to include? a. "Weigh yourself daily to monitor for weight gain." b. "The prednisone dose should be decreased gradually." c. "A weight-bearing exercise program will help minimize risk for osteoporosis." d. "Call the health care provider if you have mood changes with the prednisone."

ANS: B Acute adrenal insufficiency may occur if exogenous corticosteroids are suddenly stopped. Mood alterations and weight gain are possible adverse effects of corticosteroid use, but these are not life-threatening effects. Osteoporosis occurs when patients take corticosteroids for longer periods. DIF: Cognitive Level: Analyze (analysis) REF: 1177 TOP: Nursing Process: Implementation MSC: NCLEX: Physiological Integrity

A hospitalized patient with a history of cluster headache awakens during the night with a severe stabbing headache. Which action should the nurse take first? a. Put a moist hot pack on the patient's neck. b. Start the prescribed PRN O2 at 6 L/min. c. Give the ordered PRN acetaminophen (Tylenol). d. Notify the patient's health care provider immediately.

ANS: B Acute treatment for cluster headache is administration of 100% O2 at 6 to 8 L/min. If the patient obtains relief with the O2, there is no immediate need to notify the health care provider. Cluster headaches last only 60 to 90 minutes, so oral pain medications have minimal effect. Hot packs are helpful for tension headaches but are not as likely to reduce pain associated with a cluster headache. DIF: Cognitive Level: Analyze (analysis) REF: 1372 OBJ: Special Questions: Prioritization TOP: Nursing Process: Implementation MSC: NCLEX: Physiological Integrity

Norepinephrine has been prescribed for a patient who was admitted with dehydration and hypotension. Which patient data indicate that the nurse should consult with the health care provider before starting the norepinephrine? a. The patient is receiving low dose dopamine. b. The patient's central venous pressure is 3 mm Hg. c. The patient is in sinus tachycardia at 120 beats/min. d. The patient has had no urine output since being admitted.

ANS: B Adequate fluid administration is essential before giving vasopressors to patients with hypovolemic shock. The patient's low central venous pressure indicates a need for more volume replacement. The other patient data are not contraindications to norepinephrine administration. DIF: Cognitive Level: Apply (application)

Which action will the nurse include in the plan of care for a patient with a new diagnosis of rheumatoid arthritis (RA)? a. Instruct the patient to purchase a soft mattress. b. Encourage the patient to take a nap in the afternoon. c. Teach the patient to use lukewarm water when bathing. d. Suggest exercise with light weights several times daily.

ANS: B Adequate rest helps decrease the fatigue and pain associated with RA. Patients are taught to avoid stressing joints, use warm baths to relieve stiffness, and use a firm mattress. When the disease is stabilized, a therapeutic exercise program is usually developed by a physical therapist to include exercises that improve flexibility and strength of affected joints, as well as the patient's general endurance. DIF: Cognitive Level: Apply (application) REF: 1531 TOP: Nursing Process: Planning MSC: NCLEX: Physiological Integrity

A patient is admitted with diabetes insipidus. Which action will be appropriate for the registered nurse (RN) to delegate to an experienced licensed practical/vocational nurse (LPN/LVN)? a. Titrate the infusion of 5% dextrose in water. b. Administer prescribed subcutaneous DDAVP. c. Assess the patient's overall hydration status every 8 hours. d. Teach the patient how to use desmopressin (DDAVP) nasal spray.

ANS: B Administration of medications is included in LPN/LVN education and scope of practice. Assessments, patient teaching, and titrating fluid infusions are more complex skills and should be done by the RN. DIF: Cognitive Level: Apply (application) REF: 1161 OBJ: Special Questions: Delegation TOP: Nursing Process: Implementation MSC: NCLEX: Safe and Effective Care Environment

An unconscious male patient has just arrived in the emergency department with a head injury caused by a motorcycle crash. Which order should the nurse question? a. Obtain x-rays of the skull and spine. b. Prepare the patient for lumbar puncture. c. Send for computed tomography (CT) scan. d. Perform neurologic checks every 15 minutes.

ANS: B After a head injury, the patient may be experiencing intracranial bleeding and increased intracranial pressure, and herniation of the brain could result if lumbar puncture is performed. The other orders are appropriate. DIF: Cognitive Level: Apply (application) REF: 1310 TOP: Nursing Process: Implementation MSC: NCLEX: Physiological Integrity

Which statement by patient who is being discharged from the emergency department (ED) after a concussion indicates a need for intervention by the nurse? a. "I will return if I feel dizzy or nauseated." b. "I am going to drive home and go to bed." c. "I do not even remember being in an accident." d. "I can take acetaminophen (Tylenol) for my headache."

ANS: B After a head injury, the patient should avoid driving and operating heavy machinery. Retrograde amnesia is common after a concussion. The patient can take acetaminophen for headache and should return if symptoms of increased intracranial pressure such as dizziness or nausea occur. DIF: Cognitive Level: Apply (application)

A patient in the clinic reports a recent episode of dysphasia and left-sided weakness at home that resolved after 2 hours. The nurse will anticipate teaching the patient about a. tPA. c. warfarin (Coumadin). b. aspirin . d. nimodipine

ANS: B After a transient ischemic attack, patients typically are started on medications such as aspirin to inhibit platelet function and decrease stroke risk. tPA is used for acute ischemic stroke. Coumadin is usually used for patients with atrial fibrillation. Nimodipine is used to prevent cerebral vasospasm after a subarachnoid hemorrhage. DIF: Cognitive Level: Apply (application) REF: 1353 TOP: Nursing Process: Planning MSC: NCLEX: Physiological Integrity

Which information about a 76-yr-old patient should the nurse report as uncharacteristic of normal aging? a. Triceps reflex response graded at 1/5 b. Unintended weight loss of 15 pounds c. 10 mm Hg orthostatic drop in systolic blood pressure d. Patient complaint of chronic difficulty in falling asleep

ANS: B Although changes in appetite are normal with aging, a 15-lb weight loss requires further investigation. Orthostatic drops in blood pressure, changes in sleep patterns, and slowing of reflexes are normal changes in aging. DIF: Cognitive Level: Apply (application) REF: 1301 TOP: Nursing Process: Assessment MSC: NCLEX: Health Promotion and Maintenance

Before administration of captopril to a patient with stage 2 chronic kidney disease (CKD), the nurse will check the patient's a. glucose.. b. potassium. c. creatinine d. phosphate.

ANS: B Angiotensin-converting enzyme (ACE) inhibitors are frequently used in patients with CKD because they delay the progression of the CKD, but they cause potassium retention. Therefore careful monitoring of potassium levels is needed in patients who are at risk for hyperkalemia. The other laboratory values would also be monitored in patients with CKD but would not affect whether the captopril was given or not. DIF: Cognitive Level: Apply (application) REF: 1075 TOP: Nursing Process: Assessment MSC: NCLEX: Physiological Integrity

The nurse is administering a mental status examination to a patient who has hypertension. The nurse suspects depression when the patient responds to the nurse's questions with a. "Is that right?" c. "Wait, let me think about that." b. "I don't know." d. "Who are those people over there?"

ANS: B Answers such as "I don't know" are more typical of depression than dementia. The response "Who are those people over there?" is more typical of the distraction seen in a patient with delirium. The remaining two answers are more typical of a patient with mild to moderate dementia. DIF: Cognitive Level: Apply (application) REF: 1400 TOP: Nursing Process: Assessment MSC: NCLEX: Psychosocial Integrity

After change-of-shift report in the progressive care unit, who should the nurse care for first? a. Patient who had an inferior myocardial infarction 2 days ago and has crackles in the lung bases b. Patient with suspected urosepsis who has new orders for urine and blood cultures and antibiotics c. Patient who had a T5 spinal cord injury 1 week ago and currently has a heart rate of 54 beats/minute d. Patient admitted with anaphylaxis 3 hours ago who now has clear lung sounds and a blood pressure of 108/58 mm Hg

ANS: B Antibiotics should be given within the first hour for patients who have sepsis or suspected sepsis in order to prevent progression to systemic inflammatory response syndrome and septic shock. The data on the other patients indicate that they are more stable. Crackles heard only at the lung bases do not require immediate intervention in a patient who has had a myocardial infarction. Mild bradycardia does not usually require atropine in patients who have a spinal cord injury. The findings for the patient admitted with anaphylaxis indicate resolution of bronchospasm and hypotension. DIF: Cognitive Level: Analyze (analysis)

Which finding is the best indicator that the fluid resuscitation for a 90-kg patient with hypovolemic shock has been effective? a. Hemoglobin is within normal limits. b. Urine output is 65 mL over the past hour. c. Central venous pressure (CVP) is normal. d. Mean arterial pressure (MAP) is 72 mm Hg.

ANS: B Assessment of end organ perfusion, such as an adequate urine output, is the best indicator that fluid resuscitation has been successful. Urine output should be equal to or more than 0.5 mL/kg/hr. The hemoglobin level, CVP, and MAP are useful in determining the effects of fluid administration, but they are not as useful as data indicating good organ perfusion. DIF: Cognitive Level: Analyze (analysis)

Several patients have been hospitalized for diagnosis of neurologic problems. Which patient will the nurse assess first? a. A patient with a transient ischemic attack (TIA) returning from carotid duplex studies b. A patient with a brain tumor who has just arrived on the unit after a cerebral angiogram c. A patient with a seizure disorder who has just completed an electroencephalogram (EEG) d. A patient prepared for a lumbar puncture whose health care provider is waiting for assistance

ANS: B Because cerebral angiograms require insertion of a catheter into the femoral artery, bleeding is a possible complication. The nurse will need to check the pulse, blood pressure, and the catheter insertion site in the groin as soon as the patient arrives. Carotid duplex studies and EEG are noninvasive. The nurse will need to assist with the lumbar puncture as soon as possible, but monitoring for hemorrhage after cerebral angiogram has a higher priority. DIF: Cognitive Level: Analyze (analysis) REF: 1310 OBJ: Special Questions: Prioritization | Special Questions: Multiple Patients TOP: Nursing Process: Planning MSC: NCLEX: Physiological Integrity

A patient with psoriatic arthritis and back pain is receiving etanercept (Enbrel). Which finding is most important for the nurse to report to the health care provider? a. Red, scaly patches are noted on the arms. b. Crackles are auscultated in the lung bases. c. Hemoglobin is 11.1g/dL, and hematocrit is 35%. d. Patient has continued pain after first week of etanercept therapy.

ANS: B Because heart failure is a possible adverse effect of etanercept, the medication may need to be discontinued. The other information will also be reported to the health care provider but does not indicate a need for a change in treatment. Red, scaly patches of skin and mild anemia are commonly seen with psoriatic arthritis. Treatment with biologic therapies requires time to improve symptoms. DIF: Cognitive Level: Analyze (analysis) REF: 1537 OBJ: Special Questions: Prioritization TOP: Nursing Process: Assessment MSC: NCLEX: Physiological Integrity

The nurse determines additional instruction is needed when a patient diagnosed with scleroderma makes which statement? a. "Paraffin baths can be used to help my hands." b. "I should lie down for an hour after each meal." c. "Lotions will help if I rub them in for a long time." d. "I should perform range-of-motion exercises daily."

ANS: B Because of the esophageal scarring, patients should sit up for 2 hours after eating. The other patient statements are correct and indicate teaching has been effective. DIF: Cognitive Level: Apply (application) REF: 1544 TOP: Nursing Process: Evaluation MSC: NCLEX: Physiological Integrity

Which intervention will the nurse include in the plan of care for a patient who has cardiogenic shock? a. Check temperature every 2 hours. b. Monitor breath sounds frequently. c. Maintain patient in supine position. d. Assess skin for flushing and itching.

ANS: B Because pulmonary congestion and dyspnea are characteristics of cardiogenic shock, the nurse should assess the breath sounds frequently. The head of the bed is usually elevated to decrease dyspnea in patients with cardiogenic shock. Elevated temperature and flushing or itching of the skin are not typical of cardiogenic shock. DIF: Cognitive Level: Apply (application)

After endotracheal suctioning, the nurse notes that the intracranial pressure (ICP) for a patient with a traumatic head injury has increased from 14 to 17 mm Hg. Which action should the nurse take first? a. Document the increase in intracranial pressure. b. Ensure that the patient's neck is in neutral position. c. Notify the health care provider about the change in pressure. d. Increase the rate of the prescribed propofol (Diprivan) infusion.

ANS: B Because suctioning will cause a transient increase in ICP, the nurse should initially check for other factors that might be contributing to the increase and observe the patient for a few minutes. Documentation is needed, but this is not the first action. There is no need to notify the health care provider about this expected reaction to suctioning. Propofol is used to control patient anxiety or agitation. There is no indication that anxiety has contributed to the increase in ICP. DIF: Cognitive Level: Analyze (analysis)

After change-of-shift report, which patient should the nurse assess first? a. A 19-yr-old patient with type 1 diabetes who has a hemoglobin A1C of 12% b. A 23-yr-old patient with type 1 diabetes who has a blood glucose of 40 mg/dL c. A 40-yr-old patient who is pregnant and whose oral glucose tolerance test is 202 mg/dL d. A 50-yr-old patient who uses exenatide (Byetta) and is complaining of acute abdominal pain

ANS: B Because the brain requires glucose to function, untreated hypoglycemia can cause unconsciousness, seizures, and death. The nurse will rapidly assess and treat the patient with low blood glucose. The other patients also have symptoms that require assessments or interventions, but they are not at immediate risk for life-threatening complications. DIF: Cognitive Level: Analyze (analysis) REF: 1146 OBJ: Special Questions: Prioritization | Special Questions: Multiple Patients TOP: Nursing Process: Planning MSC: NCLEX: Safe and Effective Care Environment

Which finding would the nurse expect when assessing the legs of a patient who has a lower motor neuron lesion? a. Spasticity b. Flaccidity c. Impaired sensation d. Hyperactive reflexes

ANS: B Because the cell bodies of lower motor neurons are located in the spinal cord, damage to the neuron will decrease motor activity of the affected muscles. Spasticity and hyperactive reflexes are caused by upper motor neuron damage. Sensation is not impacted by motor neuron lesions. DIF: Cognitive Level: Understand (comprehension) REF: 1296 TOP: Nursing Process: Assessment MSC: NCLEX: Physiological Integrity

A patient with muscular dystrophy is hospitalized with pneumonia. Which nursing action will be included in the plan of care? a. Logroll the patient every 2 hours. b. Assist the patient with ambulation. c. Discuss the need for genetic testing with the patient. d. Teach the patient about the muscle biopsy procedure.

ANS: B Because the goal for the patient with muscular dystrophy is to keep the patient active for as long as possible, assisting the patient to ambulate will be part of the care plan. The patient will not require logrolling. Muscle biopsies are necessary to confirm the diagnosis but are not necessary for a patient who already has a diagnosis. There is no need for genetic testing because the patient already knows the diagnosis. DIF: Cognitive Level: Apply (application) REF: 1502 TOP: Nursing Process: Planning MSC: NCLEX: Physiological Integrity

Which question will the nurse ask a patient who has been admitted with a benign occipital lobe tumor to assess for functional deficits? a. "Do you have difficulty in hearing?" b. "Are you experiencing visual problems?" c. "Are you having any trouble with your balance?" d. "Have you developed any weakness on one side?"

ANS: B Because the occipital lobe is responsible for visual reception, the patient with a tumor in this area is likely to have problems with vision. The other questions will be better for assessing function of the temporal lobe, cerebellum, and frontal lobe. DIF: Cognitive Level: Apply (application)

Which nursing action will be most effective in ensuring daily medication compliance for a patient with mild dementia? a. Setting the medications up monthly in a medication box b. Having the patient's family member administer the medication c. Posting reminders to take the medications in the patient's house d. Calling the patient weekly with a reminder to take the medication

ANS: B Because the patient with mild dementia will have difficulty with learning new skills and forgetfulness, the most appropriate nursing action is to have someone else administer the drug. The other nursing actions will not be as effective in ensuring that the patient takes the medications. DIF: Cognitive Level: Analyze (analysis) REF: 1413 TOP: Nursing Process: Implementation MSC: NCLEX: Physiological Integrity

A patient with paraplegia resulting from a T9 spinal cord injury has a neurogenic reflexic bladder. Which action will the nurse include in the plan of care? a. Teach the patient the Credé method. b. Instruct the patient how to self-catheterize. c. Catheterize for residual urine after voiding. d. Assist the patient to the toilet every 2 hours.

ANS: B Because the patient's bladder is spastic and will empty in response to overstretching of the bladder wall, the most appropriate method is to avoid incontinence by emptying the bladder at regular intervals through intermittent catheterization. Assisting the patient to the toilet will not be helpful because the bladder will not empty. The Credé method is more appropriate for a bladder that is flaccid, such as occurs with areflexic neurogenic bladder. Catheterization after voiding will not resolve the patient's incontinence. DIF: Cognitive Level: Apply (application)

A patient admitted with acute respiratory failure has ineffective airway clearance related to thick secretions. Which nursing intervention would specifically address this patient problem? a. Encourage use of the incentive spirometer. b. Offer the patient fluids at frequent intervals. c. Teach the patient the importance of ambulation. d. Titrate oxygen level to keep O2 saturation above 93%.

ANS: B Because the reason for the poor airway clearance is the thick secretions, the best action will be to encourage the patient to improve oral fluid intake. Patients should be instructed to use the incentive spirometer on a regular basis (e.g., every hour) to facilitate the clearance of the secretions. The other actions may also be helpful in improving the patient's gas exchange, but they do not address the thick secretions that are causing the poor airway clearance. DIF: Cognitive Level: Apply (application)

A 48-yr-old male patient who weighs 242 lb (110 kg) undergoes a nephrectomy for massive kidney trauma from a motor vehicle crash. Which postoperative assessment finding is most important to communicate to the surgeon? a. Blood pressure is 102/58. b. Urine output is 20 mL/hr for 2 hours. c. Incisional pain level is reported as 9/10. d. Crackles are heard at bilateral lung bases.

ANS: B Because the urine output should be at least 0.5 mL/kg/hr, a 40-mL output for 2 hours indicates that the patient may have decreased renal perfusion because of bleeding, inadequate fluid intake, or obstruction at the suture site. The blood pressure requires ongoing monitoring but does not indicate inadequate perfusion at this time. The patient should cough and deep breathe, but the crackles do not indicate a need for an immediate change in therapy. The incisional pain should be addressed, but this is not as potentially life threatening as decreased renal perfusion. In addition, the nurse can medicate the patient for pain.

A patient with a possible pituitary adenoma is scheduled for a computed tomography (CT) scan with contrast media. Which patient information is important for the nurse to communicate to the health care provider before the test? a. Bilateral poor peripheral vision c. Recent weight loss of 20 lb b. Allergies to iodine and shellfish d. Complaint of ongoing headaches

ANS: B Because the usual contrast media is iodine-based, the health care provider will need to know about the allergy before the CT scan. The other findings are common with any mass in the brain such as a pituitary adenoma. DIF: Cognitive Level: Apply (application) REF: 1115 OBJ: Special Questions: Prioritization TOP: Nursing Process: Assessment MSC: NCLEX: Physiological Integrity

Which finding about a patient who is receiving vasopressin to treat septic shock indicates an immediate need for the nurse to report the finding to the health care provider? a. The patient's urine output is 18 mL/hr. b. The patient is complaining of chest pain. c. The patient's peripheral pulses are weak. d. The patient's heart rate is 110 beats/minute.

ANS: B Because vasopressin is a potent vasoconstrictor, it may decrease coronary artery perfusion. The other information is consistent with the patient's diagnosis, and should be reported to the health care provider but does not indicate an immediate need for a change in therapy. DIF: Cognitive Level: Apply (application)

The nurse is preparing to teach a 43-yr-old man who is newly diagnosed with type 2 diabetes about home management of the disease. Which action should the nurse take first? a. Ask the patient's family to participate in the diabetes education program. b. Assess the patient's perception of what it means to have diabetes mellitus. c. Demonstrate how to check glucose using capillary blood glucose monitoring. d. Discuss the need for the patient to actively participate in diabetes management.

ANS: B Before planning teaching, the nurse should assess the patient's interest in and ability to self-manage the diabetes. After assessing the patient, the other nursing actions may be appropriate, but planning needs to be individualized to each patient. DIF: Cognitive Level: Analyze (analysis) REF: 1139 OBJ: Special Questions: Prioritization TOP: Nursing Process: Planning MSC: NCLEX: Health Promotion and Maintenance

Which laboratory result will the nurse monitor to determine if prednisone has been effective for a patient with an acute exacerbation of rheumatoid arthritis? a. Blood glucose c. Serum electrolytes b. C-reactive protein d. Liver function tests

ANS: B C-reactive protein is a serum marker for inflammation, and a decrease would indicate the corticosteroid therapy was effective. Blood glucose and serum electrolytes will also be monitored to assess for side effects of prednisone. Liver function is not routinely monitored in patients receiving corticosteroids. DIF: Cognitive Level: Apply (application) REF: 1527 TOP: Nursing Process: Evaluation MSC: NCLEX: Physiological Integrity

Which information will the nurse monitor in order to determine the effectiveness of prescribed calcium carbonate (Caltrate) for a patient with chronic kidney disease (CKD)? a. Blood pressure b. Phosphate level c. Neurologic status d. Creatinine clearance

ANS: B Calcium carbonate is prescribed to bind phosphorus and prevent mineral and bone disease in patients with CKD. The other data will not be helpful in evaluating the effectiveness of calcium carbonate. DIF: Cognitive Level: Apply (application) REF: 1081 TOP: Nursing Process: Evaluation MSC: NCLEX: Physiological Integrity

A patient admitted with a diffuse axonal injury has a systemic blood pressure (BP) of 106/52 mm Hg and an intracranial pressure (ICP) of 14 mm Hg. Which action should the nurse take first? a. Document the BP and ICP in the patient's record. b. Report the BP and ICP to the health care provider. c. Elevate the head of the patient's bed to 60 degrees. d. Continue to monitor the patient's vital signs and ICP.

ANS: B Calculate the cerebral perfusion pressure (CPP): (CPP = Mean arterial pressure [MAP] - ICP). MAP = DBP + 1/3 (Systolic blood pressure [SBP] - Diastolic blood pressure [DBP]). Therefore the MAP is 70, and the CPP is 56 mm Hg, which are below the normal values of 60 to 100 mm Hg and are approaching the level of ischemia and neuronal death. Immediate changes in the patient's therapy such as fluid infusion or vasopressor administration are needed to improve the CPP. Adjustments in the head elevation should only be done after consulting with the health care provider. Continued monitoring and documentation will also be done, but they are not the first actions that the nurse should take. DIF: Cognitive Level: Analyze (analysis)

The nurse is assessing a male patient diagnosed with a pituitary tumor causing panhypopituitarism. Assessment findings consistent with panhypopituitarism include a. high blood pressure. c. elevated blood glucose. b. decreased facial hair. d. tachycardia and palpitations.

ANS: B Changes in male secondary sex characteristics such as decreased facial hair, testicular atrophy, diminished spermatogenesis, loss of libido, impotence, and decreased muscle mass are associated with decreases in follicle-stimulating hormone (FSH) and luteinizing hormone (LH). Fasting hypoglycemia and hypotension occur in panhypopituitarism as a result of decreases in adrenocorticotropic hormone (ACTH) and cortisol. Bradycardia is likely due to the decrease in thyroid-stimulating hormone (TSH) and thyroid hormones associated with panhypopituitarism. DIF: Cognitive Level: Apply (application) REF: 1158 TOP: Nursing Process: Assessment MSC: NCLEX: Physiological Integrity

How will the nurse assess for flank tenderness in a patient with suspected pyelonephritis? a. Palpate along both sides of the lumbar vertebral column. b. Strike a flat hand covering the costovertebral angle (CVA). c. Push fingers upward into the two lowest intercostal spaces. d. Percuss between the iliac crest and ribs at the midaxillary line.

ANS: B Checking for flank pain is best performed by percussion of the CVA and asking about pain. The other techniques would not assess for flank pain. DIF: Cognitive Level: Understand (comprehension) REF: 1023 TOP: Nursing Process: Assessment MSC: NCLEX: Physiological Integrity

The nurse will plan to teach a 27-yr-old woman who smokes two packs of cigarettes daily about the increased risk for a. kidney stones. b. bladder cancer. c. bladder infection. d. interstitial cystitis.

ANS: B Cigarette smoking is a risk factor for bladder cancer. The patient's risk for developing interstitial cystitis, urinary tract infection, or kidney stones will not be reduced by quitting smoking.

Which patient action indicates good understanding of the nurse's teaching about administration of aspart (NovoLog) insulin? a. The patient avoids injecting the insulin into the upper abdominal area. b. The patient cleans the skin with soap and water before insulin administration. c. The patient stores the insulin in the freezer after administering the prescribed dose. d. The patient pushes the plunger down while removing the syringe from the injection site.

ANS: B Cleaning the skin with soap and water is acceptable. Insulin should not be frozen. The patient should leave the syringe in place for about 5 seconds after injection to be sure that all the insulin has been injected. The upper abdominal area is one of the preferred areas for insulin injection. DIF: Cognitive Level: Apply (application) REF: 1128 TOP: Nursing Process: Evaluation MSC: NCLEX: Physiological Integrity

A 20-yr-old male patient is admitted with a head injury after a collision while playing football. After noting that the patient has developed clear nasal drainage, which action should the nurse take? a. Have the patient gently blow the nose. b. Check the drainage for glucose content. c. Teach the patient that rhinorrhea is expected after a head injury. d. Obtain a specimen of the fluid to send for culture and sensitivity.

ANS: B Clear nasal drainage in a patient with a head injury suggests a dural tear and cerebrospinal fluid (CSF) leakage. If the drainage is CSF, it will test positive for glucose. Fluid leaking from the nose will have normal nasal flora, so culture and sensitivity will not be useful. Blowing the nose is avoided to prevent CSF leakage. DIF: Cognitive Level: Apply (application)

When teaching about clopidogrel (Plavix), the nurse will tell the patient with cerebral atherosclerosis a. to monitor and record the blood pressure daily. b. to call the health care provider if stools are tarry. c. that clopidogrel will dissolve clots in the cerebral arteries. d. that clopidogrel will reduce cerebral artery plaque formation.

ANS: B Clopidogrel inhibits platelet function and increases the risk for gastrointestinal bleeding, so patients should be advised to notify the health care provider about any signs of bleeding. The medication does not lower blood pressure, decrease plaque formation, or dissolve clots. DIF: Cognitive Level: Apply (application) REF: 1353 TOP: Nursing Process: Implementation MSC: NCLEX: Physiological Integrity

A patient who has been in the intensive care unit for 4 days has disturbed sensory perception from sleep deprivation. Which action should the nurse include in the plan of care? a. Administer prescribed sedatives or opioids at bedtime to promote sleep. b. Cluster nursing activities so that the patient has uninterrupted rest periods. c. Silence the alarms on the cardiac monitors to allow 30- to 40-minute naps. d. Eliminate assessments between 2200 and 0600 to allow uninterrupted sleep.

ANS: B Clustering nursing activities and providing uninterrupted rest periods will minimize sleep-cycle disruption. Sedative and opioid medications tend to decrease the amount of rapid eye movement (REM) sleep and can contribute to sleep disturbance and disturbed sensory perception. Silencing the alarms on the cardiac monitors would be unsafe in a critically ill patient, as would discontinuing all assessments during the night. DIF: Cognitive Level: Apply (application)

Which information will the nurse include when preparing teaching materials for a patient with an exacerbation of rheumatoid arthritis? a. Affected joints should not be exercised when pain is present. b. Applying cold packs before exercise may decrease joint pain. c. Exercises should be performed passively by someone other than the patient. d. Walking may substitute for range-of-motion (ROM) exercises on some days.

ANS: B Cold application is helpful in reducing pain during periods of exacerbation of RA. Because the joint pain is chronic, patients are instructed to exercise even when joints are painful. ROM exercises are intended to strengthen joints and improve flexibility, so passive ROM alone is not sufficient. Recreational exercise is encouraged but is not a replacement for ROM exercises. DIF: Cognitive Level: Apply (application) REF: 1531 TOP: Nursing Process: Implementation MSC: NCLEX: Physiological Integrity

Which information about a patient who is scheduled for an oral glucose tolerance test should be reported to the health care provider before starting the test? a. The patient reports having occasional orthostatic dizziness. b. The patient takes oral corticosteroids for rheumatoid arthritis. c. The patient has had a 10-lb weight gain in the last month. d. The patient drank several glasses of water an hour previously.

ANS: B Corticosteroids can affect blood glucose results. The other information will be provided to the health care provider but will not affect the test results. DIF: Cognitive Level: Apply (application) REF: 1109 TOP: Nursing Process: Assessment MSC: NCLEX: Physiological Integrity

Which assessment information obtained by the nurse indicates a patient with an exacerbation of rheumatoid arthritis (RA) is experiencing a side effect of prednisone? a. The patient has joint pain and stiffness. b. The patient's blood glucose is 165 mg/dL. c. The patient has experienced a recent 5-pound weight loss. d. The patient's erythrocyte sedimentation rate (ESR) has increased.

ANS: B Corticosteroids have the potential to cause diabetes mellitus. The finding of elevated blood glucose reflects this side effect of prednisone. Corticosteroids increase appetite and lead to weight gain. An elevated ESR with no improvement in symptoms would indicate the prednisone was not effective but would not be side effects of the medication. DIF: Cognitive Level: Apply (application) REF: 1530 TOP: Nursing Process: Evaluation MSC: NCLEX: Physiological Integrity

The nurse notes crackling sounds and a grating sensation with palpation of an older patient's elbow. How will this finding be documented? a. Torticollis c. Subluxation b. Crepitation d. Epicondylitis

ANS: B Crackling sounds and a grating sensation that accompany movement are described as crepitus or crepitation. Torticollis is a twisting of the neck to one side, subluxation is a partial dislocation of the joint, and epicondylitis is an inflammation of the elbow causing a dull ache that increases with movement. DIF: Cognitive Level: Understand (comprehension) REF: 1456 TOP: Nursing Process: Assessment MSC: NCLEX: Physiological Integrity

12. The nurse notes crackling sounds and a grating sensation with palpation of an older patient's elbow. How will this finding be documented? a. Torticollis b. Crepitation c. Subluxation d. Epicondylitis

ANS: B Crackling sounds and a grating sensation that accompany movement are described as crepitus or crepitation. Torticollis is a twisting of the neck to one side, subluxation is a partial dislocation of the joint, and epicondylitis is an inflammation of the elbow that causes a dull ache that increases with movement.

Which assessment finding for a patient using naproxen (Naprosyn) to treat osteoarthritis is likely to require a change in medication? a. The patient has gained 3 lb. b. The patient has dark-colored stools. c. The patient's pain affects multiple joints. d. The patient uses capsaicin cream (Zostrix).

ANS: B Dark-colored stools may indicate the patient is experiencing gastrointestinal bleeding caused by the naproxen. The patient's ongoing pain and weight gain will also be reported and may indicate a need for a different treatment and/or counseling about avoiding weight gain, but these are not as large a concern as the possibility of gastrointestinal bleeding. Use of capsaicin cream with oral medications is appropriate. DIF: Cognitive Level: Apply (application) REF: 1521 TOP: Nursing Process: Assessment MSC: NCLEX: Physiological Integrity

Which information will the nurse include when teaching a patient with newly diagnosed ankylosing spondylitis (AS) about management of the condition? a. Exercise by taking long walks. b. Do daily deep-breathing exercises. c. Sleep on the side with hips flexed. d. Take frequent naps during the day.

ANS: B Deep-breathing exercises are used to decrease the risk for pulmonary complications that may result from reduced chest expansion that can occur with AS. Patients should sleep on the back and avoid flexed positions. Prolonged standing and walking should be avoided. There is no need for frequent naps. DIF: Cognitive Level: Apply (application) REF: 1537 TOP: Nursing Process: Implementation MSC: NCLEX: Physiological Integrity

The nurse determines that demeclocycline is effective for a patient with syndrome of inappropriate antidiuretic hormone (SIADH) based on finding that the patient's a. weight has increased. c. peripheral edema is increased. b. urinary output is increased. d. urine specific gravity is increased.

ANS: B Demeclocycline blocks the action of antidiuretic hormone (ADH) on the renal tubules and increases urine output. An increase in weight or an increase in urine specific gravity indicates that the SIADH is not corrected. Peripheral edema does not occur with SIADH. A sudden weight gain without edema is a common clinical manifestation of this disorder. DIF: Cognitive Level: Apply (application) REF: 1160 TOP: Nursing Process: Evaluation MSC: NCLEX: Physiological Integrity

Several weeks after a stroke, a 50-yr-old male patient has impaired awareness of bladder fullness, resulting in urinary incontinence. Which nursing intervention should be planned to begin an effective bladder training program? a. Limit fluid intake to 1200 mL daily to reduce urine volume. b. Assist the patient onto the bedside commode every 2 hours. c. Perform intermittent catheterization after each voiding to check for residual urine. d. Use an external "condom" catheter to protect the skin and prevent embarrassment.

ANS: B Developing a regular voiding schedule will prevent incontinence and may increase patient awareness of a full bladder. A 1200-mL fluid restriction may lead to dehydration. Intermittent catheterization and use of a condom catheter are appropriate in the acute phase of stroke, but should not be considered solutions for long-term management because of the risks for urinary tract infection and skin breakdown. DIF: Cognitive Level: Apply (application) REF: 1360 TOP: Nursing Process: Planning MSC: NCLEX: Physiological Integrity

A patient has arrived for a scheduled hemodialysis session. Which nursing action is most appropriate for the registered nurse (RN) to delegate to a dialysis technician? a. Teach the patient about fluid restrictions. b. Check blood pressure before starting dialysis. c. Assess for causes of an increase in predialysis weight. d. Determine the ultrafiltration rate for the hemodialysis.

ANS: B Dialysis technicians are educated in monitoring for blood pressure. Assessment, adjustment of the appropriate ultrafiltration rate, and patient teaching require the education and scope of practice of an RN. DIF: Cognitive Level: Apply (application) REF: 1089 OBJ: Special Questions: Delegation TOP: Nursing Process: Planning MSC: NCLEX: Safe and Effective Care Environment

A nurse who works on the orthopedic unit has just received change-of-shift report. Which patient should the nurse assess first? a. Patient who reports foot pain after hammertoe surgery b. Patient who has not voided 10 hours after a laminectomy c. Patient with low back pain and a positive straight-leg-raise test d. Patient with osteomyelitis who has a temperature of 100.5° F (38.1° C)

ANS: B Difficulty in voiding may indicate damage to the spinal nerves and should be assessed and reported to the surgeon immediately. The information about the other patients is consistent with their diagnoses. The nurse will need to assess them as quickly as possible, but the information about them does not indicate a need for immediate intervention. DIF: Cognitive Level: Analyze (analysis) REF: 1507 OBJ: Special Questions: Prioritization | Special Questions: Multiple Patients TOP: Nursing Process: Planning MSC: NCLEX: Safe and Effective Care Environment

Which information in a patient's history indicates to the nurse that the patient is not an appropriate candidate for kidney transplantation? a. The patient has type 1 diabetes. b. The patient has metastatic lung cancer. c. The patient has a history of chronic hepatitis C infection. d. The patient is infected with human immunodeficiency virus.

ANS: B Disseminated malignancies are a contraindication to transplantation. The conditions of the other patients are not contraindications for kidney transplant. DIF: Cognitive Level: Understand (comprehension) REF: 1092 TOP: Nursing Process: Assessment MSC: NCLEX: Physiological Integrity

15. Which action can the nurse delegate to unlicensed assistive personnel (UAP) who are working in the orthopedic clinic? a. Grade leg muscle strength for a patient with back pain. b. Obtain blood sample for uric acid from a patient with gout. c. Perform straight-leg-raise testing for a patient with sciatica. d. Check for knee joint crepitation before arthroscopic surgery.

ANS: B Drawing blood specimens is a common skill performed by UAP in clinic settings. The other actions are assessments and require registered nurse (RN)-level judgment and critical thinking.

Which assessment data for a patient who has Guillain-Barré syndrome will require the nurse's most immediate action? a. The patient's sacral area skin is reddened. b. The patient is continuously drooling saliva. c. The patient complains of severe pain in the feet. d. The patient's blood pressure (BP) is 150/82 mm Hg.

ANS: B Drooling indicates decreased ability to swallow, which places the patient at risk for aspiration and requires rapid nursing and collaborative actions such as suctioning and possible endotracheal intubation. The foot pain should be treated with appropriate analgesics, the BP requires ongoing monitoring, and the skin integrity requires intervention, but these actions are not as urgently needed as maintenance of respiratory function. DIF: Cognitive Level: Analyze (analysis)

Which nursing action has the highest priority for a patient who was admitted 16 hours earlier with a C5 spinal cord injury? a. Cardiac monitoring for bradycardia b. Assessment of respiratory rate and effort c. Administration of low-molecular-weight heparin d. Application of pneumatic compression devices to legs

ANS: B Edema around the area of injury may lead to damage above the C4 level, so the highest priority is assessment of the patient's respiratory function. The other actions also are appropriate for preventing deterioration or complications but are not as important as assessment of respiratory effort. DIF: Cognitive Level: Analyze (analysis)

Which result for a patient with systemic lupus erythematosus (SLE) is most important for the nurse to communicate to the health care provider? a. Decreased C-reactive protein (CRP) b. Elevated blood urea nitrogen (BUN) c. Positive antinuclear antibodies (ANA) d. Positive lupus erythematosus cell prep

ANS: B Elevated BUN and serum creatinine indicate possible lupus nephritis and a need for a change in therapy to avoid further renal damage. The positive lupus erythematosus cell prep and ANA would be expected in a patient with SLE. A drop in CRP shows decreased inflammation. DIF: Cognitive Level: Analysis (analyze) REF: 1541 OBJ: Special Questions: Prioritization TOP: Nursing Process: Assessment MSC: NCLEX: Physiological Integrity

A patient seen in the emergency department for severe headache and acute confusion has a serum sodium level of 118 mEq/L. The nurse will anticipate the need for which diagnostic test? a. Urinary 17-ketosteroids b. Antidiuretic hormone level c. Growth hormone stimulation test d. Adrenocorticotropic hormone level

ANS: B Elevated levels of antidiuretic hormone will cause water retention and decrease serum sodium levels. The other tests would not be helpful in determining possible causes of the patient's hyponatremia. DIF: Cognitive Level: Apply (application) REF: 1115 TOP: Nursing Process: Planning MSC: NCLEX: Physiological Integrity

The central venous oxygen saturation (ScvO2) is decreasing in a patient who has severe pancreatitis. To determine the possible cause of the decreased ScvO2, the nurse assesses the patient's a. lipase level. c. urinary output. b. temperature. d. body mass index.

ANS: B Elevated temperature increases metabolic demands and O2 use by tissues, resulting in a drop in O2 saturation of central venous blood. Information about the patient's body mass index, urinary output, and lipase will not help in determining the cause of the patient's drop in ScvO2. DIF: Cognitive Level: Apply (application)

The cardiac telemetry unit charge nurse receives status reports from other nursing units about four patients who need cardiac monitoring. Which patient should be transferred to the cardiac unit first? a. Patient with Hashimoto's thyroiditis and a heart rate of 102 b. Patient with tetany who has a new order for IV calcium chloride c. Patient with Cushing syndrome and a blood glucose of 140 mg/dL d. Patient with Addison's disease who takes hydrocortisone twice daily

ANS: B Emergency treatment of tetany requires IV administration of calcium; electrocardiographic monitoring will be required because cardiac arrest may occur if high calcium levels result from too-rapid administration. The information about the other patients indicates that they are more stable than the patient with tetany. DIF: Cognitive Level: Analyze (analysis) REF: 1168 OBJ: Special Questions: Multiple Patients | Special Questions: Prioritization TOP: Nursing Process: Planning MSC: NCLEX: Safe and Effective Care Environment

To verify the correct placement of an oral endotracheal tube (ET) after insertion, the best initial action by the nurse is to a. obtain a portable chest x-ray. b. use an end-tidal CO2 monitor. c. auscultate for bilateral breath sounds. d. observe for symmetrical chest movement.

ANS: B End-tidal CO2 monitors are currently recommended for rapid verification of ET placement. Auscultation for bilateral breath sounds and checking chest expansion are also used, but they are not as accurate as end-tidal CO2 monitoring. A chest x-ray confirms the placement but is done after the tube is secured. DIF: Cognitive Level: Analyze (analysis)

Which action will the public health nurse take to reduce the incidence of epidemic encephalitis in a community? a. Teach about prophylactic antibiotics after exposure to encephalitis. b. Encourage the use of effective insect repellent during mosquito season. c. Remind patients that most cases of viral encephalitis can be cared for at home. d. Arrange to screen school-age children for West Nile virus during the school year.

ANS: B Epidemic encephalitis is usually spread by mosquitoes and ticks. Use of insect repellent is effective in reducing risk. Encephalitis frequently requires that the patient be hospitalized in an intensive care unit during the initial stages. Antibiotic prophylaxis is not used to prevent encephalitis because most encephalitis is viral. West Nile virus is most common in adults over age 50 during the summer and early fall. DIF: Cognitive Level: Apply (application)

During the primary assessment of a victim of a motor vehicle collision, the nurse determines that the patient has an unobstructed airway. Which action should the nurse take next? a. Palpate extremities for bilateral pulses. b. Observe the patient's respiratory effort. c. Check the patient's level of consciousness. d. Examine the patient for any external bleeding.

ANS: B Even with a patent airway, patients can have other problems that compromise ventilation, so the next action is to assess the patient's breathing. The other actions are also part of the initial survey but assessment of breathing should be done immediately after assessing for airway patency. DIF: Cognitive Level: Apply (application)

While close family members are visiting, a patient has a respiratory arrest, and resuscitation is started. Which action by the nurse is best? a. Tell the family members that watching the resuscitation will be very stressful. b. Ask family members if they wish to remain in the room during the resuscitation. c. Take the family members quickly out of the patient room and remain with them. d. Assign a staff member to wait with family members just outside the patient room.

ANS: B Evidence indicates that many family members want the option of remaining in the room during procedures such as cardiopulmonary resuscitation (CPR) and that this decreases anxiety and facilitates grieving. The other options may be appropriate if the family decides not to remain with the patient. DIF: Cognitive Level: Analyze (analysis)

A patient whose employment requires frequent lifting has a history of chronic back pain. After the nurse has taught the patient about correct body mechanics, which patient statement indicates the teaching has been effective? a. "I will keep my back straight when I lift above than my waist." b. "I will begin doing exercises to strengthen and support my back." c. "I will tell my boss I need a job where I can stay seated at a desk." d. "I can sleep with my hips and knees extended to prevent back strain."

ANS: B Exercises can help strengthen the muscles that support the back. Flexion of the hips and knees places less strain on the back than keeping these joints extended. Sitting for prolonged periods can aggravate back pain. Modifications in the way the patient lifts boxes are needed, but the patient should not lift above the level of the elbows. DIF: Cognitive Level: Apply (application) REF: 1504 TOP: Nursing Process: Evaluation MSC: NCLEX: Health Promotion and Maintenance

A patient seen in the clinic for a bladder infection describes the following symptoms. Which information is most important for the nurse to report to the health care provider? a. Urinary urgency b. Left-sided flank pain c. Intermittent hematuria d. Burning with urination

ANS: B Flank pain indicates that the patient may have developed pyelonephritis as a complication of the bladder infection. The other clinical manifestations are consistent with a lower urinary tract infection.

The nurse is titrating the IV fluid infusion rate immediately after a patient has had kidney transplantation. Which parameter will be most important for the nurse to consider? a. Heart rate b. Urine output c. Creatinine clearance d. Blood urea nitrogen (BUN) level

ANS: B Fluid volume is replaced based on urine output after transplant because the urine output can be as high as a liter an hour. The other data will be monitored but are not the most important determinants of fluid infusion rate. DIF: Cognitive Level: Analyze (analysis) REF: 1095 OBJ: Special Questions: Prioritization TOP: Nursing Process: Implementation MSC: NCLEX: Physiological Integrity

The intensive care unit (ICU) nurse educator determines that teaching a new staff nurse about arterial pressure monitoring has been effective when the nurse a. balances and calibrates the monitoring equipment every 2 hours. b. positions the zero-reference stopcock line level with the phlebostatic axis. c. ensures that the patient is supine with the head of the bed flat for all readings. d. rechecks the location of the phlebostatic axis with changes in the patient's position.

ANS: B For accurate measurement of pressures, the zero-reference level should be at the phlebostatic axis. There is no need to rebalance and recalibrate monitoring equipment every 2 hours. Accurate hemodynamic readings are possible with the patient's head raised to 45 degrees or in the prone position. The anatomic position of the phlebostatic axis does not change when patients are repositioned. DIF: Cognitive Level: Apply (application)

A patient who has experienced blunt abdominal trauma during a motor vehicle collision is complaining of increasing abdominal pain. The nurse will plan to teach the patient about the purpose of a. peritoneal lavage. b. abdominal ultrasonography. c. nasogastric (NG) tube placement. d. magnetic resonance imaging (MRI).

ANS: B For patients who are at risk for intraabdominal bleeding, focused abdominal ultrasonography is the preferred method to assess for intraperitoneal bleeding. An MRI would not be used. Peritoneal lavage is an alternative, but it is more invasive. An NG tube would not be helpful in the diagnosis of intraabdominal bleeding. DIF: Cognitive Level: Apply (application)

An unresponsive patient with type 2 diabetes is brought to the emergency department and diagnosed with hyperosmolar hyperglycemic syndrome (HHS). The nurse will anticipate the need to a. give 50% dextrose. b. insert an IV catheter. c. initiate O2 by nasal cannula. d. administer glargine (Lantus) insulin.

ANS: B HHS is initially treated with large volumes of IV fluids to correct hypovolemia. Regular insulin is administered, not a long-acting insulin. There is no indication that the patient requires O2. Dextrose solutions will increase the patient's blood glucose and would be contraindicated. DIF: Cognitive Level: Apply (application) REF: 1145 TOP: Nursing Process: Planning MSC: NCLEX: Physiological Integrity

When planning teaching for a patient with benign nephrosclerosis, the nurse should include instructions regarding a. preventing bleeding with anticoagulants. b. monitoring and recording blood pressure. c. obtaining and documenting daily weights. d. measuring daily intake and output volumes.

ANS: B Hypertension is the major manifestation of nephrosclerosis. Measurements of intake and output and daily weights are not necessary unless the patient develops renal insufficiency. Anticoagulants are not used to treat nephrosclerosis.

A construction worker arrives at an urgent care center with a deep puncture wound from a rusty nail. The patient reports having had a tetanus booster 6 years ago. The nurse will anticipate a. IV infusion of tetanus immune globulin (TIG). b. administration of the tetanus-diphtheria (Td) booster. c. intradermal injection of an immune globulin test dose. d. initiation of the tetanus-diphtheria immunization series.

ANS: B If the patient has not been immunized in the past 5 years, administration of the Td booster is indicated because the wound is deep. Immune globulin administration is given by the IM route if the patient has no previous immunization. Administration of a series of immunization is not indicated. TIG is not indicated for this patient, and a test dose is not needed for immune globulin. DIF: Cognitive Level: Apply (application)

A patient with acute osteomyelitis of the left femur is hospitalized for regional antibiotic irrigation. Which intervention will the nurse include in the initial plan of care? a. Quadriceps-setting exercises b. Immobilization of the left leg c. Positioning the left leg in flexion d. Assisted weight-bearing ambulation

ANS: B Immobilization of the affected leg helps to decrease pain and reduce the risk for pathologic fracture. Weight-bearing exercise increases the risk for pathologic fractures. Flexion of the affected limb is avoided to prevent contractures. DIF: Cognitive Level: Apply (application) REF: 1499 TOP: Nursing Process: Planning MSC: NCLEX: Physiological Integrity

Gastric lavage and administration of activated charcoal are ordered for an unconscious patient who has been admitted to the emergency department (ED) after ingesting 30 lorazepam (Ativan) tablets. Which prescribed action should the nurse plan to do first? a. Insert a large-bore orogastric tube. b. Assist with intubation of the patient. c. Prepare a 60-mL syringe with saline. d. Give first dose of activated charcoal.

ANS: B In an unresponsive patient, intubation is done before gastric lavage and activated charcoal administration to prevent aspiration. The other actions will be implemented after intubation. DIF: Cognitive Level: Analyze (analysis)

Which action can the nurse delegate to unlicensed assistive personnel (UAP) who are working in the orthopedic clinic? a. Grade leg muscle strength for a patient with back pain. b. Obtain blood sample for uric acid from a patient with gout. c. Perform straight-leg-raise testing for a patient with sciatica. d. Check for knee joint crepitation before arthroscopic surgery.

ANS: B In clinic setting, drawing blood specimens is a common skill performed by UAP who are trained. The other actions are assessments and require registered nurse (RN)-level judgment and critical thinking. DIF: Cognitive Level: Apply (application) REF: 1459 OBJ: Special Questions: Delegation | Special Questions: Multiple Patients TOP: Nursing Process: Planning MSC: NCLEX: Safe and Effective Care Environment

Which nursing assessment of a 70-yr-old patient is most important to make during initiation of thyroid replacement with levothyroxine (Synthroid)? a. Fluid balance c. Nutritional intake b. Apical pulse rate d. Orientation and alertness

ANS: B In older patients, initiation of levothyroxine therapy can increase myocardial oxygen demand and cause angina or dysrhythmias. The medication also is expected to improve mental status and fluid balance and will increase metabolic rate and nutritional needs, but these changes will not result in potentially life-threatening complications. DIF: Cognitive Level: Analyze (analysis) REF: 1169 TOP: Nursing Process: Assessment MSC: NCLEX: Physiological Integrity

Which action will the nurse take when caring for a patient who develops tetanus from injectable substance use? a. Avoid use of sedatives. b. Provide a quiet environment. c. Provide range-of-motion exercises daily. d. Check pupil reaction to light every 4 hours.

ANS: B In patients with tetanus, painful seizures can be precipitated by jarring, loud noises, or bright lights, so the nurse will minimize noise and avoid shining light into the patient's eyes. Range-of-motion exercises may also stimulate the patient and cause seizures. Although the patient has a history of injectable drug use, sedative medications will be needed to decrease spasms. DIF: Cognitive Level: Apply (application)

While caring for a patient who has been admitted with a pulmonary embolism, the nurse notes a change in the patient's oxygen saturation (SpO2) from 94% to 88%. Which action should the nurse take? a. Suction the patient's oropharynx. b. Increase the prescribed O2 flow rate. c. Instruct the patient to cough and deep breathe. d. Help the patient to sit in a more upright position.

ANS: B Increasing O2 flow rate will usually improve O2 saturation in patients with ventilation-perfusion mismatch, as occurs with pulmonary embolism. Because the problem is with perfusion, actions that improve ventilation, such as deep breathing and coughing, sitting upright, and suctioning, are not likely to improve oxygenation. DIF: Cognitive Level: Apply (application)

After evacuation of an epidural hematoma, a patient's intracranial pressure (ICP) is being monitored with an intraventricular catheter. Which information obtained by the nurse requires urgent communication with the health care provider? a. Pulse of 102 beats/min b. Temperature of 101.6° F c. Intracranial pressure of 15 mm Hg d. Mean arterial pressure of 90 mm Hg

ANS: B Infection is a serious consideration with ICP monitoring, especially with intraventricular catheters. The temperature indicates the need for antibiotics or removal of the monitor. The ICP, arterial pressure, and apical pulse only require ongoing monitoring at this time. DIF: Cognitive Level: Analyze (analysis)

Propranolol (Inderal), a β-adrenergic blocker that inhibits sympathetic nervous system activity, is prescribed for a patient who has extreme anxiety about public speaking. The nurse monitors the patient for a. dry mouth. b. bradycardia. c. constipation. d. urinary retention.

ANS: B Inhibition of the fight-or-flight response leads to a decreased heart rate. Dry mouth, constipation, and urinary retention are associated with peripheral nervous system blockade. DIF: Cognitive Level: Understand (comprehension) REF: 1299 TOP: Nursing Process: Evaluation MSC: NCLEX: Physiological Integrity

Which finding will the nurse expect when assessing a patient who has osteoarthritis (OA) of the knee? a. Presence of Heberden's nodules b. Discomfort with joint movement c. Redness and swelling of the knee joint d. Stiffness that increases with movement

ANS: B Initial symptoms of OA include pain with joint movement. Heberden's nodules occur on the fingers. Redness of the joint is associated with inflammatory arthritis such as rheumatoid arthritis. Stiffness in OA is worse right after the patient rests and decreases with joint movement. DIF: Cognitive Level: Understand (comprehension) REF: 1518 TOP: Nursing Process: Assessment MSC: NCLEX: Physiological Integrity

After receiving change-of-shift report about the following four patients, which patient should the nurse assess first? a. A 31-yr-old female patient with Cushing syndrome and a blood glucose level of 244 mg/dL b. A 70-yr-old female patient taking levothyroxine (Synthroid) who has an irregular pulse of 134 c. A 53-yr-old male patient who has Addison's disease and is due for a prescribed dose of hydrocortisone (Solu-Cortef). d. A 22-yr-old male patient admitted with syndrome of inappropriate antidiuretic hormone (SIADH) who has a serum sodium level of 130 mEq/L

ANS: B Initiation of thyroid replacement in older adults may cause angina and cardiac dysrhythmias. The patient's high pulse rate needs rapid investigation by the nurse to assess for and intervene with any cardiac problems. The other patients also require nursing assessment and/or actions but are not at risk for life-threatening complications. DIF: Cognitive Level: Analyze (analysis) REF: 1169 OBJ: Special Questions: Prioritization | Special Questions: Multiple Patients TOP: Nursing Process: Assessment MSC: NCLEX: Safe and Effective Care Environment

A patient has scleroderma manifested by CREST (calcinosis, Raynaud's phenomenon, esophageal dysfunction, sclerodactyly, and telangiectasia) syndrome. Which action will the nurse include in the plan of care? a. Avoid use of capsaicin cream on hands. b. Keep the environment warm and draft free. c. Obtain capillary blood glucose before meals. d. Assist to bathroom every 2 hours while awake.

ANS: B Keeping the room warm will decrease the incidence of Raynaud's phenomenon, one aspect of the CREST syndrome. Capsaicin cream may be used to improve circulation and decrease pain. There is no need to obtain blood glucose or to assist the patient to the bathroom every 2 hours. DIF: Cognitive Level: Apply (application) REF: 1544 TOP: Nursing Process: Assessment MSC: NCLEX: Physiological Integrity

The home health registered nurse (RN) is planning care for a patient with a seizure disorder related to a recent head injury. Which nursing action can be delegated to a licensed practical/vocational nurse (LPN/LVN)? a. Make referrals to appropriate community agencies. b. Place medications in the home medication organizer. c. Teach the patient and family how to manage seizures. d. Assess for use of medications that may precipitate seizures.

ANS: B LPN/LVN education includes administration of medications. The other activities require RN education and scope of practice. DIF: Cognitive Level: Apply (application) REF: 1381 OBJ: Special Questions: Delegation TOP: Nursing Process: Planning MSC: NCLEX: Safe and Effective Care Environment

The health care provider has prescribed the following interventions for a patient who is taking azathioprine (Imuran) for systemic lupus erythematosus. Which order will the nurse question? a. Draw anti-DNA blood titer. c. Naproxen (Aleve) 200 mg BID. b. Administer varicella vaccine. d. Famotidine (Pepcid) 20 mg daily.

ANS: B Live virus vaccines, such as varicella, are contraindicated in a patient taking immunosuppressive drugs. The other orders are appropriate for the patient. DIF: Cognitive Level: Apply (application) REF: 1540 TOP: Nursing Process: Implementation MSC: NCLEX: Physiological Integrity

A patient with gout has a new prescription for losartan (Cozaar) to control the condition. The nurse will plan to monitor a. blood glucose. c. erythrocyte count. b. blood pressure. d. lymphocyte count.

ANS: B Losartan, an angiotensin II receptor antagonist, will lower blood pressure. It does not affect blood glucose, red blood cells, or lymphocytes. DIF: Cognitive Level: Apply (application) REF: 1534 TOP: Nursing Process: Planning MSC: NCLEX: Physiological Integrity

Which question will the nurse in the endocrine clinic ask to help determine a patient's risk factors for goiter? a. "How much milk do you drink?" b. "What medications are you taking?" c. "Are your immunizations up to date?" d. "Have you had any recent neck injuries?"

ANS: B Medications that contain thyroid-inhibiting substances can cause goiter. Milk intake, neck injury, and immunization history are not risk factors for goiter. DIF: Cognitive Level: Understand (comprehension) REF: 1162 TOP: Nursing Process: Assessment MSC: NCLEX: Physiological Integrity

A 29-yr-old woman is taking methotrexate to treat rheumatoid arthritis. Which information from the patient's health history is important for the nurse to report to the health care provider related to the methotrexate? a. The patient had a history of infectious mononucleosis as a teenager. b. The patient is trying to get pregnant before her disease becomes more severe. c. The patient has a family history of age-related macular degeneration of the retina. d. The patient has been using large doses of vitamins and health foods to treat the RA.

ANS: B Methotrexate is teratogenic, and the patient should be taking contraceptives during methotrexate therapy. The other information will not impact the choice of methotrexate as therapy. DIF: Cognitive Level: Apply (application) REF: 1528 TOP: Nursing Process: Assessment MSC: NCLEX: Physiological Integrity

Which nursing action can the registered nurse (RN) delegate to unlicensed assistive personnel (UAP) who are assisting with the care of a patient with scleroderma? a. Monitor for difficulty in breathing. b. Document the patient's oral intake. c. Check finger strength and movement. d. Apply capsaicin (Zostrix) cream to hands.

ANS: B Monitoring and documenting patient's oral intake is included in UAP education and scope of practice. Assessments for changes in physical status and administration of medications require more education and scope of practice, and should be done by RNs. DIF: Cognitive Level: Apply (application) REF: 1529 OBJ: Special Questions: Delegation TOP: Nursing Process: Planning MSC: NCLEX: Physiological Integrity

The nurse is caring for a patient who has an intraaortic balloon pump in place. Which action should be included in the plan of care? a. Avoid the use of anticoagulant medications. b. Measure the patient's urinary output every hour. c. Provide passive range of motion for all extremities. d. Position the patient supine with head flat at all times.

ANS: B Monitoring urine output will help determine whether the patient's cardiac output has improved and also help monitor for balloon displacement blocking the renal arteries. The head of the bed can be elevated up to 30 degrees. Heparin is used to prevent thrombus formation. Limited movement is allowed for the extremity with the balloon insertion site to prevent displacement of the balloon. DIF: Cognitive Level: Apply (application)

The nurse determines a need for additional instruction when the patient with newly diagnosed type 1 diabetes says which of the following? a. "I will need a bedtime snack because I take an evening dose of NPH insulin." b. "I can choose any foods, as long as I use enough insulin to cover the calories." c. "I can have an occasional beverage with alcohol if I include it in my meal plan." d. "I will eat something at meal times to prevent hypoglycemia, even if I am not hungry."

ANS: B Most patients with type 1 diabetes need to plan diet choices very carefully. Patients who are using intensified insulin therapy have considerable flexibility in diet choices but still should restrict dietary intake of items such as fat, protein, and alcohol. The other patient statements are correct and indicate good understanding of the diet instruction. DIF: Cognitive Level: Apply (application) REF: 1132 TOP: Nursing Process: Evaluation MSC: NCLEX: Physiological Integrity

A patient with massive trauma and possible spinal cord injury is admitted to the emergency department (ED). Which assessment finding by the nurse will help confirm a diagnosis of neurogenic shock? a. Inspiratory crackles c. Cool, clammy extremities b. Heart rate 45 beats/min d. Temperature 101.2°F (38.4°C)

ANS: B Neurogenic shock is characterized by hypotension and bradycardia. The other findings would be more consistent with other types of shock. DIF: Cognitive Level: Understand (comprehension)

The nurse is admitting a patient who has a neck fracture at the C6 level to the intensive care unit. Which assessment findings indicate neurogenic shock? a. Involuntary and spastic movement b. Hypotension and warm extremities c. Hyperactive reflexes below the injury d. Lack of sensation or movement below the injury

ANS: B Neurogenic shock is characterized by hypotension, bradycardia, and vasodilation leading to warm skin temperature. Spasticity and hyperactive reflexes do not occur at this stage of spinal cord injury. Lack of movement and sensation indicate spinal cord injury but not neurogenic shock. DIF: Cognitive Level: Understand (comprehension)

After the nurse has finished teaching a patient with osteoarthritis (OA) of the right hip about how to manage the OA, which patient statement indicates a need for more teaching? a. "I can exercise every day to help maintain joint motion." b. "I will take 1 g of acetaminophen (Tylenol) every 4 hours." c. "I will take a shower in the morning to help relieve stiffness." d. "I can use a cane to decrease the pressure and pain in my hip."

ANS: B No more than 4 g of acetaminophen (1 g every 6 hours) should be taken daily to decrease the risk for liver damage. Regular exercise, moist heat, and supportive equipment are recommended for OA management. DIF: Cognitive Level: Apply (application) REF: 1523 TOP: Nursing Process: Evaluation MSC: NCLEX: Physiological Integrity

Which intervention will the nurse include in the plan of care for a patient with primary restless legs syndrome (RLS) who is having difficulty sleeping? a. Teach about the use of antihistamines to improve sleep. b. Suggest that the patient exercise regularly during the day. c. Make a referral to a massage therapist for deep massage of the legs. d. Assure the patient that the problem is transient and likely to resolve.

ANS: B Nondrug interventions such as getting regular exercise are initially suggested to improve sleep quality in patients with RLS. Antihistamines may aggravate RLS. Massage does not alleviate RLS symptoms, and RLS is likely to progress in most patients. DIF: Cognitive Level: Apply (application) REF: 1382 TOP: Nursing Process: Planning MSC: NCLEX: Physiological Integrity

A patient seen in the outpatient clinic is diagnosed with mild cognitive impairment (MCI). Which action will the nurse include in the plan of care? a. Suggest a move into an assisted living facility. b. Schedule the patient for more frequent appointments. c. Ask family members to supervise the patient's daily activities. d. Discuss the preventive use of acetylcholinesterase medications.

ANS: B Ongoing monitoring is recommended for patients with MCI. MCI does not usually interfere with activities of daily living, acetylcholinesterase drugs are not used for MCI, and an assisted living facility is not indicated for a patient with MCI. DIF: Cognitive Level: Apply (application) REF: 1405 TOP: Nursing Process: Planning MSC: NCLEX: Psychosocial Integrity

Which patient assessment will help the nurse identify potential complications of trigeminal neuralgia? a. Have the patient clench the jaws. b. Inspect the oral mucosa and teeth. c. Palpate the face to compare skin temperature bilaterally. d. Identify trigger zones by lightly touching the affected side.

ANS: B Oral hygiene is frequently neglected because of fear of triggering facial pain and may lead to gum disease, dental caries, or an abscess. Having the patient clench the facial muscles will not be useful because the sensory branches (rather than motor branches) of the nerve are affected by trigeminal neuralgia. Light touch and palpation may be triggers for pain and should be avoided. DIF: Cognitive Level: Apply (application)

To prevent recurrence of uric acid renal calculi, the nurse teaches the patient to avoid eating a. milk and cheese. b. sardines and liver. c. spinach and chocolate. d. legumes and dried fruit.

ANS: B Organ meats and fish such as sardines increase purine levels and uric acid. Spinach, chocolate, and tomatoes should be avoided in patients who have oxalate stones. Milk, dairy products, legumes, and dried fruits may increase the incidence of calcium-containing stones.

A registered nurse (RN) is caring for a patient with a goiter and possible hyperthyroidism. Which action by the RN indicates that the charge nurse needs to provide the RN with additional teaching? a. The RN checks the blood pressure in both arms. b. The RN palpates the neck to assess thyroid size. c. The RN orders saline eye drops to lubricate the patient's bulging eyes. d. The RN lowers the thermostat to decrease the temperature in the room.

ANS: B Palpation can cause the release of thyroid hormones in a patient with an enlarged thyroid and should be avoided. The other actions by the new RN are appropriate when caring for a patient with an enlarged thyroid. DIF: Cognitive Level: Apply (application) REF: 1112 OBJ: Special Questions: Delegation TOP: Nursing Process: Evaluation MSC: NCLEX: Safe and Effective Care Environment

Which additional information will the nurse need to consider when reviewing the laboratory results for a patient's total calcium level? a. The blood glucose c. The phosphate level b. The serum albumin d. The magnesium level

ANS: B Part of the total calcium is bound to albumin, so hypoalbuminemia can lead to misinterpretation of calcium levels. The other laboratory values will not affect total calcium interpretation. DIF: Cognitive Level: Apply (application) REF: 1117 TOP: Nursing Process: Assessment MSC: NCLEX: Physiological Integrity

The nurse is taking a health history from a 29-yr-old pregnant patient at the first prenatal visit. The patient reports that she has no personal history of diabetes, but her mother has diabetes. Which action will the nurse plan to take? a. Teach the patient about administering regular insulin. b. Schedule the patient for a fasting blood glucose level. c. Teach about an increased risk for fetal problems with gestational diabetes. d. Schedule an oral glucose tolerance test for the twenty-fourth week of pregnancy.

ANS: B Patients at high risk for gestational diabetes should be screened for diabetes on the initial prenatal visit. An oral glucose tolerance test may also be used to check for diabetes, but it would be done before the twenty-fourth week. Teaching plans would depend on the outcome of a fasting blood glucose test and other tests. DIF: Cognitive Level: Apply (application) REF: 1138 OBJ: Special Questions: Prioritization TOP: Nursing Process: Planning MSC: NCLEX: Physiological Integrity

When a patient with acute kidney injury (AKI) has an arterial blood pH of 7.30, the nurse will expect an assessment finding of a. persistent skin tenting b. rapid, deep respirations. c. hot, flushed face and neck. d. bounding peripheral pulses.

ANS: B Patients with metabolic acidosis caused by AKI may have Kussmaul respirations as the lungs try to regulate carbon dioxide. Bounding pulses and vasodilation are not associated with metabolic acidosis. Because the patient is likely to have fluid retention, poor skin turgor would not be a finding in AKI. DIF: Cognitive Level: Apply (application) REF: 1072 TOP: Nursing Process: Assessment MSC: NCLEX: Physiological Integrity

The nurse caring for a patient after cystoscopy plans that the patient a. learns to request narcotics for pain. b. understands to expect blood-tinged urine. c. restricts activity to bed rest for 4 to 6 hours. d. remains NPO for 8 hours to prevent vomiting.

ANS: B Pink-tinged urine and urinary frequency are expected after cystoscopy. Burning on urination is common, but pain that requires opioids for relief is not expected. A good fluid intake is encouraged after this procedure. Bed rest is not required after cystoscopy. DIF: Cognitive Level: Apply (application) REF: 1028 TOP: Nursing Process: Planning MSC: NCLEX: Physiological Integrity

A patient who has severe Alzheimer's disease (AD) is being admitted to the hospital for surgery. Which intervention will the nurse include in the plan of care? a. Encourage the patient to discuss events from the past. b. Maintain a consistent daily routine for the patient's care. c. Reorient the patient to the date and time every 2 to 3 hours. d. Provide the patient with current newspapers and magazines.

ANS: B Providing a consistent routine will decrease anxiety and confusion for the patient. Reorientation to time and place will not be helpful to the patient with severe AD, and the patient will not be able to read. The patient with severe AD will probably not be able to remember events from the past. DIF: Cognitive Level: Apply (application) REF: 1414 TOP: Nursing Process: Planning MSC: NCLEX: Psychosocial Integrity

Which action should the nurse take after a patient treated with intramuscular glucagon for hypoglycemia regains consciousness? a. Assess the patient for symptoms of hyperglycemia. b. Give the patient a snack of peanut butter and crackers. c. Have the patient drink a glass of orange juice or nonfat milk. d. Administer a continuous infusion of 5% dextrose for 24 hours.

ANS: B Rebound hypoglycemia can occur after glucagon administration, but having a meal containing complex carbohydrates plus protein and fat will help prevent hypoglycemia. Orange juice and nonfat milk will elevate blood glucose rapidly, but the cheese and crackers will stabilize blood glucose. Administration of IV glucose might be used in patients who were unable to take in nutrition orally. The patient should be assessed for symptoms of hypoglycemia after glucagon administration. DIF: Cognitive Level: Apply (application) REF: 1129 TOP: Nursing Process: Implementation MSC: NCLEX: Physiological Integrity

The nurse is caring for a patient who has an arterial catheter in the left radial artery for arterial pressure-based cardiac output (APCO) monitoring. Which information obtained by the nurse requires a report to the health care provider? a. The patient has a positive Allen test result. b. There is redness at the catheter insertion site. c. The mean arterial pressure (MAP) is 86 mm Hg. d. The dicrotic notch is visible in the arterial waveform.

ANS: B Redness at the catheter insertion site indicates possible infection. The Allen test is performed before arterial line insertion, and a positive test result indicates normal ulnar artery perfusion. A MAP of 86 mm Hg is normal, and the dicrotic notch is normally present on the arterial waveform. DIF: Cognitive Level: Apply (application)

The nurse is admitting a patient with a basal skull fracture. The nurse notes ecchymoses around both eyes and clear drainage from the patient's nose. Which admission order should the nurse question? a. Keep the head of bed elevated. b. Insert nasogastric tube to low suction. c. Turn patient side to side every 2 hours. d. Apply cold packs intermittently to face.

ANS: B Rhinorrhea may indicate a dural tear with cerebrospinal fluid leakage. Insertion of a nasogastric tube will increase the risk for infections such as meningitis. Turning the patient, elevating the head, and applying cold packs are appropriate orders. DIF: Cognitive Level: Apply (application)

A patient with Parkinson's disease has bradykinesia. Which action will the nurse include in the plan of care? a. Instruct the patient in activities that can be done while lying or sitting. b. Suggest that the patient rock from side to side to initiate leg movement. c. Have the patient take small steps in a straight line directly in front of the feet. d. Teach the patient to keep the feet in contact with the floor and slide them forward.

ANS: B Rocking the body from side to side stimulates balance and improves mobility. The patient will be encouraged to continue exercising because this will maintain functional abilities. Maintaining a wide base of support will help with balance. The patient should lift the feet and avoid a shuffling gait. DIF: Cognitive Level: Apply (application) REF: 1392 TOP: Nursing Process: Planning MSC: NCLEX: Physiological Integrity

Which hemodynamic parameter best reflects the effectiveness of drugs that the nurse gives to reduce a patient's left ventricular afterload? a. Mean arterial pressure (MAP) b. Systemic vascular resistance (SVR) c. Pulmonary vascular resistance (PVR) d. Pulmonary artery wedge pressure (PAWP)

ANS: B SVR reflects the resistance to ventricular ejection, or afterload. The other parameters may be monitored but do not reflect afterload as directly. DIF: Cognitive Level: Apply (application)

Which statement by a patient with systemic lupus erythematosus (SLE) indicates the patient has understood the nurse's teaching about the condition? a. "I will exercise even if I am tired." b. "I will use sunscreen when I am outside." c. "I should avoid nonsteroidal antiinflammatory drugs." d. "I should take birth control pills to avoid getting pregnant."

ANS: B Severe skin reactions can occur in patients with SLE who are exposed to the sun. Patients should avoid fatigue by balancing exercise with rest periods as needed. Oral contraceptives can exacerbate lupus. Aspirin and nonsteroidal antiinflammatory drugs are used to treat the musculoskeletal manifestations of SLE. DIF: Cognitive Level: Apply (application) REF: 1542 TOP: Nursing Process: Evaluation MSC: NCLEX: Physiological Integrity

Family members of a patient who has a traumatic brain injury ask the nurse about the purpose of the ventriculostomy system being used for intracranial pressure monitoring. Which response by the nurse is best for this situation? a. "This type of monitoring system is complex and it is managed by skilled staff." b. "The monitoring system helps show whether blood flow to the brain is adequate." c. "The ventriculostomy monitoring system helps check for alterations in cerebral perfusion pressure." d. "This monitoring system has multiple benefits including facilitation of cerebrospinal fluid drainage."

ANS: B Short and simple explanations should be given initially to patients and family members. The other explanations are either too complicated to be easily understood or may increase the family members' anxiety. DIF: Cognitive Level: Analyze (analysis)

The nurse is caring for a patient who has just returned after having left carotid artery angioplasty and stenting. Which assessment information is of most concern to the nurse? a. The pulse rate is 102 beats/min. b. The patient has difficulty speaking. c. The blood pressure is 144/86 mm Hg. d. There are fine crackles at the lung bases.

ANS: B Small emboli can occur during carotid artery angioplasty and stenting, and the aphasia indicates a possible stroke during the procedure. Slightly elevated pulse rate and blood pressure are not unusual because of anxiety associated with the procedure. Fine crackles at the lung bases may indicate atelectasis caused by immobility during the procedure. The nurse should have the patient take some deep breaths. DIF: Cognitive Level: Analyze (analysis) REF: 1351 OBJ: Special Questions: Prioritization TOP: Nursing Process: Assessment MSC: NCLEX: Physiological Integrity

Which information will the nurse include when teaching a 50-yr-old male patient about somatropin (Genotropin)? a. The medication will be needed for 3 to 6 months. b. Inject the medication subcutaneously every day. c. Blood glucose levels may decrease when taking the medication. d. Stop taking the medication if swelling of the hands or feet occurs.

ANS: B Somatropin is injected subcutaneously on a daily basis, preferably in the evening. The patient will need to continue on somatropin for life. If swelling or other common adverse effects occur, the health care provider should be notified. Growth hormone will increase blood glucose levels. DIF: Cognitive Level: Apply (application) REF: 1158 TOP: Nursing Process: Implementation MSC: NCLEX: Physiological Integrity

The nurse suggests that a patient recently diagnosed with rheumatoid arthritis (RA) plan to start each day with a. a brief routine of isometric exercises. b. a warm bath followed by a short rest. c. active range-of-motion (ROM) exercises. d. stretching exercises to relieve joint stiffness.

ANS: B Taking a warm shower or bath is recommended to relieve joint stiffness, which is worse in the morning. Isometric exercises would place stress on joints and would not be recommended. Stretching and ROM should be done later in the day, when joint stiffness is decreased. DIF: Cognitive Level: Apply (application) REF: 1531 TOP: Nursing Process: Implementation MSC: NCLEX: Physiological Integrity

Which action will the nurse plan to take for a patient with multiple sclerosis who has urinary retention caused by a flaccid bladder? a. Encourage a decreased evening intake of fluid. b. Teach the patient how to use the Credé method. c. Suggest the use of adult incontinence briefs for nighttime only. d. Assist the patient to the commode every 2 hours during the day.

ANS: B The Credé method can be used to improve bladder emptying. Decreasing fluid intake will not improve bladder emptying and may increase risk for urinary tract infection and dehydration. The use of incontinence briefs and frequent toileting will not improve bladder emptying. DIF: Cognitive Level: Apply (application) REF: 1387 TOP: Nursing Process: Planning MSC: NCLEX: Physiological Integrity

A patient who is unconscious after a fall from a ladder is transported to the emergency department by emergency medical personnel. During the primary survey of the patient, the nurse should a. obtain a complete set of vital signs. b. obtain a Glasgow Coma Scale score. c. attach an electrocardiogram monitor. d. ask about chronic medical conditions.

ANS: B The Glasgow Coma Scale is included when assessing for disability during the primary survey. The other information is part of the secondary survey. DIF: Cognitive Level: Apply (application)

The public health nurse is planning a program to decrease the incidence of meningitis in teenagers and young adults. Which action is most likely to be effective? a. Emphasize the importance of hand washing. b. Immunize adolescents and college freshman. c. Support serving healthy nutritional options in the college cafeteria. d. Encourage adolescents and young adults to avoid crowds in the winter.

ANS: B The Neisseria meningitides vaccination is recommended for children ages 11 and 12 years, unvaccinated teens entering high school, and college freshmen. Hand washing may help decrease the spread of bacteria, and good nutrition may increase resistance to infection. but those are not as effective as immunization. Because adolescents and young adults are in school or the workplace, avoiding crowds is not realistic. DIF: Cognitive Level: Analyze (analysis)

A patient with cardiogenic shock has the following vital signs: BP 102/50, pulse 128, respirations 28. The pulmonary artery wedge pressure (PAWP) is increased, and cardiac output is low. The nurse will anticipate an order for which medication? a. 5% albumin infusion c. epinephrine (Adrenalin) drip b. furosemide (Lasix) IV d. hydrocortisone (Solu-Cortef)

ANS: B The PAWP indicates that the patient's preload is elevated, and furosemide is indicated to reduce the preload and improve cardiac output. Epinephrine would further increase the heart rate and myocardial oxygen demand. 5% albumin would also increase the PAWP. Hydrocortisone might be considered for septic or anaphylactic shock. DIF: Cognitive Level: Apply (application)

The nurse will plan to monitor a patient diagnosed with a pheochromocytoma for a. flushing. c. bradycardia. b. headache. d. hypoglycemia.

ANS: B The classic clinical manifestations of pheochromocytoma are hypertension, tachycardia, severe headache, diaphoresis, and abdominal or chest pain. Elevated blood glucose may also occur because of sympathetic nervous system stimulation. Bradycardia and flushing would not be expected. DIF: Cognitive Level: Apply (application) REF: 1181 TOP: Nursing Process: Planning MSC: NCLEX: Physiological Integrity

When a 74-yr-old patient is seen in the health clinic with new development of a stooped posture, shuffling gait, and pill rolling-type tremor, the nurse will anticipate teaching the patient about a. oral corticosteroids. b. antiparkinsonian drugs. c. magnetic resonance imaging (MRI). d. electroencephalogram (EEG) testing.

ANS: B The clinical diagnosis of Parkinson's is made when tremor, rigidity, and akinesia, and postural instability are present. The confirmation of the diagnosis is made on the basis of improvement when antiparkinsonian drugs are administered. MRI and EEG are not useful in diagnosing Parkinson's disease, and corticosteroid therapy is not used to treat it. DIF: Cognitive Level: Apply (application) REF: 1389 TOP: Nursing Process: Planning MSC: NCLEX: Physiological Integrity

The nurse educator is evaluating the performance of a new registered nurse (RN) who is providing care to a patient who is receiving mechanical ventilation with 15 cm H2O of peak end-expiratory pressure (PEEP). Which action indicates that the new RN is safe? a. The RN plans to suction the patient every 1 to 2 hours. b. The RN uses a closed-suction technique to suction the patient. c. The RN tapes the connection between the ventilator tubing and the ET. d. The RN changes the ventilator circuit tubing routinely every 48 hours.

ANS: B The closed-suction technique is used when patients require high levels of PEEP (>10 cm H2O) to prevent the loss of PEEP that occurs when disconnecting the patient from the ventilator. Suctioning should not be scheduled routinely, but it should be done only when patient assessment data indicate the need for suctioning. Taping connections between the ET and ventilator tubing would restrict the ability of the tubing to swivel in response to patient repositioning. Ventilator tubing changes increase the risk for ventilator-associated pneumonia and are not indicated routinely. DIF: Cognitive Level: Apply (application)

The nurse is assessing a patient 4 hours after a kidney transplant. Which information is most important to communicate to the health care provider? a. The urine output is 900 to 1100 mL/hr. b. The patient's central venous pressure (CVP) is decreased. c. The patient has a level 7 (0- to 10-point scale) incisional pain. d. The blood urea nitrogen (BUN) and creatinine levels are elevated.

ANS: B The decrease in CVP suggests hypovolemia, which must be rapidly corrected to prevent renal hypoperfusion and acute tubular necrosis. The other information is not unusual in a patient after a transplant. DIF: Cognitive Level: Analyze (analysis) REF: 1095 OBJ: Special Questions: Prioritization TOP: Nursing Process: Assessment MSC: NCLEX: Physiological Integrity

A patient is being evaluated for Alzheimer's disease (AD). The nurse explains to the patient's adult children that a. the most important risk factor for AD is a family history of the disorder. b. a diagnosis of AD is made only after other causes of dementia are ruled out. c. new drugs have been shown to reverse AD deterioration dramatically in some patients. d. brain atrophy detected by magnetic resonance imaging (MRI) would confirm the diagnosis of AD.

ANS: B The diagnosis of AD is usually one of exclusion. Age is the most important risk factor for development of AD. Drugs may slow the deterioration but do not reverse the effects of AD. Brain atrophy is a common finding in AD, but it can occur in other diseases as well and does not confirm a diagnosis of AD. DIF: Cognitive Level: Understand (comprehension) REF: 1405 TOP: Nursing Process: Implementation MSC: NCLEX: Psychosocial Integrity

A patient who has been involved in a motor vehicle crash arrives in the emergency department (ED) with cool, clammy skin; tachycardia; and hypotension. Which intervention ordered by the health care provider should the nurse implement first? a. Insert two large-bore IV catheters. b. Provide O2 at 100% per non-rebreather mask. c. Draw blood to type and crossmatch for transfusions. d. Initiate continuous electrocardiogram (ECG) monitoring.

ANS: B The first priority in the initial management of shock is maintenance of the airway and ventilation. ECG monitoring, insertion of IV catheters, and obtaining blood for transfusions should also be rapidly accomplished but only after actions to maximize O2 delivery have been implemented. DIF: Cognitive Level: Analyze (analysis)

The nurse admitting a patient who has a right frontal lobe tumor would expect the patient may have a. expressive aphasia. b. impaired judgment. c. right-sided weakness d. difficulty swallowing.

ANS: B The frontal lobe controls intellectual activities such as judgment. Speech is controlled in the parietal lobe. Weakness and hemiplegia occur on the contralateral side from the tumor. Swallowing is controlled by the brainstem. DIF: Cognitive Level: Apply (application)

Which action will the nurse include in the plan of care for a patient with impaired functioning of the left glossopharyngeal nerve (CN IX) and vagus nerve (CN X)? a. Assist to stand and ambulate. b. Withhold oral fluids and food. c. Insert an oropharyngeal airway. d. Apply artificial tears every hour.

ANS: B The glossopharyngeal and vagus nerves innervate the pharynx and control the gag reflex. A patient with impaired function of these nerves is at risk for aspiration. An oral airway may be needed when a patient is unconscious and unable to maintain the airway, but it will not decrease aspiration risk. Taste and eye blink are controlled by the facial nerve. Balance and coordination are cerebellar functions. DIF: Cognitive Level: Apply (application) REF: 1305 TOP: Nursing Process: Planning MSC: NCLEX: Physiological Integrity

A 62-yr-old female patient has been hospitalized for 4 days with acute kidney injury (AKI) caused by dehydration. Which information will be most important for the nurse to report to the health care provider? a. The creatinine level is 3.0 mg/dL. b. Urine output over an 8-hour period is 2500 mL. c. The blood urea nitrogen (BUN) level is 67 mg/dL. d. The glomerular filtration rate is less than 30 mL/min/1.73 m2

ANS: B The high urine output indicates a need to increase fluid intake to prevent hypovolemia. The other information is typical of AKI and will not require a change in therapy. DIF: Cognitive Level: Analyze (analysis) REF: 1072 OBJ: Special Questions: Prioritization TOP: Nursing Process: Assessment MSC: NCLEX: Physiological Integrity

A 25-yr-old male patient has been admitted with a severe crushing injury after an industrial accident. Which laboratory result will be most important to report to the health care provider? a. Serum creatinine level of 2.1 mg/dL b. Serum potassium level of 6.5 mEq/L c. White blood cell count of 11,500/μL d. Blood urea nitrogen (BUN) of 56 mg/dL

ANS: B The hyperkalemia associated with crushing injuries may cause cardiac arrest and should be treated immediately. The nurse also will report the other laboratory values, but abnormalities in these are not immediately life threatening. DIF: Cognitive Level: Analyze (analysis) REF: 1072 OBJ: Special Questions: Prioritization TOP: Nursing Process: Assessment MSC: NCLEX: Physiological Integrity

A patient with two school-age children has recently been diagnosed with rheumatoid arthritis (RA) and tells the nurse that home life is very stressful. Which initial response by the nurse is most appropriate? a. "You need to see a family therapist for some help with stress." b. "Tell me more about the situations that are causing you stress." c. "Your family should understand the impact of your rheumatoid arthritis." d. "Perhaps it would be helpful for your family to be involved in a support group."

ANS: B The initial action by the nurse should be further assessment. The other three responses might be appropriate based on the information the nurse obtains with further assessment. DIF: Cognitive Level: Analyze (analysis) REF: 1532 TOP: Nursing Process: Implementation MSC: NCLEX: Psychosocial Integrity

A 70-yr-old female patient with left-sided hemiparesis arrives by ambulance to the emergency department. Which action should the nurse take first? a. Take the patient's blood pressure. b. Check the respiratory rate and effort. c. Assess the Glasgow Coma Scale score. d. Send the patient for a computed tomography (CT) scan.

ANS: B The initial nursing action should be to assess the airway and take any needed actions to ensure a patent airway. The other activities should take place quickly after the ABCs (airway, breathing, and circulation) are completed. DIF: Cognitive Level: Analyze (analysis) REF: 1354 OBJ: Special Questions: Prioritization TOP: Nursing Process: Implementation MSC: NCLEX: Physiological Integrity

Which assessment finding for a patient who has just been admitted with acute pyelonephritis is most important for the nurse to report to the health care provider? a. Complaint of flank pain b. Blood pressure 90/48 mm Hg c. Cloudy and foul-smelling urine d. Temperature 100.1° F (57.8° C)

ANS: B The low blood pressure indicates that urosepsis and septic shock may be occurring and should be immediately reported. The other findings are typical of pyelonephritis.

The nurse assessing a patient with newly diagnosed trigeminal neuralgia will ask the patient about a. visual problems caused by ptosis. b. triggers leading to facial discomfort. c. poor appetite caused by loss of taste. d. weakness on the affected side of the face.

ANS: B The major clinical manifestation of trigeminal neuralgia is severe facial pain triggered by cutaneous stimulation of the nerve. Ptosis, loss of taste, and facial weakness are not characteristics of trigeminal neuralgia. DIF: Cognitive Level: Apply (application)

A 37-yr-old patient has just arrived in the postanesthesia recovery unit (PACU) after a thyroidectomy. Which information about the patient is most important to communicate to the surgeon? a. Difficult to awaken. c. Reports 7/10 incisional pain. b. Increasing neck swelling. d. Cardiac rate 112 beats/minute.

ANS: B The neck swelling may lead to respiratory difficulty, and rapid intervention is needed to prevent airway obstruction. The incisional pain should be treated but is not unusual after surgery. A heart rate of 112 beats/min is not unusual in a patient who has been hyperthyroid and has just arrived in the PACU from surgery. Sleepiness in the immediate postoperative period is expected. DIF: Cognitive Level: Analyze (analysis) REF: 1168 OBJ: Special Questions: Prioritization TOP: Nursing Process: Assessment MSC: NCLEX: Physiological Integrity

The nurse admits a patient to the hospital in Addisonian crisis. Which patient statement supports the need to plan additional teaching? a. "I frequently eat at restaurants, and my food has a lot of added salt." b. "I had the flu earlier this week, so I couldn't take the hydrocortisone." c. "I always double my dose of hydrocortisone on the days that I go for a long run." d. "I take twice as much hydrocortisone in the morning dose as I do in the afternoon."

ANS: B The need for hydrocortisone replacement is increased with stressors such as illness, and the patient needs to be taught to call the health care provider because medication and IV fluids and electrolytes may need to be given. The other patient statements indicate appropriate management of the Addison's disease. DIF: Cognitive Level: Apply (application) REF: 1179 TOP: Nursing Process: Planning MSC: NCLEX: Physiological Integrity 25. A 29-yr-old woman with systemic lupus erythematosus has been prescribed 2 weeks

A high school teacher who has been diagnosed with epilepsy after having a generalized tonic-clonic seizure tells the nurse, "I cannot teach any more. It will be too upsetting if I have a seizure at work." Which response by the nurse specifically addresses the patient's concern? a. "You might benefit from some psychologic counseling." b. "Epilepsy usually can be well controlled with medications." c. "You will want to contact the Epilepsy Foundation for assistance." d. "The Department of Vocational Rehabilitation can help with work retraining."

ANS: B The nurse should inform the patient that most patients with seizure disorders are controlled with medication. The other information may be necessary if the seizures persist after treatment with antiseizure medications is implemented. DIF: Cognitive Level: Apply (application) REF: 1381 TOP: Nursing Process: Implementation MSC: NCLEX: Psychosocial Integrity

A 74-yr-old patient who is progressing to stage 5 chronic kidney disease asks the nurse, "Do you think I should go on dialysis? Which initial response by the nurse is best? a. "It depends on which type of dialysis you are considering." b. "Tell me more about what you are thinking regarding dialysis." c. "You are the only one who can make the decision about dialysis." d. "Many people your age use dialysis and have a good quality of life."

ANS: B The nurse should initially clarify the patient's concerns and questions about dialysis. The patient is the one responsible for the decision, and many people using dialysis do have good quality of life, but these responses block further assessment of the patient's concerns. Referring to which type of dialysis the patient might use only indirectly responds to the patient's question. DIF: Cognitive Level: Analyze (analysis) REF: 1091 OBJ: Special Questions: Prioritization TOP: Nursing Process: Assessment MSC: NCLEX: Psychosocial Integrity

Which action will the nurse take first when a patient is seen in the outpatient clinic with neck pain? a. Provide information about therapeutic neck exercises. b. Ask about numbness or tingling of the hands and arms. c. Suggest the patient alternate the use of heat and cold to the neck. d. Teach about the use of nonsteroidal antiinflammatory drugs (NSAIDs).

ANS: B The nurse's initial action should be further assessment of related symptoms because cervical nerve root compression will require different treatment than musculoskeletal neck pain. The other actions may also be appropriate, depending on the assessment findings. DIF: Cognitive Level: Analyze (analysis) REF: 1507 OBJ: Special Questions: Prioritization TOP: Nursing Process: Assessment MSC: NCLEX: Physiological Integrity

When assessing an older patient admitted to the emergency department (ED) with a broken arm and facial bruises, the nurse observes several additional bruises in various stages of healing. Which statement or question by the nurse should be first? a. "You should not go home." b. "Do you feel safe at home?" c. "Would you like to see a social worker?" d. "I need to report my concerns to the police."

ANS: B The nurse's initial response should be to further assess the patient's situation. Telling the patient not to return home may be an option once further assessment is done. A social worker or police report may be appropriate once further assessment is completed. DIF: Cognitive Level: Analyze (analysis)

A patient with a head injury opens his eyes to verbal stimulation, curses when stimulated, and does not respond to a verbal command to move but attempts to push away a painful stimulus. The nurse records the patient's Glasgow Coma Scale score as a. 9. b. 11. c. 13. d. 15.

ANS: B The patient has scores of 3 for eye opening, 3 for best verbal response, and 5 for best motor response. DIF: Cognitive Level: Apply (application)

A 20-yr-old patient who sustained a T2 spinal cord injury 10 days ago tells the nurse, "I want to be transferred to a hospital where the nurses know what they are doing." Which action by the nurse is appropriate? a. Respond that abusive language will not be tolerated. b. Request that the patient provide input for the plan of care. c. Perform care without responding to the patient's comments. d. Reassure the patient about the competence of the nursing staff.

ANS: B The patient is demonstrating behaviors consistent with the anger phase of the grief process, and the nurse should allow expression of anger and seek the patient's input into care. Expression of anger is appropriate at this stage, and should be accepted by the nurse. Reassurance about the competency of the staff will not be helpful in responding to the patient's concerns. Ignoring the patient's comments will increase the patient's anger and sense of helplessness. DIF: Cognitive Level: Apply (application)

The nurse identifies a need for additional teaching when the patient who is self-monitoring blood glucose a. washes the puncture site using warm water and soap. b. chooses a puncture site in the center of the finger pad. c. hangs the arm down for a minute before puncturing the site. d. says the result of 120 mg indicates good blood sugar control.

ANS: B The patient is taught to choose a puncture site at the side of the finger pad because there are fewer nerve endings along the side of the finger pad. The other patient actions indicate that teaching has been effective. DIF: Cognitive Level: Apply (application) REF: 1136 TOP: Nursing Process: Evaluation MSC: NCLEX: Health Promotion and Maintenance

Which nursing action will be included in the plan of care for a patient with Graves' disease who has exophthalmos? a. Place cold packs on the eyes to relieve pain and swelling. b. Elevate the head of the patient's bed to reduce periorbital fluid. c. Apply alternating eye patches to protect the corneas from irritation. d. Teach the patient to blink every few seconds to lubricate the corneas.

ANS: B The patient should sit upright as much as possible to promote fluid drainage from the periorbital area. With exophthalmos, the patient is unable to close the eyes completely to blink. Lubrication of the eyes, rather than eye patches, will protect the eyes from developing corneal scarring. The swelling of the eye is not caused by excessive blood flow to the eye, so cold packs will not be helpful. DIF: Cognitive Level: Apply (application) REF: 1167 TOP: Nursing Process: Planning MSC: NCLEX: Physiological Integrity

A patient is being discharged after 1 week of IV antibiotic therapy for acute osteomyelitis in the right leg. Which information will be included in the discharge teaching? a. How to apply warm packs to the leg to reduce pain b. How to monitor and care for a long-term IV catheter c. The need for daily aerobic exercise to help maintain muscle strength d. The reason for taking oral antibiotics for 7 to 10 days after discharge

ANS: B The patient will be taking IV antibiotics for several months. The patient will need to recognize signs of infection at the IV site and know how to care for the catheter during daily activities such as bathing. IV antibiotics rather than oral antibiotics are used for acute osteomyelitis. Patients are instructed to avoid exercise and heat application because these will increase swelling and the risk for spreading infection. DIF: Cognitive Level: Apply (application) REF: 1499 TOP: Nursing Process: Implementation MSC: NCLEX: Physiological Integrity

When the nurse is developing a rehabilitation plan for a 30-yr-old patient with a C6 spinal cord injury, an appropriate goal is that the patient will be able to a. drive a car with powered hand controls. b. push a manual wheelchair on a flat surface. c. turn and reposition independently when in bed. d. transfer independently to and from a wheelchair.

ANS: B The patient with a C6 injury will be able to use the hands to push a wheelchair on flat, smooth surfaces. Because flexion of the thumb and fingers is minimal, the patient will not be able to grasp a wheelchair during transfer, drive a car with powered hand controls, or turn independently in bed. DIF: Cognitive Level: Apply (application)

The nurse will explain to the patient who has a T2 spinal cord transection injury that a. use of the shoulders will be limited. b. function of both arms should be retained. c. total loss of respiratory function may occur. d. tachycardia is common with this type of injury.

ANS: B The patient with a T2 injury can expect to retain full motor and sensory function of the arms. Use of only the shoulders is associated with cervical spine injury. Loss of respiratory function occurs with cervical spine injuries. Bradycardia is associated with injuries above the T6 level. DIF: Cognitive Level: Understand (comprehension)

Which intervention will be included in the plan of care for a patient with acute kidney injury (AKI) who has a temporary vascular access catheter in the left femoral vein? a. Start continuous pulse oximetry. b. Restrict physical activity to bed rest. c. Restrict the patient's oral protein intake. d. Discontinue the urethral retention catheter.

ANS: B The patient with a femoral vein catheter must be on bed rest to prevent trauma to the vein. Protein intake is likely to be increased when the patient is receiving dialysis. The retention catheter is likely to remain in place because accurate measurement of output will be needed. There is no indication that the patient needs continuous pulse oximetry. DIF: Cognitive Level: Apply (application) REF: 1088 TOP: Nursing Process: Planning MSC: NCLEX: Physiological Integrity

A 56-yr-old female patient is admitted to the hospital with new-onset nephrotic syndrome. Which assessment data will the nurse expect? a. Poor skin turgor b. Recent weight gain c. Elevated urine ketones d. Decreased blood pressure

ANS: B The patient with a nephrotic syndrome will have weight gain associated with edema. Hypertension is a clinical manifestation of nephrotic syndrome. Skin turgor is normal because of the edema. Urine protein is high. Ketones are not related to nephrotic syndrome.

A patient with primary hyperparathyroidism has a serum phosphorus level of 1.7 mg/dL (0.55 mmol/L) and calcium of 14 mg/dL (3.5 mmol/L). Which nursing action should be included in the plan of care? a. Restrict the patient to bed rest. b. Encourage 4000 mL of fluids daily. c. Institute routine seizure precautions. d. Assess for positive Chvostek's sign.

ANS: B The patient with hypercalcemia is at risk for kidney stones, which may be prevented by a high fluid intake. Seizure precautions and monitoring for Chvostek's or Trousseau's sign are appropriate for hypocalcemic patients. The patient should engage in weight-bearing exercise to decrease calcium loss from bone. DIF: Cognitive Level: Apply (application) REF: 1173 TOP: Nursing Process: Planning MSC: NCLEX: Physiological Integrity

Which intervention will the nurse include in the plan of care for a patient with moderate dementia who had a fractured hip repair 2 days ago? a. Provide complete personal hygiene care for the patient. b. Remind the patient frequently about being in the hospital. c. Reposition the patient frequently to avoid skin breakdown. d. Place suction at the bedside to decrease the risk for aspiration.

ANS: B The patient with moderate dementia will have problems with short- and long-term memory and will need reminding about the hospitalization. The other interventions would be used for a patient with severe dementia, who would have difficulty with swallowing, self-care, and immobility. DIF: Cognitive Level: Apply (application) REF: 1403 TOP: Nursing Process: Planning MSC: NCLEX: Physiological Integrity

For a patient who had a right hemisphere stroke, the nurse anticipates planning interventions to manage a. impaired physical mobility related to right-sided hemiplegia. b. risk for injury related to denial of deficits and impulsiveness. c. impaired verbal communication related to speech-language deficits. d. ineffective coping related to depression and distress about disability.

ANS: B The patient with right-sided brain damage typically denies any deficits and has poor impulse control, leading to risk for injury when the patient attempts activities such as transferring from a bed to a chair. Right-sided brain damage causes left hemiplegia. Left-sided brain damage typically causes language deficits. Left-sided brain damage is associated with depression and distress about the disability. DIF: Cognitive Level: Apply (application) REF: 1350 TOP: Nursing Process: Diagnosis MSC: NCLEX: Physiological Integrity

A 79-yr-old patient has been admitted with benign prostatic hyperplasia. What is most appropriate to include in the nursing plan of care? a. Limit fluid intake to no more than 1000 mL/day. b. Leave a light on in the bathroom during the night. c. Ask the patient to use a urinal so that urine can be measured. d. Pad the patient's bed to accommodate overflow incontinence.

ANS: B The patient's age and diagnosis indicate a likelihood of nocturia, so leaving the light on in the bathroom is appropriate. Fluids should be encouraged because dehydration is more common in older patients. The information in the question does not indicate that measurement of the patient's output is necessary or that the patient has overflow incontinence. DIF: Cognitive Level: Analyze (analysis) REF: 1022 TOP: Nursing Process: Planning MSC: NCLEX: Physiological Integrity

The oxygen saturation (SpO2) for a patient with left lower lobe pneumonia is 90%. The patient has wheezes, a weak cough effort, and complains of fatigue. Which action should the nurse take next? a. Position the patient on the left side. b. Assist the patient with staged coughing. c. Place a humidifier in the patient's room. d. Schedule a 4-hour rest period for the patient.

ANS: B The patient's assessment indicates that assisted coughing is needed to help remove secretions, which will improve oxygenation. A 4-hour rest period at this time may allow the O2 saturation to drop further. Humidification will not be helpful unless the secretions can be mobilized. Positioning on the left side may cause a further decrease in oxygen saturation because perfusion will be directed more toward the more poorly ventilated lung. DIF: Cognitive Level: Apply (application)

A patient who had a subtotal thyroidectomy earlier today develops laryngeal stridor and a cramp in the right hand upon returning to the surgical nursing unit. Which collaborative action will the nurse anticipate next? a. Suction the patient's airway. b. Administer IV calcium gluconate. c. Plan for emergency tracheostomy. d. Prepare for endotracheal intubation.

ANS: B The patient's clinical manifestations of stridor and cramping are consistent with tetany caused by hypocalcemia resulting from damage to the parathyroid glands during surgery. Endotracheal intubation or tracheostomy may be needed if the calcium does not resolve the stridor. Suctioning will not correct the stridor. DIF: Cognitive Level: Apply (application) REF: 1168 TOP: Nursing Process: Planning MSC: NCLEX: Physiological Integrity

During routine hemodialysis, a patient complains of nausea and dizziness. Which action should the nurse take first? a. Slow down the rate of dialysis. b. Check the blood pressure (BP). c. Review the hematocrit (Hct) level. d. Give prescribed PRN antiemetic drugs.

ANS: B The patient's complaints of nausea and dizziness suggest hypotension, so the initial action should be to check the BP. The other actions may also be appropriate based on the blood pressure obtained. DIF: Cognitive Level: Analyze (analysis) REF: 1090 OBJ: Special Questions: Prioritization TOP: Nursing Process: Implementation MSC: NCLEX: Physiological Integrity

After change-of-shift report on the neurology unit, which patient will the nurse assess first? a. Patient with Bell's palsy who has herpes vesicles in front of the ear b. Patient with botulism who is drooling and experiencing difficulty swallowing c. Patient with neurosyphilis who has tabes dorsalis and decreased deep tendon reflexes d. Patient with an abscess caused by injectable drug use who needs tetanus immune globulin

ANS: B The patient's diagnosis and difficulty swallowing indicate the nurse should rapidly assess for respiratory distress. The information about the other patients is consistent with their diagnoses and does not indicate any immediate need for assessment or intervention. DIF: Cognitive Level: Analyze (analysis)

A nurse is caring for a patient whose hemodynamic monitoring indicates a blood pressure of 92/54 mm Hg, a pulse of 64 beats/min, and an elevated pulmonary artery wedge pressure (PAWP). Which intervention ordered by the health care provider should the nurse question? a. Elevate head of bed to 30 degrees. b. Infuse normal saline at 250 mL/hr. c. Hold nitroprusside if systolic BP is less than 90 mm Hg. d. Titrate dobutamine to keep systolic BP is greater than 90 mm Hg.

ANS: B The patient's elevated PAWP indicates volume excess in relation to cardiac pumping ability, consistent with cardiogenic shock. A saline infusion at 250 mL/hr will exacerbate the volume excess. The other actions will help to improve cardiac output, which should lower the PAWP and may raise the BP. DIF: Cognitive Level: Apply (application)

After a thymectomy, a patient with myasthenia gravis receives the usual dose of pyridostigmine (Mestinon). An hour later, the patient complains of nausea and severe abdominal cramps. Which action should the nurse take first? a. Auscultate the patient's bowel sounds. b. Notify the patient's health care provider. c. Administer the prescribed PRN antiemetic drug. d. Give the scheduled dose of prednisone (Deltasone).

ANS: B The patient's history and symptoms indicate a possible cholinergic crisis. The health care provider should be notified immediately, and it is likely that atropine will be prescribed. The other actions will be appropriate if the patient is not experiencing a cholinergic crisis. DIF: Cognitive Level: Analyze (analysis) REF: 1394 OBJ: Special Questions: Prioritization TOP: Nursing Process: Implementation MSC: NCLEX: Physiological Integrity

An 81-yr-old patient who has been in the intensive care unit (ICU) for a week is now stable and transfer to the progressive care unit is planned. On rounds, the nurse notices that the patient has new onset confusion. The nurse will plan to a. give PRN lorazepam (Ativan) and cancel the transfer. b. inform the receiving nurse and then transfer the patient. c. notify the health care provider and postpone the transfer. d. obtain an order for restraints as needed and transfer the patient.

ANS: B The patient's history and symptoms most likely indicate delirium associated with the sleep deprivation and sensory overload in the ICU environment. Informing the receiving nurse and transferring the patient is appropriate. Postponing the transfer is likely to prolong the delirium. Benzodiazepines and restraints contribute to delirium and agitation. DIF: Cognitive Level: Apply (application)

A patient with respiratory failure has a respiratory rate of 6 breaths/min and an oxygen saturation (SpO2) of 88%. The patient is increasingly lethargic. Which intervention will the nurse anticipate? a. Administration of 100% O2 by non-rebreather mask b. Endotracheal intubation and positive pressure ventilation c. Insertion of a mini-tracheostomy with frequent suctioning d. Initiation of continuous positive pressure ventilation (CPAP)

ANS: B The patient's lethargy, low respiratory rate, and SpO2 indicate the need for mechanical ventilation with ventilator-controlled respiratory rate. Giving high-flow O2 will not be helpful because the patient's respiratory rate is so low. Insertion of a mini-tracheostomy will facilitate removal of secretions, but it will not improve the patient's respiratory rate or oxygenation. CPAP requires that the patient initiate an adequate respiratory rate to allow adequate gas exchange. DIF: Cognitive Level: Apply (application)

A patient complains of shoulder pain when the nurse moves the patient's arm behind the back. Which question should the nurse ask? a. "Are you able to feed yourself without difficulty?" b. "Do you have difficulty when you are putting on a shirt?" c. "Are you able to sleep through the night without waking?" d. "Do you ever have trouble lowering yourself to the toilet?"

ANS: B The patient's pain will make it more difficult to accomplish tasks such as putting on a shirt or jacket. This pain should not affect the patient's ability to feed himself or use the toilet because these tasks do not involve moving the arm behind the patient. The arm will not usually be positioned behind the patient during sleeping. DIF: Cognitive Level: Apply (application) REF: 1452 TOP: Nursing Process: Assessment MSC: NCLEX: Health Promotion and Maintenance

The nurse reviews the electronic health record for a patient scheduled for a total hip replacement. Which assessment data shown in the accompanying figure increase the patient's risk for respiratory complications after surgery? History Lab Data Physical Assessment *Age 81 H&H 11.8/38% *LCTA * Med/Surg history: Albumin 2.7 *Mildly confused: Recent 15lb weight loss, disoriented to date, Knee arthroscopy 3 oriented to person and place months ago a. Older age and anemia b. Albumin level and weight loss c. Recent arthroscopic procedure d. Confusion and disorientation to time

ANS: B The patient's recent weight loss and low protein stores indicate possible muscle weakness, which make it more difficult for an older patient to recover from the effects of general anesthesia and immobility associated with the hip surgery. The other information will also be noted by the nurse but does not place the patient at higher risk for respiratory failure. DIF: Cognitive Level: Analyze (analysis)

A patient develops carpopedal spasms and tingling of the lips following a parathyroidectomy. Which action will provide the patient with rapid relief from the symptoms? a. Administer the prescribed muscle relaxant. b. Have the patient rebreathe from a paper bag. c. Start the PRN O2 at 2 L/min per cannula. d. Stretch the muscles with passive range of motion.

ANS: B The patient's symptoms suggest mild hypocalcemia. The symptoms of hypocalcemia will be temporarily reduced by having the patient breathe into a paper bag, which will raise the PaCO2 and create a more acidic pH. Applying as-needed O2 or range of motion will have no impact on the ionized calcium level. Calcium supplements will be given to normalize calcium levels quickly, but oral supplements will take time to be absorbed. DIF: Cognitive Level: Apply (application) REF: 1174 TOP: Nursing Process: Implementation MSC: NCLEX: Physiological Integrity

A patient who had surgery for creation of an ileal conduit 3 days ago will not look at the stoma and requests that only the ostomy nurse specialist does the stoma care. The nurse identifies a nursing diagnosis of a. anxiety related to effects of procedure on lifestyle. b. disturbed body image related to change in function. c. readiness for enhanced coping related to need for information. d. self-care deficit (toileting) related to denial of altered body function.

ANS: B The patient's unwillingness to look at the stoma or participate in care indicates that disturbed body image is the best way to describe the problem. No data suggest that the impact on lifestyle is a concern for the patient. The patient does not appear to be ready for enhanced coping. The patient's insistence that only the ostomy nurse care for the stoma indicates that denial is not present.

After laminectomy with a spinal fusion to treat a herniated disc, a patient reports numbness and tingling of the right lower leg. The first action the nurse should take is to a. report the patient's complaint to the surgeon. b. check the chart for preoperative assessment data. c. check the vital signs for indications of hemorrhage. d. turn the patient to the left to relieve pressure on the right leg.

ANS: B The postoperative movement and sensation of the extremities should be unchanged (or improved) from the preoperative assessment. If the numbness and tingling are new, this information should be immediately reported to the surgeon. Numbness and tingling are not symptoms associated with hemorrhage at the site. Turning the patient will not relieve the numbness. DIF: Cognitive Level: Analyze (analysis) REF: 1507 OBJ: Special Questions: Prioritization TOP: Nursing Process: Implementation MSC: NCLEX: Physiological Integrity

The nurse is planning care for a patient with severe heart failure who has developed elevated blood urea nitrogen (BUN) and creatinine levels. The primary treatment goal in the plan will be a. augmenting fluid volume. . b. maintaining cardiac output. c. diluting nephrotoxic substances d. preventing systemic hypertension.

ANS: B The primary goal of treatment for acute kidney injury (AKI) is to eliminate the cause and provide supportive care while the kidneys recover. Because this patient's heart failure is causing AKI, the care will be directed toward treatment of the heart failure. For renal failure caused by hypertension, hypovolemia, or nephrotoxins, the other responses would be correct. DIF: Cognitive Level: Apply (application) REF: 1073 TOP: Nursing Process: Planning MSC: NCLEX: Physiological Integrity

A patient who is suspected of having an epidural hematoma is admitted to the emergency department. Which action will the nurse expect to take? a. Administer IV furosemide (Lasix). b. Prepare the patient for craniotomy. c. Initiate high-dose barbiturate therapy. d. Type and crossmatch for blood transfusion.

ANS: B The principal treatment for epidural hematoma is rapid surgery to remove the hematoma and prevent herniation. If intracranial pressure is elevated after surgery, furosemide or high-dose barbiturate therapy may be needed, but these will not be of benefit unless the hematoma is removed. Minimal blood loss occurs with head injuries, and transfusion is usually not necessary. DIF: Cognitive Level: Apply (application)

Which cerebrospinal fluid analysis result should the nurse recognize as abnormal and communicate to the health care provider? a. Specific gravity of 1.007 b. Protein of 65 mg/dL (0.65 g/L) c. Glucose of 45 mg/dL (1.7 mmol/L) d. White blood cell (WBC) count of 4 cells/μL

ANS: B The protein level is high. The specific gravity, WBCs, and glucose values are normal. DIF: Cognitive Level: Understand (comprehension) REF: 1298 TOP: Nursing Process: Implementation MSC: NCLEX: Physiological Integrity

The nurse will teach a patient who is scheduled to complete a 24-hour urine collection for 17-ketosteroids to a. insert and maintain a retention catheter. b. keep the specimen refrigerated or on ice. c. drink at least 3 L of fluid during the 24 hours. d. void and save that specimen to start the collection.

ANS: B The specimen must be kept on ice or refrigerated until the collection is finished. Voided or catheterized specimens are acceptable for the test. The initial voided specimen is discarded. There is no fluid intake requirement for the 24-hour collection. DIF: Cognitive Level: Apply (application) REF: 1117 TOP: Nursing Process: Implementation MSC: NCLEX: Physiological Integrity

A nurse is caring for a patient with ARDS who is being treated with mechanical ventilation and high levels of positive end-expiratory pressure (PEEP). Which assessment finding by the nurse indicates that the PEEP may need to be reduced? a. The patient's PaO2 is 50 mm Hg and the SaO2 is 88%. b. The patient has subcutaneous emphysema on the upper thorax. c. The patient has bronchial breath sounds in both the lung fields. d. The patient has a first-degree atrioventricular heart block with a rate of 58 beats/min.

ANS: B The subcutaneous emphysema indicates barotrauma caused by positive pressure ventilation and PEEP. Bradycardia, hypoxemia, and bronchial breath sounds are all concerns and will need to be addressed, but they are not specific indications that PEEP should be reduced. DIF: Cognitive Level: Apply (application)

Which information will the nurse include when teaching a patient who has type 2 diabetes about glyburide ? a. Glyburide decreases glucagon secretion from the pancreas. b. Glyburide stimulates insulin production and release from the pancreas. c. Glyburide should be taken even if the morning blood glucose level is low. d. Glyburide should not be used for 48 hours after receiving IV contrast media.

ANS: B The sulfonylureas stimulate the production and release of insulin from the pancreas. If the glucose level is low, the patient should contact the health care provider before taking glyburide because hypoglycemia can occur with this class of medication. Metformin should be held for 48 hours after administration of IV contrast media, but this is not necessary for glyburide. Glucagon secretion is not affected by glyburide. DIF: Cognitive Level: Apply (application) REF: 1130 TOP: Nursing Process: Implementation MSC: NCLEX: Physiological Integrity

Which action will the nurse anticipate taking for an otherwise healthy 50-yr-old who has just been diagnosed with stage 1 renal cell carcinoma? a. Prepare patient for a renal biopsy. b. Provide preoperative teaching about nephrectomy. c. Teach the patient about chemotherapy medications. d. Schedule for a follow-up appointment in 3 months.

ANS: B The treatment of choice in patients with localized renal tumors who have no co-morbid conditions is partial or total nephrectomy. A renal biopsy will not be needed in a patient who has already been diagnosed with renal cancer. Chemotherapy is used for metastatic renal cancer. Because renal cell cancer frequently metastasizes, treatment will be started as soon as possible after the diagnosis.

The health care provider is considering the use of sumatriptan (Imitrex) for a 54-yr-old male patient with migraine headaches. Which information obtained by the nurse is most important to report to the health care provider? a. The patient drinks 1 to 2 cups of coffee daily. b. The patient had a recent acute myocardial infarction. c. The patient has had migraine headaches for 30 years. d. The patient has taken topiramate (Topamax) for 2 months.

ANS: B The triptans cause coronary artery vasoconstriction and should be avoided in patients with coronary artery disease. The other information will be reported to the health care provider, but none of it indicates that sumatriptan would be an inappropriate treatment. DIF: Cognitive Level: Analyze (analysis) REF: 1372 OBJ: Special Questions: Prioritization TOP: Nursing Process: Assessment MSC: NCLEX: Physiological Integrity

A female patient with a suspected urinary tract infection (UTI) is to provide a clean-catch urine specimen for culture and sensitivity testing. To obtain the specimen, the nurse will a. have the patient empty the bladder completely; then obtain the next urine specimen that the patient is able to void. b. teach the patient to clean the urethral area, void a small amount into the toilet, and then void into a sterile specimen cup. c. insert a short sterile "mini" catheter attached to a collecting container into the urethra and bladder to obtain the specimen. d. clean the area around the meatus with a povidone-iodine (Betadine) swab and then have the patient void into a sterile container.

ANS: B This answer describes the technique for obtaining a clean-catch specimen. The answer beginning, "insert a short, small, 'mini' catheter attached to a collecting container" describes a technique that would result in a sterile specimen, but a health care provider's order for a catheterized specimen would be required. Using Betadine before obtaining the specimen is not necessary and might result in suppressing the growth of some bacteria. The technique described in the answer beginning "have the patient empty the bladder completely" would not result in a sterile specimen. DIF: Cognitive Level: Apply (application) REF: 1025 TOP: Nursing Process: Implementation MSC: NCLEX: Physiological Integrity

Before administering botulinum antitoxin to a patient in the emergency department, it is most important for the nurse to a. obtain the patient's temperature. b. administer an intradermal test dose. c. document the neurologic symptoms. d. ask the patient about an allergy to eggs.

ANS: B To assess for possible allergic reactions, an intradermal test dose of the antitoxin should be administered. Although temperature, allergy history, and symptom assessment and documentation are appropriate, these assessments will not affect the decision to administer the antitoxin. DIF: Cognitive Level: Analyze (analysis)

When administering alendronate (Fosamax) to a patient with osteoporosis, the nurse will a. ask about any leg cramps or hot flashes. b. assist the patient to sit up at the bedside. c. be sure that the patient has recently eaten. d. administer the ordered calcium carbonate.

ANS: B To avoid esophageal erosions, the patient taking bisphosphonates should be upright for at least 30 minutes after taking the medication. Fosamax should be taken on an empty stomach, not after taking other medications or eating. Leg cramps and hot flashes are not side effects of bisphosphonates. DIF: Cognitive Level: Apply (application) REF: 1513 TOP: Nursing Process: Implementation MSC: NCLEX: Physiological Integrity

An active 32-yr-old male who has type 1 diabetes is being seen in the endocrine clinic. Which finding indicates a need for the nurse to discuss a possible a change in therapy with the health care provider? a. Hemoglobin A1C level of 6.2% b. Blood pressure of 140/88 mmHg c. Heart rate at rest of 58 beats/minute d. High density lipoprotein (HDL) level of 65 mg/dL

ANS: B To decrease the incidence of macrovascular and microvascular problems in patients with diabetes, the goal blood pressure is usually 130/80 mm Hg. An A1C less than 6.5%, a low resting heart rate (consistent with regular aerobic exercise in a young adult), and an HDL level of 65 mg/dL all indicate that the patient's diabetes and risk factors for vascular disease are well controlled. DIF: Cognitive Level: Apply (application) REF: 1148 TOP: Nursing Process: Assessment MSC: NCLEX: Physiological Integrity

Which information about the patient who has had a subarachnoid hemorrhage is most important to communicate to the health care provider? a. The patient complains of having a stiff neck. b. The patient's blood pressure (BP) is 90/50 mm Hg. c. The patient reports a severe and unrelenting headache. d. The cerebrospinal fluid (CSF) report shows red blood cells (RBCs).

ANS: B To prevent cerebral vasospasm and maintain cerebral perfusion, BP needs to be maintained at a level higher than 90 mm Hg systolic after a subarachnoid hemorrhage. A low BP or drop in BP indicates a need to administer fluids and/or vasopressors to increase the BP. An ongoing headache, RBCs in the CSF, and a stiff neck are all typical clinical manifestations of a subarachnoid hemorrhage and do not need to be rapidly communicated to the health care provider. DIF: Cognitive Level: Analyze (analysis) REF: 1350 OBJ: Special Questions: Prioritization TOP: Nursing Process: Assessment MSC: NCLEX: Physiological Integrity

A patient who has diabetes and reported burning foot pain at night receives a new prescription. Which information should the nurse teach the patient about amitriptyline ? a. Amitriptyline decreases the depression caused by your foot pain. b. Amitriptyline helps prevent transmission of pain impulses to the brain. c. Amitriptyline corrects some of the blood vessel changes that cause pain. d. Amitriptyline improves sleep and makes you less aware of nighttime pain.

ANS: B Tricyclic antidepressants (TCAs) decrease the transmission of pain impulses to the spinal cord and brain. TCAs also improve sleep quality and are used for depression, but that is not the major purpose for their use in diabetic neuropathy. The blood vessel changes that contribute to neuropathy are not affected by TCAs. DIF: Cognitive Level: Apply (application) REF: 1150 TOP: Nursing Process: Implementation MSC: NCLEX: Physiological Integrity

The nurse expects the assessment of a patient who is experiencing a cluster headache to include a. nuchal rigidity. b. unilateral ptosis. c. projectile vomiting. d. throbbing, bilateral facial pain.

ANS: B Unilateral eye edema, tearing, and ptosis are characteristic of cluster headaches. Nuchal rigidity suggests meningeal irritation, such as occurs with meningitis. Although nausea and vomiting may occur with migraine headaches, projectile vomiting is more consistent with increased intracranial pressure. Unilateral sharp, stabbing pain, rather than throbbing pain, is characteristic of cluster headaches. DIF: Cognitive Level: Understand (comprehension) REF: 1371 TOP: Nursing Process: Assessment MSC: NCLEX: Physiological Integrity

Which equipment will the nurse obtain to assess vibration sense in a patient with diabetes who has peripheral nerve dysfunction? a. Sharp pin b. Tuning fork c. Reflex hammer d. Calibrated compass

ANS: B Vibration sense is testing by touching the patient with a vibrating tuning fork. The other equipment is needed for testing of pain sensation, reflexes, and two-point discrimination. DIF: Cognitive Level: Understand (comprehension) REF: 1307 TOP: Nursing Process: Assessment MSC: NCLEX: Physiological Integrity

The priority nursing assessment for a patient being admitted with a brainstem infarction is a. pupil reaction. b. respiratory rate. c. reflex reaction time. d. level of consciousness.

ANS: B Vital centers that control respiration are located in the medulla and part of the brainstem, and will require priority assessments because changes in respiratory function may be life threatening. The other information will also be obtained by the nurse but is not as urgent. DIF: Cognitive Level: Apply (application) REF: 1297 OBJ: Special Questions: Prioritization TOP: Nursing Process: Assessment MSC: NCLEX: Physiological Integrity

A patient has been newly diagnosed with type 2 diabetes mellitus. Which information about the patient will be most useful to the nurse who is helping the patient develop strategies for successful adaptation to this disease? a. Ideal weight c. Activity level b. Value system d. Visual changes

ANS: B When dealing with a patient with a chronic condition such as diabetes, identification of the patient's values and beliefs can assist the interprofessional team in choosing strategies for successful lifestyle change. The other information also will be useful but is not as important in developing an individualized plan for the necessary lifestyle changes. DIF: Cognitive Level: Analyze (analysis) REF: 1111 TOP: Nursing Process: Assessment MSC: NCLEX: Psychosocial Integrity

A patient who has had progressive chronic kidney disease (CKD) for several years has just begun regular hemodialysis. Which information about diet will the nurse include in patient teaching? a. Increased calories are needed because glucose is lost during hemodialysis. b. More protein is allowed because urea and creatinine are removed by dialysis. c. Dietary potassium is not restricted because the level is normalized by dialysis. d. Unlimited fluids are allowed because retained fluid is removed during dialysis.

ANS: B When the patient is started on dialysis and nitrogenous wastes are removed, more protein in the diet is encouraged. Fluids are still restricted to avoid excessive weight gain and complications such as shortness of breath. Glucose is not lost during hemodialysis. Sodium and potassium intake continues to be restricted to avoid the complications associated with high levels of these electrolytes. DIF: Cognitive Level: Apply (application) REF: 1087 TOP: Nursing Process: Implementation MSC: NCLEX: Physiological Integrity

A 56-yr-old patient who is disoriented and reports a headache and muscle cramps is hospitalized with possible syndrome of inappropriate antidiuretic hormone (SIADH). The nurse would expect the initial laboratory results to include a(n) a. elevated hematocrit. c. increased serum chloride. b. decreased serum sodium. d. low urine specific gravity.

ANS: B When water is retained, the serum sodium level will drop below normal, causing the clinical manifestations reported by the patient. The hematocrit will decrease because of the dilution caused by water retention. Urine will be more concentrated with a higher specific gravity. The serum chloride level will usually decrease along with the sodium level. DIF: Cognitive Level: Understand (comprehension) REF: 1160 TOP: Nursing Process: Assessment MSC: NCLEX: Physiological Integrity

Which actions could the nurse delegate to a licensed practical/vocational nurse (LPN/LVN) who is part of the team caring for a patient with Alzheimer's disease (select all that apply)? a. Develop a plan to minimize difficult behavior. b. Administer the prescribed memantine (Namenda). c. Remove potential safety hazards from the patient's environment. d. Refer the patient and caregivers to appropriate community resources. e. Help the patient and caregivers choose memory enhancement methods. f. Evaluate the effectiveness of the prescribed enteral feedings on patient nutrition.

ANS: B, C LPN/LVN education and scope of practice includes medication administration and monitoring for environmental safety in stable patients. Planning of interventions such as ways to manage behavior or improve memory, referrals, and evaluation of the effectiveness of interventions require registered nurse (RN)-level education and scope of practice. DIF: Cognitive Level: Apply (application) REF: 1412 OBJ: Special Questions: Delegation TOP: Nursing Process: Planning MSC: NCLEX: Safe and Effective Care Environment

Which assessments will the nurse make to monitor a patient's cerebellar function (select all that apply)? a. Test for graphesthesia. b. Observe arm swing with gait. c. Perform the finger-to-nose test. d. Assess heat and cold sensation. e. Measure strength against resistance.

ANS: B, C The cerebellum is responsible for coordination and is assessed by looking at the patient's gait and the finger-to-nose test. The other assessments will be used for other parts of the neurologic assessment. DIF: Cognitive Level: Apply (application) REF: 1306 TOP: Nursing Process: Assessment MSC: NCLEX: Health Promotion and Maintenance

Which actions should the nurse start to reduce the risk for ventilator-associated pneumonia (VAP) (select all that apply)? a. Obtain arterial blood gases daily. b. Provide a "sedation holiday" daily. c. Give prescribed pantoprazole (Protonix). d. Elevate the head of the bed to at least 30°. e. Provide oral care with chlorhexidine (0.12%) solution daily.

ANS: B, C, D, E All of these interventions are part of the ventilator bundle that is recommended to prevent VAP. Arterial blood gases may be done daily but are not always necessary and do not help prevent VAP.

1. To monitor for complications in a patient with type 2 diabetes, which tests will the nurse in the diabetic clinic schedule at least annually (select all that apply)? a. Chest x-ray b. Blood pressure c. Serum creatinine d. Urine for microalbuminuria e. Complete blood count (CBC) f. Monofilament testing of the foot

ANS: B, C, D, F Blood pressure, serum creatinine, urine testing for microalbuminuria, and monofilament testing of the foot are recommended at least annually to screen for possible microvascular and macrovascular complications of diabetes. Chest x-ray and CBC might be ordered if the patient with diabetes presents with symptoms of respiratory or infectious problems but are not routinely included in screening. DIF: Cognitive Level: Apply (application) REF: 1148 TOP: Nursing Process: Planning MSC: NCLEX: Physiological Integrity

The spouse of a 67-yr-old male patient with early stage Alzheimer's disease (AD) tells the nurse, "I am exhausted from worrying all the time. I don't know what to do." Which actions are best for the nurse to take next (select all that apply)? a. Suggest that a long-term care facility be considered. b. Offer ideas for ways to distract or redirect the patient. c. Teach the spouse about adult day care as a possible respite. d. Suggest that the spouse consult with the physician for antianxiety drugs. e. Ask the spouse what she knows and has considered about dementia care options.

ANS: B, C, E The stress of being a caregiver can be managed with a multicomponent approach. This includes respite care, learning ways to manage challenging behaviors, and further assessment of what the spouse may already have considered for care options. The patient is in the early stages and does not need long-term placement. Antianxiety medications may be appropriate, but other measures should be tried first. DIF: Cognitive Level: Apply (application) REF: 1411 TOP: Nursing Process: Implementation MSC: NCLEX: Psychosocial Integrity

A patient has been diagnosed with urinary tract calculi that are high in uric acid. Which foods will the nurse teach the patient to avoid (select all that apply)? a. Milk b. Liver c. Spinach d. Chicken e. Cabbage f. Chocolate

ANS: B, D Meats contain purines, which are metabolized to uric acid. The other foods might be restricted in patients who have calcium or oxalate stones.

A 40-yr-old patient is diagnosed with early Huntington's disease (HD). When teaching the patient, spouse, and adult children about this disorder, the nurse will provide information about the a. use of levodopa-carbidopa (Sinemet) to help reduce HD symptoms. b. prophylactic antibiotics to decrease the risk for aspiration pneumonia. c. option of genetic testing for the patient's children to determine their own HD risks. d. lifestyle changes of improved nutrition and exercise that delay disease progression.

ANS: C Genetic testing is available to determine whether an asymptomatic individual has the HD gene. The patient and family should be informed of the benefits and problems associated with genetic testing. Sinemet will increase symptoms of HD because HD involves an increase in dopamine. Antibiotic therapy will not reduce the risk for aspiration. There are no effective treatments or lifestyle changes that delay the progression of symptoms in HD. DIF: Cognitive Level: Apply (application) REF: 1396 TOP: Nursing Process: Implementation MSC: NCLEX: Physiological Integrity

A patient with Alzheimer's disease (AD) who is being admitted to a long-term care facility has had several episodes of wandering away from home. Which action will the nurse include in the plan of care? a. Reorient the patient several times daily. b. Have the family bring in familiar items. c. Place the patient in a room close to the nurses' station. d. Ask the patient why the wandering episodes have occurred.

ANS: C Patients at risk for problems with safety require close supervision. Placing the patient near the nurse's station will allow nursing staff to observe the patient more closely. The use of "why" questions can be frustrating for patients with AD because they are unable to understand clearly or verbalize the reason for wandering behaviors. Because of the patient's short-term memory loss, reorientation will not help prevent wandering behavior. Because the patient had wandering behavior at home, familiar objects will not prevent wandering. DIF: Cognitive Level: Apply (application) REF: 1412 TOP: Nursing Process: Planning MSC: NCLEX: Safe and Effective Care Environment

Which information will the nurse include when teaching the patient with a urinary tract infection (UTI) about the use of phenazopyridine? a. Take phenazopyridine for at least 7 days. b. Phenazopyridine may cause photosensitivity c. Phenazopyridine may change the urine color d. Take phenazopyridine before sexual intercourse.

ANS: C Patients should be taught that phenazopyridine will color the urine deep orange. Urinary analgesics should only be needed for a few days until the prescribed antibiotics decrease the bacterial count. Phenazopyridine does not cause photosensitivity. Taking phenazopyridine before intercourse will not be helpful in reducing the risk for UTI.

An intraaortic balloon pump (IABP) is being used for a patient who is in cardiogenic shock. Which assessment data indicate to the nurse that the goals of treatment with the IABP are being met? a. Urine output of 25 mL/hr b. Heart rate of 110 beats/minute c. Cardiac output (CO) of 5 L/min d. Stroke volume (SV) of 40 mL/beat

ANS: C A CO of 5 L/min is normal and indicates that the IABP has been successful in treating the shock. The low SV signifies continued cardiogenic shock. The tachycardia and low urine output also suggest continued cardiogenic shock. DIF: Cognitive Level: Apply (application)

Which laboratory value should the nurse review to determine whether a patient's hypothyroidism is caused by a problem with the anterior pituitary gland or with the thyroid gland? a. Thyroxine (T4) level b. Triiodothyronine (T3) level c. Thyroid-stimulating hormone (TSH) level d. Thyrotropin-releasing hormone (TRH) level

ANS: C A low TSH level indicates that the patient's hypothyroidism is caused by decreased anterior pituitary secretion of TSH. Low T3and T4 levels are not diagnostic of the primary cause of the hypothyroidism. TRH levels indicate the function of the hypothalamus. DIF: Cognitive Level: Apply (application) REF: 1116 TOP: Nursing Process: Assessment MSC: NCLEX: Physiological Integrity

A 38-yr-old patient who had a kidney transplant 8 years ago is receiving the immunosuppressants tacrolimus (Prograf), cyclosporine (Sandimmune), and prednisone . Which assessment data will be of most concern to the nurse? a. Skin is thin and fragile. b. Blood pressure is 150/92. c. A nontender axillary lump. d. Blood glucose is 144 mg/dL.

ANS: C A nontender lump suggests a malignancy such as a lymphoma, which could occur as a result of chronic immunosuppressive therapy. The elevated glucose, skin change, and hypertension are possible side effects of the prednisone and should be addressed, but they are not as great a concern as the possibility of a malignancy. DIF: Cognitive Level: Analyze (analysis) REF: 1096 OBJ: Special Questions: Prioritization TOP: Nursing Process: Assessment MSC: NCLEX: Physiological Integrity

A male patient in the clinic provides a urine sample that is red-orange in color. Which action should the nurse take? a. Notify the patient's health care provider. b. Teach correct midstream urine collection. c. Ask the patient about current medications. d. Question the patient about urinary tract infection (UTI) risk factors.

ANS: C A red-orange color in the urine is normal with some over-the-counter (OTC) medications such as phenazopyridine . The color would not be expected with urinary tract infection, is not a sign that poor technique was used in obtaining the specimen, and does not need to be communicated to the health care provider until further assessment is done. DIF: Cognitive Level: Apply (application) REF: 1020 TOP: Nursing Process: Assessment MSC: NCLEX: Physiological Integrity

A patient is being admitted with a possible stroke. Which information from the assessment indicates that the nurse should consult with the health care provider before giving the prescribed aspirin? a. The patient has dysphasia. b. The patient has atrial fibrillation. c. The patient reports that symptoms began with a severe headache. d. The patient has a history of brief episodes of right-sided hemiplegia.

ANS: C A sudden onset headache is typical of a subarachnoid hemorrhage, and aspirin is contraindicated. Atrial fibrillation, dysphasia, and transient ischemic attack are not contraindications to aspirin use. DIF: Cognitive Level: Apply (application) REF: 1348 TOP: Nursing Process: Assessment MSC: NCLEX: Physiological Integrity

To determine possible causes, the nurse will ask a patient admitted with acute glomerulonephritis about a. recent bladder infection. b. history of kidney stones. c. recent sore throat and fever. d. history of high blood pressure.

ANS: C Acute glomerulonephritis frequently occurs after a streptococcal infection such as strep throat. It is not caused by kidney stones, hypertension, or urinary tract infection.

The nurse is caring for a patient who has been experiencing stroke symptoms for 60 minutes. Which action can the nurse delegate to a licensed practical/vocational nurse (LPN/LVN)? a. Assess the patient's gag and cough reflexes. b. Determine when the stroke symptoms began. c. Administer the prescribed short-acting insulin. d. Infuse the prescribed IV metoprolol (Lopressor).

ANS: C Administration of subcutaneous medications is included in LPN/LVN education and scope of practice. The other actions require more education and scope of practice and should be done by the registered nurse (RN). DIF: Cognitive Level: Apply (application) REF: 1359 OBJ: Special Questions: Delegation TOP: Nursing Process: Planning MSC: NCLEX: Safe and Effective Care Environment

The nurse is caring for a patient following an adrenalectomy. The highest priority in the immediate postoperative period is to a. protect the patient's skin. c. balance fluids and electrolytes. b. monitor for signs of infection. d. prevent emotional disturbances.

ANS: C After adrenalectomy, the patient is at risk for circulatory instability caused by fluctuating hormone levels, and the focus of care is to assess and maintain fluid and electrolyte status through the use of IV fluids and corticosteroids. The other goals are also important for the patient but are not as immediately life threatening as the circulatory collapse that can occur with fluid and electrolyte disturbances. DIF: Cognitive Level: Analyze (analysis) REF: 1177 OBJ: Special Questions: Prioritization TOP: Nursing Process: Planning MSC: NCLEX: Physiological Integrity

During change-of-shift report on a medical unit, the nurse learns that a patient with aspiration pneumonia who was admitted with respiratory distress has become increasingly agitated. Which action should the nurse take first? a. Give the prescribed PRN sedative drug. b. Offer reassurance and reorient the patient. c. Use pulse oximetry to check the oxygen saturation. d. Notify the health care provider about the patient's status.

ANS: C Agitation may be an early indicator of hypoxemia. The other actions may also be appropriate, depending on the findings about O2 saturation. DIF: Cognitive Level: Analyze (analysis)

A patient has just arrived on the unit after a thyroidectomy. Which action should the nurse take first? a. Observe the dressing for bleeding. b. Check the blood pressure and pulse. c. Assess the patient's respiratory effort. d. Support the patient's head with pillows.

ANS: C Airway obstruction is a possible complication after thyroidectomy because of swelling or bleeding at the site or tetany. The priority nursing action is to assess the airway. The other actions are also part of the standard nursing care postthyroidectomy but are not as high of a priority. DIF: Cognitive Level: Analyze (analysis) REF: 1168 OBJ: Special Questions: Prioritization TOP: Nursing Process: Implementation MSC: NCLEX: Physiological Integrity

Family members are in the patient's room when the patient has a cardiac arrest and the staff start resuscitation measures. Which action should the nurse take next? a. Keep the family in the room and assign a staff member to explain the care given and answer questions. b. Ask the family to wait outside the patient's room with a designated staff member to provide emotional support. c. Ask the family members whether they would prefer to remain in the patient's room or wait outside the room. d. Tell the family members that patients are comforted by having family members present during resuscitation efforts.

ANS: C Although many family members and patients report benefits from family presence during resuscitation efforts, the nurse's initial action should be to determine the preference of these family members. The other actions may be appropriate, but this will depend on what is learned when assessing family preferences. DIF: Cognitive Level: Analyze (analysis)

A patient with a T4 spinal cord injury asks the nurse if he will be able to be sexually active. Which initial response by the nurse is best? a. Reflex erections frequently occur, but orgasm may not be possible. b. Sildenafil (Viagra) is used by many patients with spinal cord injury. c. Multiple options are available to maintain sexuality after spinal cord injury. d. Penile injection, prostheses, or vacuum suction devices are possible options.

ANS: C Although sexuality will be changed by the patient's spinal cord injury, there are options for expression of sexuality and for fertility. The other information also is correct, but the choices will depend on the degrees of injury and the patient's individual feelings about sexuality. DIF: Cognitive Level: Analyze (analysis)

Anakinra (Kineret) is prescribed for a patient with rheumatoid arthritis (RA). When teaching the patient about this drug, the nurse will include information about a. avoiding concurrent aspirin use. b. symptoms of gastrointestinal (GI) bleeding. c. self-administration of subcutaneous injections. d. taking the medication with at least 8 oz of fluid.

ANS: C Anakinra is administered by subcutaneous injection. GI bleeding is not a side effect of this medication. Because the medication is injected, instructions to take it with 8 oz of fluid would not be appropriate. The patient is likely to be concurrently taking aspirin or nonsteroidal antiinflammatory drugs (NSAIDs), and these should not be discontinued. DIF: Cognitive Level: Apply (application) REF: 1529 TOP: Nursing Process: Implementation MSC: NCLEX: Physiological Integrity

After a patient with a pituitary adenoma has had a hypophysectomy, the nurse will teach about the need for a. sodium restriction to prevent fluid retention. b. insulin to maintain normal blood glucose levels. c. oral corticosteroids to replace endogenous cortisol. d. chemotherapy to prevent malignant tumor recurrence.

ANS: C Antidiuretic hormone (ADH), cortisol, and thyroid hormone replacement will be needed for life after hypophysectomy. Without the effects of adrenocorticotropic hormone (ACTH) and cortisol, the blood glucose and serum sodium will be low unless cortisol is replaced. An adenoma is a benign tumor, and chemotherapy will not be needed. DIF: Cognitive Level: Apply (application) REF: 1158 TOP: Nursing Process: Planning MSC: NCLEX: Physiological Integrity

Which diagnostic test will provide the nurse with the most specific information to evaluate the effectiveness of interventions for a patient with ventilatory failure? a. Chest x-ray b. O2 saturation c. Arterial blood gas analysis d. Central venous pressure monitoring

ANS: C Arterial blood gas (ABG) analysis is most useful in this setting because ventilatory failure causes problems with CO2 retention, and ABGs provide information about the PaCO2 and pH. The other tests may also be done to help in assessing oxygenation or determining the cause of the patient's ventilatory failure. DIF: Cognitive Level: Apply (application)

Which assessment finding may indicate that a patient is experiencing adverse effects to a corticosteroid prescribed after kidney transplantation? a. Postural hypotension b. Recurrent tachycardia c. Knee and hip joint pain d. Increased serum creatinine

ANS: C Aseptic necrosis of the weight-bearing joints can occur when patients take corticosteroids over a prolonged period. Increased creatinine level, orthostatic dizziness, and tachycardia are not caused by corticosteroid use. DIF: Cognitive Level: Apply (application) REF: 1096 TOP: Nursing Process: Evaluation MSC: NCLEX: Physiological Integrity

A patient with an acute attack of gout in the right great toe has a new prescription for probenecid. Which information about the patient's home routine indicates a need for teaching regarding gout management? a. The patient sleeps 8-10 hours each night. b. The patient usually eats beef once a week. c. The patient takes one aspirin a day to prevent angina. d. The patient usually drinks about 3 quarts water each day.

ANS: C Aspirin interferes with the effectiveness of probenecid and should not be taken when the patient is taking probenecid. The patient's sleep pattern will not affect gout management. Drinking 3 quarts of water and eating beef only once or twice a week are appropriate for the patient with gout. DIF: Cognitive Level: Apply (application) REF: 1534 TOP: Nursing Process: Assessment MSC: NCLEX: Physiological Integrity

A patient who has a history of a transient ischemic attack (TIA) has an order for aspirin 160 mg daily. When the nurse is administering medications, the patient says, "I don't need the aspirin today. I don't have a fever." Which action should the nurse take? a. Document that the aspirin was refused by the patient. b. Tell the patient that the aspirin is used to prevent a fever. c. Explain that the aspirin is ordered to decrease stroke risk. d. Call the health care provider to clarify the medication order.

ANS: C Aspirin is ordered to prevent stroke in patients who have experienced TIAs. Documentation of the patient's refusal to take the medication is an inadequate response by the nurse. There is no need to clarify the order with the health care provider. The aspirin is not ordered to prevent aches and pains. DIF: Cognitive Level: Apply (application) REF: 1353 TOP: Nursing Process: Implementation MSC: NCLEX: Physiological Integrity

A 28-yr-old male patient is diagnosed with polycystic kidney disease. Which information is most appropriate for the nurse to include in teaching at this time? a. Complications of renal transplantation b. Methods for treating severe chronic pain c. Options to consider for genetic counseling d. Differences between hemodialysis and peritoneal dialysis

ANS: C Because a 28-yr-old patient may be considering having children, the nurse should include information about genetic counseling when teaching the patient. A well-managed patient will not need to choose between hemodialysis and peritoneal dialysis or know about the effects of transplantation for many years. There is no indication that the patient has chronic pain.

Which nursing action will be included in the plan of care for a patient who has had cerebral angiography? a. Monitor for headache and photophobia. b. Keep patient NPO until gag reflex returns. c. Check pulse and blood pressure frequently. d. Assess orientation to person, place, and time.

ANS: C Because a catheter is inserted into an artery (e.g., the femoral artery) during cerebral angiography, the nurse should assess for bleeding after this procedure that can affect pulse and blood pressure. The other nursing assessments are not needed after angiography. DIF: Cognitive Level: Apply (application) REF: 1310 TOP: Nursing Process: Planning MSC: NCLEX: Physiological Integrity

A patient is taking spironolactone (Aldactone), a drug that blocks the action of aldosterone on the kidney, for hypertension. The nurse will monitor for a. increased serum sodium. b. decreased urinary output. c. elevated serum potassium. d. evidence of fluid overload.

ANS: C Because aldosterone increases the excretion of potassium, a medication that blocks aldosterone will tend to cause hyperkalemia. Aldosterone also promotes the reabsorption of sodium and water in the renal tubules, so spironolactone will tend to cause increased urine output, a decreased or normal serum sodium level, and signs of dehydration. DIF: Cognitive Level: Apply (application) REF: 1108 TOP: Nursing Process: Evaluation MSC: NCLEX: Physiological Integrity

The nurse is planning care for a patient with hypertension and gout who has a red, painful right great toe. Which nursing action will be included in the plan of care? a. Gently palpate the toe to assess swelling. b. Use pillows to keep the right foot elevated. c. Use a footboard to hold bedding away from the toe. d. Teach the patient to avoid use of acetaminophen (Tylenol).

ANS: C Because any touch on the area of inflammation may increase pain, bedding should be held away from the toe, and touching the toe should be avoided. Elevation of the foot will not reduce the pain, which is caused by urate crystals. Acetaminophen can be used for pain management. DIF: Cognitive Level: Understand (comprehension) REF: 1534 TOP: Nursing Process: Planning MSC: NCLEX: Physiological Integrity

A patient admitted with possible stroke has been aphasic for 3 hours, and his current blood pressure (BP) is 174/94 mm Hg. Which order by the health care provider should the nurse question? a. Keep head of bed elevated at least 30 degrees. b. Infuse normal saline intravenously at 75 mL/hr. c. Start a labetalol drip to keep BP less than 140/90 mm Hg. d. Administer tissue plasminogen activator (tPA) intravenously per protocol.

ANS: C Because elevated BP may be a protective response to maintain cerebral perfusion, antihypertensive therapy is recommended only if mean arterial pressure (MAP) is greater than130 mm Hg or systolic pressure is greater than 220 mm Hg. Fluid intake should be 1500 to 2000 mL/day to maintain cerebral blood flow. The head of the bed should be elevated to at least 30 degrees unless the patient has symptoms of poor tissue perfusion. tPA may be administered if the patient meets the other criteria for tPA use. DIF: Cognitive Level: Apply (application) REF: 1354 TOP: Nursing Process: Implementation MSC: NCLEX: Physiological Integrity

After the nurse has finished teaching a patient who has a new prescription for exenatide (Byetta), which patient statement indicates that the teaching has been effective? a. "I may feel hungrier than usual when I take this medicine." b. "I will not need to worry about hypoglycemia with the Byetta." c. "I should take my daily aspirin at least an hour before the Byetta." d. "I will take the pill at the same time I eat breakfast in the morning."

ANS: C Because exenatide slows gastric emptying, oral medications should be taken at least 1 hour before the exenatide to avoid slowing absorption. Exenatide is injected and increases feelings of satiety. Hypoglycemia can occur with this medication. DIF: Cognitive Level: Apply (application) REF: 1132 TOP: Nursing Process: Evaluation MSC: NCLEX: Physiological Integrity

A patient who has deep human bite wounds on the left hand is being treated in the urgent care center. Which action will the nurse plan to take? a. Prepare to administer rabies immune globulin (BayRab). b. Assist the health care provider with suturing of the bite wounds. c. Teach the patient the reason for the use of prophylactic antibiotics. d. Keep the wounds dry until the health care provider can assess them.

ANS: C Because human bites of the hand frequently become infected, prophylactic antibiotics are usually prescribed to prevent infection. To minimize infection, deep bite wounds on the extremities are left open. Rabies immune globulin might be used after an animal bite. Initial treatment of bite wounds includes copious irrigation to help clean out contaminants and microorganisms. DIF: Cognitive Level: Apply (application)

When administering a mental status examination to a patient with delirium, the nurse should a. wait until the patient is well-rested. b. administer an anxiolytic medication. c. choose a place without distracting stimuli. d. reorient the patient during the examination.

ANS: C Because overstimulation by environmental factors can distract the patient from the task of answering the nurse's questions, these stimuli should be avoided. The nurse will not wait to give the examination because action to correct the delirium should occur as soon as possible. Reorienting the patient is not appropriate during the examination. Antianxiety medications may increase the patient's delirium. DIF: Cognitive Level: Apply (application) REF: 1416 TOP: Nursing Process: Implementation MSC: NCLEX: Psychosocial Integrity

A 22-yr-old patient who experienced a drowning accident in a local pool, but now is awake and breathing spontaneously, is admitted for observation. Which assessment will be most important for the nurse to take during the observation period? a. Auscultate heart sounds. c. Auscultate breath sounds. b. Palpate peripheral pulses. d. Check mental orientation.

ANS: C Because pulmonary edema is a common complication after drowning, the nurse should assess the breath sounds frequently. The other information also will be obtained by the nurse, but it is not as pertinent to the patient's admission diagnosis. DIF: Cognitive Level: Analyze (analysis)

Which assessment is most important for the nurse to make regarding a patient with myasthenia gravis? a. Pupil size c. Respiratory effort b. Grip strength d. Level of consciousness

ANS: C Because respiratory insufficiency may be life threatening, it will be most important to monitor respiratory function. The other data also will be assessed but are not as critical. DIF: Cognitive Level: Analyze (analysis) REF: 1393 OBJ: Special Questions: Prioritization TOP: Nursing Process: Assessment MSC: NCLEX: Physiological Integrity

A patient who had a C7 spinal cord injury 1 week ago has a weak cough effort and crackles. The initial intervention by the nurse should be to a. suction the patient's nasopharynx. b. notify the patient's health care provider. c. push upward on the epigastric area as the patient coughs. d. encourage incentive spirometry every 2 hours during the day.

ANS: C Because the cough effort is poor, the initial action should be to use assisted coughing techniques to improve the patient's ability to mobilize secretions. The use of the spirometer may improve respiratory status, but the patient's ability to take deep breaths is limited by the loss of intercostal muscle function. Suctioning may be needed if the patient is unable to expel secretions by coughing but should not be the nurse's first action. The health care provider should be notified if airway clearance interventions are not effective or additional collaborative interventions are needed. DIF: Cognitive Level: Analyze (analysis)

While the nurse is transporting a patient on a stretcher to the radiology department, the patient begins having a tonic-clonic seizure. Which action should the nurse take? a. Insert an oral airway during the seizure to maintain a patent airway. b. Restrain the patient's arms and legs to prevent injury during the seizure. c. Time and observe and record the details of the seizure and postictal state. d. Avoid touching the patient to prevent further nervous system stimulation.

ANS: C Because the diagnosis and treatment of seizures frequently are based on the description of the seizure, recording the length and details of the seizure is important. Insertion of an oral airway and restraining the patient during the seizure are contraindicated. The nurse may need to move the patient to decrease the risk of injury during the seizure. DIF: Cognitive Level: Apply (application) REF: 1381 TOP: Nursing Process: Implementation MSC: NCLEX: Physiological Integrity

A left-handed patient with left-sided hemiplegia has difficulty feeding himself. Which intervention should the nurse include in the plan of care? a. Provide a wide variety of food choices. b. Provide oral care before and after meals. c. Assist the patient to eat with the right hand. d. Teach the patient the "chin-tuck" technique.

ANS: C Because the patient has difficulty feeding himself, the appropriate interventions will focus on teaching the patient to use the right hand for self-feeding. The other interventions are appropriate for patients with other etiologies for the imbalanced nutrition. DIF: Cognitive Level: Apply (application) REF: 1361 TOP: Nursing Process: Planning MSC: NCLEX: Physiological Integrity

The nurse notes thick, white secretions in the endotracheal tube (ET) of a patient who is receiving mechanical ventilation. Which intervention will most directly treat this finding? a. Reposition the patient every 1 to 2 hours. b. Increase suctioning frequency to every hour. c. Add additional water to the patient's enteral feedings. d. Instill 5 mL of sterile saline into the ET before suctioning.

ANS: C Because the patient's secretions are thick, better hydration is indicated. Suctioning every hour without any specific evidence for the need will increase the incidence of mucosal trauma and would not address the etiology of the ineffective airway clearance. Instillation of saline does not liquefy secretions and may decrease the SpO2. Repositioning the patient is appropriate but will not decrease the thickness of secretions. DIF: Cognitive Level: Apply (application)

Which question from a nurse during a patient interview will provide focused information about a possible thyroid disorder? a. "What methods do you use to help cope with stress?" b. "Have you experienced any blurring or double vision?" c. "Have you had a recent unplanned weight gain or loss?" d. "Do you have to get up at night to empty your bladder?"

ANS: C Because thyroid function affects metabolic rate, changes in weight may indicate hyperfunction or hypofunction of the thyroid gland. Nocturia, visual difficulty, and changes in stress level are associated with other endocrine disorders. DIF: Cognitive Level: Apply (application) REF: 1109 TOP: Nursing Process: Assessment MSC: NCLEX: Physiological Integrity

A 46-yr-old female patient returns to the clinic with recurrent dysuria after being treated with trimethoprim and sulfamethoxazole for 3 days. Which action will the nurse plan to take? a. Remind the patient about the need to drink 1000 mL of fluids daily. b. Obtain a midstream urine specimen for culture and sensitivity testing. c. Suggest that the patient use acetaminophen (Tylenol) to relieve symptoms. d. Teach the patient to take the prescribed trimethoprim and sulfamethoxazole for 3 more days.

ANS: C Because uncomplicated urinary tract infections (UTIs) are usually successfully treated with 3 days of antibiotic therapy, this patient will need a urine culture and sensitivity to determine appropriate antibiotic therapy. Acetaminophen would not be as effective as other over-the-counter medications such as phenazopyridine in treating dysuria. The fluid intake should be increased to at least 1800 mL/day. Because the UTI has persisted after treatment with trimethoprim and sulfamethoxazole , the patient is likely to need a different antibiotic.

When evaluating outcomes of a glycerol rhizotomy for a patient with trigeminal neuralgia, the nurse will a. assess if the patient is doing daily facial exercises. b. question if the patient is using an eye shield at night. c. ask the patient about social activities with family and friends. d. remind the patient to chew on the unaffected side of the mouth.

ANS: C Because withdrawal from social activities is a common manifestation of trigeminal neuralgia, asking about social activities will help in evaluating if the patient's symptoms have improved. Glycerol rhizotomy does not damage the corneal reflex or motor functions of the trigeminal nerve, so there is no need to use an eye shield, do facial exercises, or take precautions with chewing. DIF: Cognitive Level: Apply (application)

Which laboratory result is important to communicate to the health care provider for a patient who is taking methotrexate to treat rheumatoid arthritis (RA)? a. Rheumatoid factor is positive. b. Fasting blood glucose is 90 mg/dL. c. The white blood cell (WBC) count is 1500/μL. d. The erythrocyte sedimentation rate is elevated.

ANS: C Bone marrow suppression is a possible side effect of methotrexate, and the patient's low WBC count places the patient at high risk for infection. The elevated erythrocyte sedimentation rate and positive rheumatoid factor are expected in RA. The blood glucose is normal. DIF: Cognitive Level: Apply (application) REF: 1528 TOP: Nursing Process: Evaluation MSC: NCLEX: Physiological Integrity

The nurse will anticipate the need to teach a patient who has osteoarthritis (OA) about which medication? a. Prednisone b. Adalimumab (Humira) c. Capsaicin cream (Zostrix) d. Sulfasalazine (Azulfidine)

ANS: C Capsaicin cream blocks the transmission of pain impulses and is helpful for some patients in treating OA. The other medications would be used for patients with rheumatoid arthritis. DIF: Cognitive Level: Apply (application) REF: 1520 TOP: Nursing Process: Planning MSC: NCLEX: Physiological Integrity

A 33-yr-old patient with multiple sclerosis (MS) is to begin treatment with glatiramer acetate (Copaxone). Which information will the nurse include in patient teaching? a. Recommendation to drink at least 4 L of fluid daily b. Need to avoid driving or operating heavy machinery c. How to draw up and administer injections of the medication d. Use of contraceptive methods other than oral contraceptives

ANS: C Copaxone is administered by self-injection. Oral contraceptives are an appropriate choice for birth control. There is no need to avoid driving or drink large fluid volumes when taking glatiramer. DIF: Cognitive Level: Apply (application) REF: 1385 TOP: Nursing Process: Implementation MSC: NCLEX: Physiological Integrity

A patient will be scheduled in the outpatient clinic for blood cortisol testing. Which instruction will the nurse provide? a. "Avoid adding any salt to your foods for 24 hours before the test." b. "You will need to lie down for 30 minutes before the blood is drawn." c. "Come to the laboratory to have the blood drawn early in the morning." d. "Do not have anything to eat or drink before the blood test is obtained."

ANS: C Cortisol levels are usually drawn in the morning, when levels are highest. The other instructions would be given to patients who were having other endocrine testing. DIF: Cognitive Level: Apply (application) REF: 1117 TOP: Nursing Process: Implementation MSC: NCLEX: Physiological Integrity

Which information about a 60-yr-old patient with multiple sclerosis indicates that the nurse should consult with the health care provider before giving the prescribed dose of dalfampridine (Ampyra)? a. The patient walks a mile each day for exercise. b. The patient complains of pain with neck flexion. c. The patient has an increased serum creatinine level. d. The patient has the relapsing-remitting form of MS.

ANS: C Dalfampridine should not be given to patients with impaired renal function. The other information will not impact whether the dalfampridine should be administered. DIF: Cognitive Level: Apply (application) REF: 1386 TOP: Nursing Process: Assessment MSC: NCLEX: Physiological Integrity

Which finding in a patient with a spinal cord tumor requires an immediate report to the health care provider? a. Depression about the diagnosis b. Anxiety about scheduled surgery c. Decreased ability to move the legs d. Back pain that worsens with coughing

ANS: C Decreasing sensation and leg movement indicates spinal cord compression, an emergency that will require rapid action (such as surgery) to prevent paralysis. The other findings will also require nursing action but are not emergencies. DIF: Cognitive Level: Apply (application)

When the nurse brings medications to a patient with rheumatoid arthritis, the patient refuses the prescribed methotrexate. The patient tells the nurse, "My arthritis isn't that bad yet. The side effects of methotrexate are worse than the arthritis." The most appropriate response by the nurse is a. "You have the right to refuse to take the methotrexate." b. "Methotrexate is less expensive than some of the newer drugs." c. "It is important to start methotrexate early to decrease the extent of joint damage." d. "Methotrexate is effective and has fewer side effects than some of the other drugs."

ANS: C Disease-modifying antirheumatic drugs (DMARDs) are prescribed early to prevent the joint degeneration that occurs as soon as the first year with RA. The other statements are accurate, but the most important point for the patient to understand is that it is important to start DMARDs as quickly as possible. DIF: Cognitive Level: Analyze (analysis) REF: 1528 TOP: Nursing Process: Implementation MSC: NCLEX: Physiological Integrity

When caring for a patient with a new right-sided homonymous hemianopsia resulting from a stroke, which intervention should the nurse include in the plan of care? a. Apply an eye patch to the right eye. b. Approach the patient from the right side. c. Place needed objects on the patient's left side. d. Teach the patient that the left visual deficit will resolve.

ANS: C During the acute period, the nurse should place objects on the patient's unaffected side. Because there is a visual defect in the right half of each eye, an eye patch is not appropriate. The patient should be approached from the left side. The visual deficit may not resolve, although the patient can learn to compensate for the defect. DIF: Cognitive Level: Apply (application) REF: 1362 TOP: Nursing Process: Planning MSC: NCLEX: Physiological Integrity

A patient with increased intracranial pressure after a head injury has a ventriculostomy in place. Which action can the nurse delegate to unlicensed assistive personnel (UAP) who regularly work in the intensive care unit? a. Document intracranial pressure every hour. b. Turn and reposition the patient every 2 hours. c. Check capillary blood glucose level every 6 hours. d. Monitor cerebrospinal fluid color and volume hourly.

ANS: C Experienced UAP can obtain capillary blood glucose levels when they have been trained and evaluated in the skill. Monitoring and documentation of cerebrospinal fluid (CSF) color and intracranial pressure (ICP) require registered nurse (RN)-level education and scope of practice. Although repositioning patients is frequently delegated to UAP, repositioning a patient with a ventriculostomy is complex and should be supervised by the RN. DIF: Cognitive Level: Apply (application)

During the neurologic assessment, the patient is unable to respond verbally to the nurse but cooperates with the nurse's directions to move his hands and feet. The nurse will suspect a. cerebellar injury. b. a brainstem lesion. c. frontal lobe damage. d. a temporal lobe lesion.

ANS: C Expressive speech (ability to express the self in language) is controlled by Broca's area in the frontal lobe. The temporal lobe contains Wernicke's area, which is responsible for receptive speech (ability to understand language input). The cerebellum and brainstem do not affect higher cognitive functions such as speech. DIF: Cognitive Level: Apply (application) REF: 1296 TOP: Nursing Process: Assessment MSC: NCLEX: Physiological Integrity

To prevent autonomic hyperreflexia, which nursing action will the home health nurse include in the plan of care for a patient who has paraplegia at the T4 level ? a. Support selection of a high-protein diet. b. Discuss options for sexuality and fertility. c. Assist in planning a prescribed bowel program. d. Use quad coughing to strengthen cough efforts.

ANS: C Fecal impaction is a common stimulus for autonomic hyperreflexia. Dietary protein, coughing, and discussing sexuality and fertility should be included in the plan of care but will not reduce the risk for autonomic hyperreflexia. DIF: Cognitive Level: Apply (application)

The nurse will anticipate teaching a patient with nephrotic syndrome who develops flank pain about treatment with a. antibiotics. b. antifungals. c. anticoagulants. d. antihypertensives.

ANS: C Flank pain in a patient with nephrotic syndrome suggests a renal vein thrombosis and anticoagulation is needed. Antibiotics are used to treat a patient with flank pain caused by pyelonephritis. Fungal pyelonephritis is uncommon and is treated with antifungals. Antihypertensives are used if the patient has high blood pressure.

A patient is receiving IV antibiotics at home to treat chronic osteomyelitis of the left femur. The nurse identifies a need for additional teaching related to health maintenance when the nurse finds that the patient a. is frustrated with the length of treatment required. b. takes and records the oral temperature twice a day. c. is unable to plantar flex the foot on the affected side. d. uses crutches to avoid weight bearing on the affected leg.

ANS: C Foot drop is an indication that the foot is not being supported in a neutral position by a splint. Using crutches and monitoring the oral temperature are appropriate self-care activities. Frustration with the length of treatment is not an indicator of ineffective health maintenance of the osteomyelitis. DIF: Cognitive Level: Apply (application) REF: 1499 TOP: Nursing Process: Analysis MSC: NCLEX: Physiological Integrity

Which statement by a nurse to a patient newly diagnosed with type 2 diabetes is correct? a. Insulin is not used to control blood glucose in patients with type 2 diabetes. b. Complications of type 2 diabetes are less serious than those of type 1 diabetes. c. Changes in diet and exercise may control blood glucose levels in type 2 diabetes. d. Type 2 diabetes is usually diagnosed when the patient is admitted with a hyperglycemic coma.

ANS: C For some patients with type 2 diabetes, changes in lifestyle are sufficient to achieve blood glucose control. Insulin is frequently used for type 2 diabetes, complications are equally severe as for type 1 diabetes, and type 2 diabetes is usually diagnosed with routine laboratory testing or after a patient develops complications such as frequent yeast infections. DIF: Cognitive Level: Understand (comprehension) REF: 1134 TOP: Nursing Process: Implementation MSC: NCLEX: Physiological Integrity

Which medication information will the nurse identify as a potential risk to a patient's musculoskeletal system? a. The patient takes a daily multivitamin and calcium supplement. b. The patient takes hormone replacement therapy (HRT) to prevent "hot flashes." c. The patient has severe asthma requiring frequent therapy with oral corticosteroids. d. The patient has headaches treated with nonsteroidal antiinflammatory drugs (NSAIDs).

ANS: C Frequent or chronic corticosteroid use may lead to skeletal problems such as avascular necrosis and osteoporosis. The use of HRT and calcium supplements will help prevent osteoporosis. NSAID use does not increase the risk for musculoskeletal problems. DIF: Cognitive Level: Apply (application) REF: 1451 TOP: Nursing Process: Assessment MSC: NCLEX: Health Promotion and Maintenance

6. Which medication information will the nurse identify as a concern for a patient's musculoskeletal status? a. The patient takes a daily multivitamin and calcium supplement. b. The patient takes hormone therapy (HT) to prevent "hot flashes." c. The patient has severe asthma and requires frequent therapy with oral corticosteroids. d. The patient has migraine headaches treated with nonsteroidal antiinflammatory drugs (NSAIDs).

ANS: C Frequent or chronic corticosteroid use may lead to skeletal problems such as avascular necrosis and osteoporosis. The use of HT and calcium supplements will help prevent osteoporosis. NSAID use does not increase the risk for musculoskeletal problems.

A 37-yr-old female patient is hospitalized with acute kidney injury (AKI). Which information will be most useful to the nurse in evaluating improvement in kidney function? a. Urine volume b. Creatinine level c. Glomerular filtration rate (GFR) d. Blood urea nitrogen (BUN) level

ANS: C GFR is the preferred method for evaluating kidney function. BUN levels can fluctuate based on factors such as fluid volume status and protein intake. Urine output can be normal or high in patients with AKI and does not accurately reflect kidney function. Creatinine alone is not an accurate reflection of renal function. DIF: Cognitive Level: Analyze (analysis) REF: 1079 TOP: Nursing Process: Evaluation MSC: NCLEX: Physiological Integrity

Which action should the nurse take before administering gentamicin (Garamycin) to a patient with acute osteomyelitis? a. Ask the patient about any nausea. b. Obtain the patient's oral temperature. c. Review the patient's serum creatinine. d. Change the prescribed wet-to-dry dressing.

ANS: C Gentamicin is nephrotoxic and can cause renal failure as reflected in the patient's serum creatinine. Monitoring the patient's temperature before gentamicin administration is not necessary. Nausea is not a common side effect of IV gentamicin. There is no need to change the dressing before gentamicin administration. DIF: Cognitive Level: Apply (application) REF: 1498 TOP: Nursing Process: Assessment MSC: NCLEX: Physiological Integrity

When a patient who takes metformin (Glucophage) to manage type 2 diabetes develops an allergic rash from an unknown cause, the health care provider prescribes prednisone. The nurse will anticipate that the patient may a. need a diet higher in calories while receiving prednisone. b. develop acute hypoglycemia while taking the prednisone. c. require administration of insulin while taking prednisone. d. have rashes caused by metformin-prednisone interactions.

ANS: C Glucose levels increase when patients are taking corticosteroids, and insulin may be required to control blood glucose. Hypoglycemia is not a side effect of prednisone. Rashes are not an adverse effect caused by taking metformin and prednisone simultaneously. The patient may have an increased appetite when taking prednisone but will not need a diet that is higher in calories. DIF: Cognitive Level: Apply (application) REF: 1124 TOP: Nursing Process: Planning MSC: NCLEX: Physiological Integrity

The nurse reviews a patient's glycosylated hemoglobin (A1C) results to evaluate a. fasting preprandial glucose levels. b. glucose levels 2 hours after a meal. c. glucose control over the past 90 days. d. hypoglycemic episodes in the past 3 months.

ANS: C Glycosylated hemoglobin testing measures glucose control over the last 3 months. Glucose testing before/after a meal or random testing may reveal impaired glucose tolerance and indicate prediabetes, but it is not done on patients who already have a diagnosis of diabetes. There is no test to evaluate for hypoglycemic episodes in the past. DIF: Cognitive Level: Understand (comprehension) REF: 1118 TOP: Nursing Process: Evaluation MSC: NCLEX: Physiological Integrity

A 55-yr-old patient with end-stage kidney disease (ESKD) is scheduled to receive a prescribed dose of epoetin alfa (Procrit). Which information should the nurse report to the health care provider before giving the medication? a. Creatinine 1.6 mg/dL b. Oxygen saturation 89% c. Hemoglobin level 13 g/dL d. Blood pressure 98/56 mm Hg

ANS: C High hemoglobin levels are associated with a higher rate of thromboembolic events and increased risk of death from serious cardiovascular events (heart attack, heart failure, stroke) when erythropoietin (EPO) is administered to a target hemoglobin of greater than 12 g/dL. Hemoglobin levels higher than 12 g/dL indicate a need for a decrease in epoetin alfa dose. The other information also will be reported to the health care provider but will not affect whether the medication is administered. DIF: Cognitive Level: Apply (application) REF: 1081 TOP: Nursing Process: Assessment MSC: NCLEX: Physiological Integrity

Which stroke risk factor for a 48-yr-old male patient in the clinic is most important for the nurse to address? a. The patient is 25 lb above the ideal weight. b. The patient drinks a glass of red wine with dinner daily. c. The patient's usual blood pressure (BP) is 170/94 mm Hg. d. The patient works at a desk and relaxes by watching television.

ANS: C Hypertension is the single most important modifiable risk factor. People who drink more than 1 (for women) or 2 (for men) alcoholic beverages a day may increase their risk for hypertension. Physical inactivity and obesity contribute to stroke risk but not as much as hypertension. DIF: Cognitive Level: Analyze (analysis) REF: 1347 OBJ: Special Questions: Prioritization TOP: Nursing Process: Assessment MSC: NCLEX: Health Promotion and Maintenance

Before administration of calcium carbonate to a patient with chronic kidney disease (CKD), the nurse should check laboratory results for a. potassium level. b. total cholesterol. c. serum phosphate. d. serum creatinine.

ANS: C If serum phosphate is elevated, the calcium and phosphate can cause soft tissue calcification. Calcium carbonate should not be given until the phosphate level is lowered. Total cholesterol, creatinine, and potassium values do not affect whether calcium carbonate should be administered. DIF: Cognitive Level: Apply (application) REF: 1081 TOP: Nursing Process: Implementation MSC: NCLEX: Physiological Integrity

The health care provider suspects the Somogyi effect in a 50-yr-old patient whose 6:00 AM blood glucose is 230 mg/dL. Which action will the nurse teach the patient to take? a. Avoid snacking at bedtime. b. Increase the rapid-acting insulin dose. c. Check the blood glucose during the night d. Administer a larger dose of long-acting insulin.

ANS: C If the Somogyi effect is causing the patient's increased morning glucose level, the patient will experience hypoglycemia between 2:00 and 4:00 AM. The dose of insulin will be reduced, rather than increased. A bedtime snack is used to prevent hypoglycemic episodes during the night. DIF: Cognitive Level: Apply (application) REF: 1129 TOP: Nursing Process: Planning MSC: NCLEX: Physiological Integrity

A patient passing bloody urine is scheduled for a cystoscopy with cystogram. Which description of the procedure by the nurse is accurate? a. "Your doctor will place a catheter into an artery in your groin and inject a dye to visualize the blood supply to the kidneys." b. "Your doctor will insert a lighted tube into the bladder, and little catheters will be inserted through the tube into your kidney." c. "Your doctor will insert a lighted tube in the bladder through your urethra, inspect the bladder, and instill dye that will outline your bladder on x-ray." d. "Your doctor will inject a radioactive solution into a vein in your arm, then the distribution of the isotope in your kidneys and bladder will be visible."

ANS: C In a cystoscope and cystogram procedure, a cystoscope is inserted into the bladder for direct visualization, and then contrast solution is injected through the scope so that x-rays can be taken. The response beginning, "Your doctor will place a catheter" describes a renal arteriogram procedure. The response beginning, "Your doctor will inject a radioactive solution" describes a nuclear scan. The response beginning, "Your doctor will insert a lighted tube into the bladder, and little catheters will be inserted" describes a retrograde pyelogram. DIF: Cognitive Level: Apply (application) REF: 1028 TOP: Nursing Process: Implementation MSC: NCLEX: Physiological Integrity

When administering the Mini-Cog exam to a patient with possible Alzheimer's disease, which action will the nurse take? a. Check the patient's orientation to time and date. b. Obtain a list of the patient's prescribed medications. c. Ask the person to use a clock drawing to indicate a specific time. d. Determine the patient's ability to recognize a common object such as a pen.

ANS: C In the Mini-Cog, patients illustrate a specific time stated by the examiner by drawing the time on a clock face. The other actions may be included in assessment for Alzheimer's disease but are not part of the Mini-Cog exam. DIF: Cognitive Level: Understand (comprehension) REF: 1408 TOP: Nursing Process: Assessment MSC: NCLEX: Physiological Integrity

The nurse's initial action for a patient with moderate dementia who develops increased restlessness and agitation should be to a. reorient the patient to time, place, and person. b. administer a PRN dose of lorazepam (Ativan). c. assess for factors that might be causing discomfort. d. assign unlicensed assistive personnel (UAP) to stay in the patient's room.

ANS: C Increased motor activity in a patient with dementia is frequently the patient's only way of responding to factors such as pain, so the nurse's initial action should be to assess the patient for any precipitating factors. Administration of sedative drugs may be indicated, but this should not be done until assessment for precipitating factors has been completed and any of these factors have been addressed. Reorientation is unlikely to be helpful for the patient with moderate dementia. Assigning UAP to stay with the patient may also be necessary, but any physical changes that may be causing the agitation should be addressed first. DIF: Cognitive Level: Analyze (analysis) REF: 1411 OBJ: Special Questions: Prioritization TOP: Nursing Process: Implementation MSC: NCLEX: Physiological Integrity

A 26-yr-old female with type 1 diabetes develops a sore throat and runny nose after caring for her sick toddler. The patient calls the clinic for advice about her symptoms and a blood glucose level of 210 mg/dL despite taking her usual glargine (Lantus) and lispro (Humalog) insulin. The nurse advises the patient to a. use only the lispro insulin until the symptoms are resolved. b. limit intake of calories until the glucose is less than 120 mg/dL. c. monitor blood glucose every 4 hours and notify the clinic if it continues to rise. d. decrease intake of carbohydrates until glycosylated hemoglobin is less than 7%.

ANS: C Infection and other stressors increase blood glucose levels and the patient will need to test blood glucose frequently, treat elevations appropriately with lispro insulin, and call the health care provider if glucose levels continue to be elevated. Discontinuing the glargine will contribute to hyperglycemia and may lead to diabetic ketoacidosis (DKA). Decreasing carbohydrate or caloric intake is not appropriate because the patient will need more calories when ill. Glycosylated hemoglobin testing is not used to evaluate short-term alterations in blood glucose. DIF: Cognitive Level: Apply (application) REF: 1139 TOP: Nursing Process: Implementation MSC: NCLEX: Physiological Integrity

Which action by a patient indicates that the home health nurse's teaching about glargine and regular insulin has been successful? a. The patient administers the glargine 30 minutes before each meal. b. The patient's family prefills the syringes with the mix of insulins weekly. c. The patient discards the open vials of glargine and regular insulin after 4 weeks. d. The patient draws up the regular insulin and then the glargine in the same syringe.

ANS: C Insulin can be stored at room temperature for 4 weeks. Glargine should not be mixed with other insulins or prefilled and stored. Short-acting regular insulin is administered before meals, and glargine is given once daily. DIF: Cognitive Level: Apply (application) REF: 1127 TOP: Nursing Process: Evaluation MSC: NCLEX: Physiological Integrity

When a brain-injured patient responds to nail bed pressure with internal rotation, adduction, and flexion of the arms, the nurse reports the response as a. flexion withdrawal. c. decorticate posturing. b. localization of pain. d. decerebrate posturing.

ANS: C Internal rotation, adduction, and flexion of the arms in an unconscious patient is documented as decorticate posturing. Extension of the arms and legs is decerebrate posturing. Because the flexion is generalized, it does not indicate localization of pain or flexion withdrawal. DIF: Cognitive Level: Understand (comprehension)

The nurse determines that teaching about management of migraine headaches has been effective when the patient says which of the following? a. "I can take the (Topamax) as soon as a headache starts." b. "A glass of wine might help me relax and prevent a headache." c. "I will lie down someplace dark and quiet when the headaches begin." d. "I should avoid taking aspirin and sumatriptan (Imitrex) at the same time."

ANS: C It is recommended that the patient with a migraine rest in a dark, quiet area. Topiramate (Topamax) is used to prevent migraines and must be taken for several months to determine effectiveness. Aspirin or other nonsteroidal antiinflammatory medications can be taken with the triptans. Alcohol may precipitate migraine headaches. DIF: Cognitive Level: Apply (application) REF: 1373 TOP: Nursing Process: Evaluation MSC: NCLEX: Physiological Integrity

When a patient with type 2 diabetes is admitted for a cholecystectomy, which nursing action can the nurse delegate to a licensed practical/vocational nurse (LPN/LVN)? a. Communicate the blood glucose level and insulin dose to the circulating nurse in surgery. b. Discuss the reason for the use of insulin therapy during the immediate postoperative period. c. Administer the prescribed lispro (Humalog) insulin before transporting the patient to surgery. d. Plan strategies to minimize the risk for hypoglycemia or hyperglycemia during the postoperative period.

ANS: C LPN/LVN education and scope of practice includes administration of insulin. Communication about patient status with other departments, planning, and patient teaching are skills that require RN education and scope of practice. DIF: Cognitive Level: Apply (application) REF: 1152 OBJ: Special Questions: Delegation TOP: Nursing Process: Planning MSC: NCLEX: Safe and Effective Care Environment

The family members of a patient who has been admitted to the intensive care unit (ICU) with multiple traumatic injuries have just arrived in the ICU waiting room. Which action should the nurse take first? a. Explain ICU visitation policies and encourage family visits. b. Escort the family from the waiting room to the patient's bedside. c. Describe the patient's injuries and the care that is being provided. d. Invite the family to participate in an interprofessional care conference.

ANS: C Lack of information is a major source of anxiety for family members and should be addressed first. Family members should be prepared for the patient's appearance and the ICU environment before visiting the patient for the first time. ICU visiting should be individualized to each patient and family rather than being dictated by rigid visitation policies. Inviting the family to participate in a multidisciplinary conference is appropriate but should not be the initial action by the nurse. DIF: Cognitive Level: Analyze (analysis)

A patient is diagnosed with moderate dementia after multiple strokes. During assessment of the patient, the nurse would expect to find a. excessive nighttime sleepiness. b. difficulty eating and swallowing. c. loss of recent and long-term memory. d. fluctuating ability to perform simple tasks.

ANS: C Loss of both recent and long-term memory is characteristic of moderate dementia. Patients with dementia have frequent nighttime awakening. Dementia is progressive, and the patient's ability to perform tasks would not have periods of improvement. Difficulty eating and swallowing is characteristic of severe dementia. DIF: Cognitive Level: Understand (comprehension) REF: 1401 TOP: Nursing Process: Assessment MSC: NCLEX: Psychosocial Integrity

The nurse in the dialysis clinic is reviewing the home medications of a patient with chronic kidney disease (CKD). Which medication reported by the patient indicates that patient teaching is required? a. Acetaminophen b. Calcium phosphate c. Magnesium hydroxide d. Multivitamin with iron

ANS: C Magnesium is excreted by the kidneys, and patients with CKD should not use over-the-counter products containing magnesium. The other medications are appropriate for a patient with CKD. DIF: Cognitive Level: Apply (application) REF: 1081 TOP: Nursing Process: Assessment MSC: NCLEX: Physiological Integrity

The nurse has administered prescribed IV mannitol (Osmitrol) to an unconscious patient. Which parameter should the nurse monitor to determine the medication's effectiveness? a. Blood pressure c. Intracranial pressure b. Oxygen saturation d. Hemoglobin and hematocrit

ANS: C Mannitol is an osmotic diuretic and will reduce cerebral edema and intracranial pressure. It may initially reduce hematocrit and increase blood pressure, but these are not the best parameters for evaluation of the effectiveness of the drug. O2 saturation will not directly improve as a result of mannitol administration. DIF: Cognitive Level: Apply (application)

Which information about a patient who has a new prescription for phenytoin (Dilantin) indicates that the nurse should consult with the health care provider before administration of the medication? a. Patient has tonic-clonic seizures. b. Patient experiences an aura before seizures. c. Patient has minor elevations in the liver function tests. d. Patient's most recent blood pressure is 156/92 mm Hg.

ANS: C Many older patients (especially with compromised liver function) may not be able to metabolize phenytoin. The health care provider may need to choose another antiseizure medication. Phenytoin is an appropriate medication for patients with tonic-clonic seizures, with or without an aura. Hypertension is not a contraindication for phenytoin therapy. DIF: Cognitive Level: Apply (application) REF: 1379 TOP: Nursing Process: Implementation MSC: NCLEX: Physiological Integrity

Which information will the nurse teach a patient who has been newly diagnosed with Graves' disease? a. Exercise is contraindicated to avoid increasing metabolic rate. b. Restriction of iodine intake is needed to reduce thyroid activity. c. Antithyroid medications may take several months for full effect. d. Surgery will eventually be required to remove the thyroid gland.

ANS: C Medications used to block the synthesis of thyroid hormones may take 2 to 3 months before the full effect is seen. Large doses of iodine are used to inhibit the synthesis of thyroid hormones. Exercise using large muscle groups is encouraged to decrease the irritability and hyperactivity associated with high levels of thyroid hormones. Radioactive iodine is the most common treatment for Graves' disease, although surgery may be used. DIF: Cognitive Level: Apply (application) REF: 1165 TOP: Nursing Process: Implementation MSC: NCLEX: Physiological Integrity

A patient hospitalized with a fever and red, hot, painful knees is suspected of having septic arthritis. Information obtained during the nursing history that indicates a risk factor for septic arthritis is that the patient a. had several knee injuries as a teenager. b. recently returned from South America. c. is sexually active with multiple partners. d. has a parent who has rheumatoid arthritis.

ANS: C Neisseria gonorrhoeae is the most common cause for septic arthritis in sexually active young adults. The other information does not point to any risk for septic arthritis. DIF: Cognitive Level: Understand (comprehension) REF: 1535 TOP: Nursing Process: Assessment MSC: NCLEX: Physiological Integrity

An older patient with cardiogenic shock is cool and clammy. Hemodynamic monitoring indicates a high systemic vascular resistance (SVR). Which intervention should the nurse anticipate? a. Increase the rate for the dopamine infusion. b. Decrease the rate for the nitroglycerin infusion. c. Increase the rate for the sodium nitroprusside infusion. d. Decrease the rate for the 5% dextrose in normal saline (D5/.9 NS) infusion.

ANS: C Nitroprusside is an arterial vasodilator and will decrease the SVR and afterload, which will improve cardiac output. Changes in the D5/.9 NS and nitroglycerin infusions will not directly decrease SVR. Increasing the dopamine will tend to increase SVR. DIF: Cognitive Level: Apply (application)

An expected patient problem for a patient admitted to the hospital with symptoms of diabetes insipidus is a. excess fluid volume related to intake greater than output. b. impaired gas exchange related to fluid retention in lungs. c. sleep pattern disturbance related to frequent waking to void. d. risk for impaired skin integrity related to generalized edema.

ANS: C Nocturia occurs as a result of the polyuria caused by diabetes insipidus. Edema, excess fluid volume, and fluid retention are not expected. DIF: Cognitive Level: Apply (application) REF: 1161 TOP: Nursing Process: Planning MSC: NCLEX: Physiological Integrity

When a patient's urine dipstick test indicates a small amount of protein, the nurse's next action should be to a. send a urine specimen to the laboratory to test for ketones. b. obtain a clean-catch urine for culture and sensitivity testing. c. inquire about which medications the patient is currently taking. d. ask the patient about any family history of chronic renal failure.

ANS: C Normally the urinalysis will show zero to trace amounts of protein, but some medications may give false-positive readings. The other actions by the nurse may be appropriate, but checking for medications that may affect the dipstick accuracy should be done first. DIF: Cognitive Level: Analyze (analysis) REF: 1026 TOP: Nursing Process: Assessment MSC: NCLEX: Physiological Integrity

Which patient seen by the nurse in the outpatient clinic is most likely to require teaching about ways to reduce the risk for osteoarthritis (OA)? a. A 56-yr-old man who has a sedentary office job b. A 38-yr-old man who plays on a summer softball team c. A 56-yr-old woman who works on an automotive assembly line d. A 38-yr-old woman who is newly diagnosed with diabetes mellitus

ANS: C OA is more likely to occur in women as a result of estrogen reduction at menopause and in individuals whose work involves repetitive movements and lifting. Moderate exercise, such as softball, reduces the risk for OA. Diabetes is not a risk factor for OA. Sedentary work is not a risk factor for OA. DIF: Cognitive Level: Apply (application) REF: 1518 OBJ: Special Questions: Prioritization TOP: Nursing Process: Planning MSC: NCLEX: Physiological Integrity

A patient with hypotension and an elevated temperature after working outside on a hot day is treated in the emergency department (ED). The nurse determines that discharge teaching has been effective when the patient makes which statement? a. "I'll take salt tablets when I work outdoors in the summer." b. "I should take acetaminophen (Tylenol) if I start to feel too warm." c. "I need to drink extra fluids when working outside in hot weather." d. "I'll move to a cool environment if I notice that I'm feeling confused"

ANS: C Oral fluids and electrolyte replacement solutions such as sports drinks help replace fluid and electrolytes lost when exercising in hot weather. Salt tablets are not recommended because of the risks of gastric irritation and hypernatremia. Antipyretic drugs are not effective in lowering body temperature elevations caused by excessive exposure to heat. A patient who is confused is likely to have more severe hyperthermia and will be unable to remember to take appropriate action. DIF: Cognitive Level: Apply (application)

A licensed practical/vocational nurse (LPN/LVN) is caring for a patient with stage 2 chronic kidney disease. Which observation by the RN requires an intervention? a. The LPN/LVN administers the erythropoietin subcutaneously. b. The LPN/LVN assists the patient to ambulate out in the hallway. c. The LPN/LVN administers the iron supplement and phosphate binder with lunch. d. The LPN/LVN carries a tray containing low-protein foods into the patient's room.

ANS: C Oral phosphate binders should not be given at the same time as iron because they prevent the iron from being absorbed. The phosphate binder should be given with a meal and the iron given at a different time. The other actions by the LPN/LVN are appropriate for a patient with renal insufficiency. DIF: Cognitive Level: Apply (application) REF: 1082 OBJ: Special Questions: Delegation TOP: Nursing Process: Implementation MSC: NCLEX: Safe and Effective Care Environment

The nurse instructs a patient who has osteosarcoma of the tibia about a scheduled above-the-knee amputation. Which statement by a patient indicates additional patient teaching is needed? a. "I will need to participate in physical therapy after surgery." b. "I wish I did not need to have chemotherapy after this surgery." c. "I did not have this bone cancer until my leg broke a week ago." d. "I can use the patient-controlled analgesia (PCA) to manage postoperative pain."

ANS: C Osteogenic sarcoma may be diagnosed following a fracture, but it is not caused by the injury. The other statements indicate patient teaching has been effective. DIF: Cognitive Level: Apply (application) REF: 1501 TOP: Nursing Process: Evaluation MSC: NCLEX: Physiological Integrity

When caring for a patient with pulmonary hypertension, which parameter will the nurse use to directly evaluate the effectiveness of the treatment? a. Central venous pressure (CVP) b. Systemic vascular resistance (SVR) c. Pulmonary vascular resistance (PVR) d. Pulmonary artery wedge pressure (PAWP)

ANS: C PVR is a major contributor to pulmonary hypertension, and a decrease would indicate that pulmonary hypertension was improving. The other parameters may also be monitored but do not directly assess for pulmonary hypertension. DIF: Cognitive Level: Apply (application)

Which action by a patient who is using peritoneal dialysis (PD) indicates that the nurse should provide more teaching about PD? a. The patient leaves the catheter exit site without a dressing. b. The patient plans 30 to 60 minutes for a dialysate exchange. c. The patient cleans the catheter while taking a bath each day. d. The patient slows the inflow rate when experiencing abdominal pain.

ANS: C Patients are encouraged to take showers rather than baths to avoid infections at the catheter insertion side. The other patient actions indicate good understanding of peritoneal dialysis. DIF: Cognitive Level: Apply (application) REF: 1086 TOP: Nursing Process: Evaluation MSC: NCLEX: Physiological Integrity

A patient with diabetes rides a bicycle to and from work every day. Which site should the nurse teach the patient to use to administer the morning insulin? a. thigh. b. buttock. c. abdomen. d. upper arm.

ANS: C Patients should be taught not to administer insulin into a site that will be exercised because exercise will increase the rate of absorption. The thigh, buttock, and arm are all exercised by riding a bicycle. DIF: Cognitive Level: Apply (application) REF: 1128 TOP: Nursing Process: Implementation MSC: NCLEX: Physiological Integrity

A patient with diabetes rides a bicycle to and from work every day. Which site should the nurse teach the patient to use to administer the morning insulin? a. thigh. . b. buttock. c. abdomen d. upper arm.

ANS: C Patients should be taught not to administer insulin into a site that will be exercised because exercise will increase the rate of absorption. The thigh, buttock, and arm are all exercised by riding a bicycle. DIF: Cognitive Level: Apply (application) REF: 1128 TOP: Nursing Process: Implementation MSC: NCLEX: Physiological Integrity

When the nurse educator is evaluating the skills of a new registered nurse (RN) caring for patients experiencing shock, which action by the new RN indicates a need for more education? a. Placing the pulse oximeter on the ear for a patient with septic shock b. Keeping the head of the bed flat for a patient with hypovolemic shock c. Maintaining a cool room temperature for a patient with neurogenic shock d. Increasing the nitroprusside infusion rate for a patient with a very high SVR

ANS: C Patients with neurogenic shock have poikilothermia. The room temperature should be kept warm to avoid hypothermia. The other actions by the new RN are appropriate. DIF: Cognitive Level: Apply (application)

The nurse will plan to teach a patient to minimize physical and emotional stress while the patient is undergoing a. a water deprivation test. b. testing for serum T3 and T4 levels. c. a 24-hour urine test for free cortisol. d. a radioactive iodine (I-131) uptake test.

ANS: C Physical and emotional stress can affect the results of the free cortisol test. The other tests are not impacted by stress. DIF: Cognitive Level: Apply (application) REF: 1117 TOP: Nursing Process: Planning MSC: NCLEX: Physiological Integrity

Which menu choice by the patient who is receiving hemodialysis indicates that the nurse's teaching has been successful? a. Split-pea soup, English muffin, and nonfat milk b. Oatmeal with cream, half a banana, and herbal tea c. Poached eggs, whole-wheat toast, and apple juice d. Cheese sandwich, tomato soup, and cranberry juice

ANS: C Poached eggs would provide high-quality protein, and apple juice is low in potassium. Cheese is high in salt and phosphate, and tomato soup is high in potassium. Split-pea soup is high in potassium, and dairy products are high in phosphate. Bananas are high in potassium, and cream is high in phosphate. DIF: Cognitive Level: Apply (application) REF: 1087 TOP: Nursing Process: Evaluation MSC: NCLEX: Physiological Integrity

To evaluate the effectiveness of the pantoprazole (Protonix) ordered for a patient with systemic inflammatory response syndrome (SIRS), which assessment will the nurse perform? a. Auscultate bowel sounds. c. Check stools for occult blood. b. Ask the patient about nausea. d. Palpate for abdominal tenderness.

ANS: C Proton pump inhibitors are given to decrease the risk for stress ulcers in critically ill patients. The other assessments will also be done, but these will not help in determining the effectiveness of the pantoprazole administration. DIF: Cognitive Level: Apply (application)

A patient is being admitted with a diagnosis of Cushing syndrome. Which findings will the nurse expect during the assessment? a. Chronically low blood pressure c. Purplish streaks on the abdomen b. Bronzed appearance of the skin d. Decreased axillary and pubic hair

ANS: C Purplish-red striae on the abdomen are a common clinical manifestation of Cushing syndrome. Hypotension and bronzed-appearing skin are manifestations of Addison's disease. Decreased axillary and pubic hair occur with androgen deficiency. DIF: Cognitive Level: Understand (comprehension) REF: 1175 TOP: Nursing Process: Assessment MSC: NCLEX: Physiological Integrity

A patient with left-sided weakness that started 60 minutes earlier is admitted to the emergency department and diagnostic tests are ordered. Which test should be done first? a. Complete blood count (CBC) b. Chest radiograph (chest x-ray) c. Computed tomography (CT) scan d. 12-Lead electrocardiogram (ECG)

ANS: C Rapid screening with a noncontrast CT scan is needed before administration of tissue plasminogen activator (tPA), which must be given within 4.5 hours of the onset of clinical manifestations of the stroke. The sooner the tPA is given, the less brain injury. The other diagnostic tests give information about possible causes of the stroke and do not need to be completed as urgently as the CT scan. DIF: Cognitive Level: Analyze (analysis) REF: 1354 OBJ: Special Questions: Prioritization TOP: Nursing Process: Implementation MSC: NCLEX: Physiological Integrity

Which nursing action included in the care of a patient after laminectomy can the nurse delegate to experienced unlicensed assistive personnel (UAP)? a. Check ability to plantar and dorsiflex the foot. b. Determine the patient's readiness to ambulate. c. Log roll the patient from side to side every 2 hours. d. Ask about pain management with the patient-controlled analgesia (PCA).

ANS: C Repositioning a patient is included in the education and scope of practice of UAP, and experienced UAP will be familiar with how to maintain alignment in the postoperative patient. Evaluation of the effectiveness of pain medications, assessment of neurologic function, and evaluation of a patient's readiness to ambulate after surgery require higher level nursing education and scope of practice. DIF: Cognitive Level: Apply (application) REF: 1498 OBJ: Special Questions: Delegation TOP: Nursing Process: Planning MSC: NCLEX: Safe and Effective Care Environment

An appropriate nursing intervention for a patient who has acute low back pain and muscle spasms is to teach the patient to a. keep both feet flat on the floor when prolonged standing is required. b. twist gently from side to side to maintain range of motion in the spine. c. keep the head elevated slightly and flex the knees when resting in bed. d. avoid the use of cold packs because they will exacerbate the muscle spasms.

ANS: C Resting with the head elevated and knees flexed will reduce the strain on the back and decrease muscle spasms. Twisting from side to side will increase tension on the lumbar area. Prolonged standing will cause strain on the lumbar spine, even with both feet flat on the floor. Alternate application of cold and heat should be used to decrease pain. DIF: Cognitive Level: Apply (application) REF: 1503 TOP: Nursing Process: Planning MSC: NCLEX: Physiological Integrity

A patient being seen in the clinic has rheumatoid nodules on the elbows. Which action will the nurse take? a. Draw blood for rheumatoid factor analysis. b. Teach the patient about injections for the nodules. c. Assess the nodules for skin breakdown or infection. d. Discuss the need for surgical removal of the nodules.

ANS: C Rheumatoid nodules can break down or become infected. They are not associated with changes in rheumatoid factor, and injection is not needed. Rheumatoid nodules are usually not removed surgically because of a high probability of recurrence. DIF: Cognitive Level: Apply (application) REF: 1527 TOP: Nursing Process: Implementation MSC: NCLEX: Physiological Integrity

Which intervention will the nurse include in the plan of care for a patient with syndrome of inappropriate antidiuretic hormone (SIADH)? a. Encourage fluids to 2 to 3 L/day. b. Monitor for increasing peripheral edema. c. Offer the patient hard candies to suck on. d. Keep head of bed elevated to 30 degrees.

ANS: C Sucking on hard candies decreases thirst for a patient on fluid restriction. Patients with SIADH are on fluid restrictions of 800 to 1000 mL/day. Peripheral edema is not seen with SIADH. The head of the bed is elevated no more than 10 degrees to increase left atrial filling pressure and decrease antidiuretic hormone (ADH) release. DIF: Cognitive Level: Apply (application) REF: 1161 TOP: Nursing Process: Planning MSC: NCLEX: Physiological Integrity

The charge nurse observes an inexperienced staff nurse caring for a patient who has had a craniotomy for resection of a brain tumor. Which action by the inexperienced nurse requires the charge nurse to intervene? a. The staff nurse assesses neurologic status every hour. b. The staff nurse elevates the head of the bed to 30 degrees. c. The staff nurse suctions the patient routinely every 2 hours. d. The staff nurse administers an analgesic before turning the patient.

ANS: C Suctioning increases intracranial pressure and should only be done when the patient's respiratory condition indicates it is needed. The other actions by the staff nurse are appropriate. DIF: Cognitive Level: Apply (application)

Which information obtained by the nurse in the endocrine clinic about a patient who has been taking prednisone 40 mg daily for 3 weeks is most important to report to the health care provider? a. Patient's blood pressure is 148/94 mm Hg. b. Patient has bilateral 2+ pitting ankle edema. c. Patient stopped taking the medication 2 days ago. d. Patient has not been taking the prescribed vitamin D.

ANS: C Sudden cessation of corticosteroids after taking the medication for a week or more can lead to adrenal insufficiency, with problems such as severe hypotension and hypoglycemia. The patient will need immediate evaluation by the health care provider to prevent or treat adrenal insufficiency. The other information will also be reported but does not require rapid treatment. DIF: Cognitive Level: Analyze (analysis) REF: 1176 OBJ: Special Questions: Prioritization TOP: Nursing Process: Assessment MSC: NCLEX: Physiological Integrity

A patient had a cystectomy with an ileal conduit yesterday. Which new assessment data is most important for the nurse to communicate to the health care provider? a. Cloudy appearing urine b. Hypotonic bowel sounds c. Heart rate 102 beats/minute d. Continuous stoma drainage

ANS: C Tachycardia may indicate infection, hemorrhage, or hypovolemia, which are all serious complications of this surgery. The urine from an ileal conduit normally contains mucus and is cloudy. Hypotonic bowel sounds are expected after bowel surgery. Continuous drainage of urine from the stoma is normal.

A nurse is weaning a 68-kg patient who has chronic obstructive pulmonary disease (COPD) from mechanical ventilation. Which patient assessment finding indicates that the weaning protocol should be stopped? a. The patient's heart rate is 97 beats/min. b. The patient's oxygen saturation is 93%. c. The patient respiratory rate is 32 breaths/min. d. The patient's spontaneous tidal volume is 450 mL.

ANS: C Tachypnea is a sign that the patient's work of breathing is too high to allow weaning to proceed. The patient's heart rate is within normal limits, but the nurse should continue to monitor it. An O2 saturation of 93% is acceptable for a patient with COPD. A spontaneous tidal volume of 450 mL is within the acceptable range. DIF: Cognitive Level: Apply (application)

A patient is admitted with tetany. Which laboratory value should the nurse plan to monitor? a. Total protein c. Ionized calcium b. Blood glucose d. Serum phosphate

ANS: C Tetany is associated with hypocalcemia. The other values would not be useful for this patient. DIF: Cognitive Level: Apply (application) REF: 1113 TOP: Nursing Process: Planning MSC: NCLEX: Physiological Integrity

A patient admitted with dermal ulcers who has a history of a T3 spinal cord injury tells the nurse, "I have a pounding headache and I feel sick to my stomach." Which action should the nurse take first? a. Check for a fecal impaction. c. Assess the blood pressure (BP). b. Give the prescribed antiemetic. d. Notify the health care provider.

ANS: C The BP should be assessed immediately in a patient with an injury at the T6 level or higher who complains of a headache to determine if autonomic hyperreflexia is occurring. Notification of the patient's health care provider is appropriate after the BP is obtained. Administration of an antiemetic is indicated if autonomic hyperreflexia is ruled out as the cause of the nausea. After checking the BP, the nurse may assess for a fecal impaction using lidocaine jelly to prevent further increased BP. DIF: Cognitive Level: Analyze (analysis)

Which action will help the nurse determine whether a new patient's confusion is caused by dementia or delirium? a. Ask about a family history of dementia. b. Administer the Mini-Mental Status Exam. c. Use the Confusion Assessment Method tool. d. Obtain a list of the patient's usual medications.

ANS: C The Confusion Assessment Method tool has been extensively tested in assessing delirium. The other actions will be helpful in determining cognitive function or risk factors for dementia or delirium, but they will not be useful in differentiating between dementia and delirium. DIF: Cognitive Level: Apply (application) REF: 1415 TOP: Nursing Process: Assessment MSC: NCLEX: Psychosocial Integrity

Which information about a 30-yr-old patient who is hospitalized after a traumatic brain injury requires the most rapid action by the nurse? a. Intracranial pressure of 15 mm Hg b. Cerebrospinal fluid (CSF) drainage of 25 mL/hr c. Pressure of oxygen in brain tissue (PbtO2) is 14 mm Hg d. Cardiac monitor shows sinus tachycardia at 120 beats/minute

ANS: C The PbtO2 should be 20 to 40 mm Hg. Lower levels indicate brain ischemia. An intracranial pressure (ICP) of 15 mm Hg is at the upper limit of normal. CSF is produced at a rate of 20 to 30 mL/hr. The reason for the sinus tachycardia should be investigated, but the elevated heart rate is not as concerning as the decrease in PbtO2. DIF: Cognitive Level: Analyze (analysis)

A new clinic patient with joint swelling and pain is being tested for systemic lupus erythematosus. Which test will provide the most specific findings for the nurse to review? a. Rheumatoid factor (RF) b. Antinuclear antibody (ANA) c. Anti-Smith antibody (Anti-Sm) d. Lupus erythematosus (LE) cell prep

ANS: C The anti-Sm is antibody found almost exclusively in SLE. The other blood tests are also used in screening but are not as specific to SLE. DIF: Cognitive Level: Apply (application) REF: 1540 TOP: Nursing Process: Assessment MSC: NCLEX: Physiological Integrity

A 38-yr-old patient who has had a spinal cord injury returned home following a stay in a rehabilitation facility. The home care nurse notes the spouse is performing many of the activities that the patient had been managing unassisted during rehabilitation. The appropriate nursing action at this phase of rehabilitation is to a. remind the patient about the importance of independence in daily activities. b. tell the spouse to stop helping because the patient is able to perform activities independently. c. develop a plan to increase the patient's independence in consultation with the patient and the spouse. d. recognize that it is important for the spouse to be involved in the patient's care and encourage participation.

ANS: C The best action by the nurse will be to involve all parties in developing an optimal plan of care. Because family members who will be assisting with the patient's ongoing care need to believe their input is important, telling the spouse that the patient can perform activities independently is not the best choice. Reminding the patient about the importance of independence may not change the behaviors of the spouse. Supporting the activities of the spouse will lead to ongoing dependency by the patient. DIF: Cognitive Level: Apply (application)

A patient who has acute glomerulonephritis is hospitalized with hyperkalemia. Which information will the nurse monitor to evaluate the effectiveness of the prescribed calcium gluconate IV? a. Urine volume b. Calcium level c. Cardiac rhythm d. Neurologic status

ANS: C The calcium gluconate helps prevent dysrhythmias that might be caused by the hyperkalemia. The nurse will monitor the other data as well, but these will not be helpful in determining the effectiveness of the calcium gluconate. DIF: Cognitive Level: Apply (application) REF: 1073 TOP: Nursing Process: Evaluation MSC: NCLEX: Physiological Integrity

The nurse is caring for a patient who has a head injury and fractured right arm after being assaulted. Which assessment information requires rapid action by the nurse? a. The apical pulse is slightly irregular. b. The patient complains of a headache. c. The patient is more difficult to arouse. d. The blood pressure (BP) increases to 140/62 mm Hg.

ANS: C The change in level of consciousness (LOC) is an indicator of increased intracranial pressure (ICP) and suggests that action by the nurse is needed to prevent complications. The change in BP should be monitored but is not an indicator of a need for immediate nursing action. Headache and a slightly irregular apical pulse are not unusual in a patient after a head injury. DIF: Cognitive Level: Apply (application)

A patient who has neurogenic shock is receiving a phenylephrine infusion through a right forearm IV. Which assessment finding obtained by the nurse indicates a need for immediate action? a. The patient's heart rate is 58 beats/min. b. The patient's extremities are warm and dry. c. The patient's IV infusion site is cool and pale. d. The patient's urine output is 28 mL over the past hour.

ANS: C The coldness and pallor at the infusion site suggest extravasation of the phenylephrine. The nurse should discontinue the IV and, if possible, infuse the drug into a central line. An apical pulse of 58 beats/min is typical for neurogenic shock but does not indicate an immediate need for nursing intervention. A 28-mL urinary output over 1 hour would require the nurse to monitor the output over the next hour, but an immediate change in therapy is not indicated. Warm, dry skin is consistent with early neurogenic shock, but it does not indicate a need for a change in therapy or immediate action. DIF: Cognitive Level: Analyze (analysis)

A few weeks after an 82-yr-old patient with a new diagnosis of type 2 diabetes has been placed on metformin (Glucophage) therapy and taught about appropriate diet and exercise, the home health nurse makes a visit. Which finding should the nurse promptly discuss with the health care provider? a. Hemoglobin A1C level is 7.9%. b. Last eye examination was 18 months ago. c. Glomerular filtration rate is decreased. d. Patient has questions about the prescribed diet.

ANS: C The decrease in renal function may indicate a need to adjust the dose of metformin or change to a different medication. In older patients, the goal for A1C may be higher in order to avoid complications associated with hypoglycemia. The nurse will plan on scheduling the patient for an eye examination and addressing the questions about diet, but the area for prompt intervention is the patient's decreased renal function. DIF: Cognitive Level: Apply (application) REF: 1130 TOP: Nursing Process: Assessment MSC: NCLEX: Physiological Integrity

After change-of-shift report on a ventilator weaning unit, which patient should the nurse assess first? a. Patient who failed a spontaneous breathing trial and has been placed in a rest mode on the ventilator b. Patient who is intubated and has continuous partial pressure end-tidal CO2 (PETCO2) monitoring c. Patient who was successfully weaned and extubated 4 hours ago and has no urine output for the last 6 hours d. Patient with a central venous O2 saturation (ScvO2) of 69% while on bilevel positive airway pressure (BiPAP)

ANS: C The decreased urine output may indicate acute kidney injury or that the patient's cardiac output and perfusion of vital organs have decreased. Any of these causes would require rapid action. The data about the other patients indicate that their conditions are stable and do not require immediate assessment or changes in their care. Continuous PETCO2 monitoring is frequently used when patients are intubated. The rest mode should be used to allow patient recovery after a failed SBT, and an ScvO2 of 69% is within normal limits. DIF: Cognitive Level: Analyze (analysis)

A patient with type 2 diabetes is scheduled for a follow-up visit in the clinic several months from now. Which test will the nurse schedule to evaluate the effectiveness of treatment for the patient? a. Fasting blood glucose b. Oral glucose tolerance c. Glycosylated hemoglobin d. Urine dipstick for glucose

ANS: C The glycosylated hemoglobin (A1C) test shows the overall control of glucose over 90 to 120 days. A fasting blood level indicates only the glucose level at one time. Urine glucose testing is not an accurate reflection of blood glucose level and does not reflect the glucose over a prolonged time. Oral glucose tolerance testing is done to diagnose diabetes but is not used for monitoring glucose control after diabetes has been diagnosed. DIF: Cognitive Level: Apply (application) REF: 1124 TOP: Nursing Process: Planning MSC: NCLEX: Physiological Integrity

Which assessment finding obtained by the nurse when caring for a patient receiving mechanical ventilation indicates the need for suctioning? a. The patient was last suctioned 6 hours ago. b. The patient's oxygen saturation drops to 93%. c. The patient's respiratory rate is 32 breaths/min. d. The patient has occasional audible expiratory wheezes.

ANS: C The increase in respiratory rate indicates that the patient may have decreased airway clearance and requires suctioning. Suctioning is done when patient assessment data indicate that it is needed and not on a scheduled basis. Occasional expiratory wheezes do not indicate poor airway clearance, and suctioning the patient may induce bronchospasm and increase wheezing. An O2 saturation of 93% is acceptable and does not suggest that immediate suctioning is needed. DIF: Cognitive Level: Apply (application)

Which information from a patient's urinalysis requires that the nurse notify the health care provider? a. pH 6.2 c. WBC 20 to 26/hpf b. Trace protein d. Specific gravity 1.021

ANS: C The increased number of white blood cells (WBCs) indicates the presence of urinary tract infection or inflammation. The other findings are normal. DIF: Cognitive Level: Apply (application) REF: 1030 TOP: Nursing Process: Assessment MSC: NCLEX: Physiological Integrity

A male patient who has right-sided weakness after a stroke is making progress in learning to use the left hand for feeding and other activities. The nurse observes that when the patient's wife is visiting, she feeds and dresses him. Which nursing diagnosis is most appropriate for the patient? a. Interrupted family processes related to effects of illness of a family member b. Situational low self-esteem related to increasing dependence on spouse for care c. Disabled family coping related to inadequate understanding by patient's spouse d. Impaired nutrition: less than body requirements related to hemiplegia and aphasia

ANS: C The information supports the diagnosis of disabled family coping because the wife does not understand the rehabilitation program. There are no data supporting low self-esteem, and the patient is attempting independence. The data do not support an interruption in family processes because this may be a typical pattern for the couple. There is no indication that the patient has impaired nutrition. DIF: Cognitive Level: Apply (application) REF: 1362 TOP: Nursing Process: Diagnosis MSC: NCLEX: Psychosocial Integrity

A patient is admitted to the emergency department (ED) for shock of unknown etiology. The first action by the nurse should be to a. obtain the blood pressure. b. check the level of orientation. c. administer supplemental oxygen. d. obtain a 12-lead electrocardiogram.

ANS: C The initial actions of the nurse are focused on the ABCs—airway, breathing, and circulation—and administration of O2 should be done first. The other actions should be accomplished as rapidly as possible after providing O2. DIF: Cognitive Level: Analyze (analysis)

A 22-yr-old patient seen at the health clinic with a severe migraine headache tells the nurse about having similar headaches recently. Which initial action should the nurse take? a. Teach about the use of triptan drugs. b. Refer the patient for stress counseling. c. Ask the patient to keep a headache diary. d. Suggest the use of muscle-relaxation techniques.

ANS: C The initial nursing action should be further assessment of the precipitating causes of the headaches, quality, and location of pain. Stress reduction, muscle relaxation, and the triptan drugs may be helpful, but more assessment is needed first. DIF: Cognitive Level: Analyze (analysis) REF: 1373 OBJ: Special Questions: Prioritization TOP: Nursing Process: Implementation MSC: NCLEX: Physiological Integrity

After reviewing the information shown in the accompanying figure for a patient with pneumonia and sepsis, which information is most important to report to the health care provider? Physical Assessment Laboratory Data Vital Signs • Petechiae noted on chest and legs • Crackles heard bilaterally in lung bases • No redness or swelling at central line IV site • Blood urea nitrogen (BUN) 34 mg/Dl • Hematocrit 30% • Platelets 50,000/µL • Temperature 100°F (37.8°C) • Pulse 102/min • Respirations 26/min • BP 110/60 mm Hg • O2 saturation 93% on 2L O2 via nasal cannula a. Temperature and IV site appearance b. Oxygen saturation and breath sounds c. Platelet count and presence of petechiae d. Blood pressure, pulse rate, respiratory rate.

ANS: C The low platelet count and presence of petechiae suggest that the patient may have disseminated intravascular coagulation and that multiple organ dysfunction syndrome is developing. The other information will also be discussed with the health care provider but does not indicate that the patient's condition is deteriorating or that a change in therapy is needed immediately. DIF: Cognitive Level: Analyze (analysis)

Which action should the nurse take when the low pressure alarm sounds for a patient who has an arterial line in the left radial artery? a. Fast flush the arterial line. b. Check the left hand for pallor. c. Assess for cardiac dysrhythmias. d. Re-zero the monitoring equipment.

ANS: C The low pressure alarm indicates a drop in the patient's blood pressure, which may be caused by cardiac dysrhythmias. There is no indication to re-zero the equipment. Pallor of the left hand would be caused by occlusion of the radial artery by the arterial catheter, not by low pressure. There is no indication of a need for flushing the line. DIF: Cognitive Level: Apply (application)

A patient is hospitalized with new onset of Guillain-Barré syndrome. The most essential assessment for the nurse to complete is a. determining level of consciousness. b. checking strength of the extremities. c. observing respiratory rate and effort. d. monitoring the cardiac rate and rhythm.

ANS: C The most serious complication of Guillain-Barré syndrome is respiratory failure, and the nurse should monitor respiratory function continuously. The other assessments will also be included in nursing care, but they are not as important as respiratory assessment. DIF: Cognitive Level: Analyze (analysis)

An unconscious patient is admitted to the emergency department (ED) with a head injury. The patient's spouse and teenage children stay at the patient's side and ask many questions about the treatment being given. What action is best for the nurse to take? a. Call the family's pastor or spiritual advisor to take them to the chapel. b. Ask the family to stay in the waiting room until the assessment is completed. c. Allow the family to stay with the patient and briefly explain all procedures to them. d. Refer the family members to the hospital counseling service to deal with their anxiety.

ANS: C The need for information about the diagnosis and care is very high in family members of acutely ill patients. The nurse should allow the family to observe care and explain the procedures unless they interfere with emergent care needs. A pastor or counseling service can offer some support, but research supports information as being more effective. Asking the family to stay in the waiting room will increase their anxiety. DIF: Cognitive Level: Analyze (analysis)

The nurse notes that a patient's endotracheal tube (ET), which was at the 22-cm mark, is now at the 25-cm mark, and the patient is anxious and restless. Which action should the nurse take next? a. Check the O2 saturation. b. Offer reassurance to the patient. c. Listen to the patient's breath sounds. d. Notify the patient's health care provider.

ANS: C The nurse should first determine whether the ET tube has been displaced into the right mainstem bronchus by listening for unilateral breath sounds. If so, assistance will be needed to reposition the tube immediately. The other actions are also appropriate, but detection and correction of tube malposition are the most critical actions. DIF: Cognitive Level: Analyze (analysis)

A patient will attempt oral feedings for the first time after having a stroke. The nurse should assess the gag reflex and then a. order a varied pureed diet. c. assist the patient into a chair. b. assess the patient's appetite. d. offer the patient a sip of juice.

ANS: C The patient should be as upright as possible before attempting feeding to make swallowing easier and decrease aspiration risk. To assess swallowing ability, the nurse should initially offer water or ice to the patient. Pureed diets are not recommended because the texture is too smooth. The patient may have a poor appetite, but the oral feeding should be attempted. DIF: Cognitive Level: Apply (application) REF: 1360 TOP: Nursing Process: Implementation MSC: NCLEX: Physiological Integrity

A patient has an incomplete left spinal cord lesion at the level of T7, resulting in Brown-Séquard syndrome. Which nursing action should be included in the plan of care? a. Assessment of the patient for right arm weakness b. Assessment of the patient for increased right leg pain c. Positioning the patient's left leg when turning the patient d. Teaching the patient to look at the right leg to verify its position

ANS: C The patient with Brown-Séquard syndrome has loss of motor function on the ipsilateral side and will require the nurse to move the left leg. Pain sensation will be lost in the patient's right leg. Arm weakness will not be a problem for a patient with a T7 injury. The patient will retain position sense for the right leg. DIF: Cognitive Level: Apply (application)

Which statement by a patient with stage 5 chronic kidney disease (CKD) indicates that the nurse's teaching about management of CKD has been effective? a. "I need to get most of my protein from low-fat dairy products." b. "I will increase my intake of fruits and vegetables to 5 per day." c. "I will measure my urinary output each day to help calculate the amount I can drink." d. "I need to take erythropoietin to boost my immune system and help prevent infection."

ANS: C The patient with end-stage renal disease is taught to measure urine output as a means of determining an appropriate oral fluid intake. Erythropoietin is given to increase the red blood cell count and will not offer any benefit for immune function. Dairy products are restricted because of the high phosphate level. Many fruits and vegetables are high in potassium and should be restricted in the patient with CKD. DIF: Cognitive Level: Apply (application) REF: 1082 TOP: Nursing Process: Evaluation MSC: NCLEX: Physiological Integrity

A patient who is unconscious has ineffective cerebral tissue perfusion and cerebral tissue swelling. Which nursing intervention will be included in the plan of care? a. Encourage coughing and deep breathing. b. Position the patient with knees and hips flexed. c. Keep the head of the bed elevated to 30 degrees. d. Cluster nursing interventions to provide rest periods.

ANS: C The patient with increased intracranial pressure (ICP) should be maintained in the head-up position to help reduce ICP. Extreme flexion of the hips and knees increases abdominal pressure, which increases ICP. Because the stimulation associated with nursing interventions increases ICP, clustering interventions will progressively elevate ICP. Coughing increases intrathoracic pressure and ICP. DIF: Cognitive Level: Apply (application)

A patient who was admitted with diabetic ketoacidosis secondary to a urinary tract infection has been weaned off an insulin drip 30 minutes ago. The patient reports feeling lightheaded and sweaty. Which action should the nurse take first? a. Infuse dextrose 50% by slow IV push. b. Administer 1 mg glucagon subcutaneously. c. Obtain a glucose reading using a finger stick. d. Have the patient drink 4 ounces of orange juice.

ANS: C The patient's clinical manifestations are consistent with hypoglycemia, and the initial action should be to check the patient's glucose with a finger stick or order a stat blood glucose. If the glucose is low, the patient should ingest a rapid-acting carbohydrate, such as orange juice. Glucagon or dextrose 50% might be given if the patient's symptoms become worse or if the patient is unconscious. DIF: Cognitive Level: Analyze (analysis) REF: 1135 OBJ: Special Questions: Prioritization TOP: Nursing Process: Implementation MSC: NCLEX: Physiological Integrity

The nurse notices a circular lesion with a red border and clear center on the arm of a summer camp counselor who is in the clinic complaining of chills and muscle aches. Which action should the nurse take to follow up on that finding? a. Palpate the abdomen. b. Auscultate the heart sounds. c. Ask the patient about recent outdoor activities. d. Question the patient about immunization history.

ANS: C The patient's clinical manifestations suggest possible Lyme disease. A history of recent outdoor activities such as hikes will help confirm the diagnosis. The patient's symptoms do not suggest cardiac or abdominal problems or lack of immunization. DIF: Cognitive Level: Apply (application) REF: 1534 TOP: Nursing Process: Assessment MSC: NCLEX: Physiological Integrity

After the insertion of an arteriovenous graft (AVG) in the right forearm, a patient complains of pain and coldness of the right fingers. Which action should the nurse take? a. Teach the patient about normal AVG function. b. Remind the patient to take a daily low-dose aspirin tablet. c. Report the patient's symptoms to the health care provider. d. Elevate the patient's arm on pillows to above the heart level.

ANS: C The patient's complaints suggest the development of distal ischemia (steal syndrome) and may require revision of the AVG. Elevation of the arm above the heart will further decrease perfusion. Pain and coolness are not normal after AVG insertion. Aspirin therapy is not used to maintain grafts. DIF: Cognitive Level: Apply (application) REF: 1088 TOP: Nursing Process: Implementation MSC: NCLEX: Physiological Integrity

After change-of-shift report, which patient will the nurse assess first? a. A 19-yr-old patient with type 1 diabetes who was admitted with possible dawn phenomenon b. A 35-yr-old patient with type 1 diabetes whose most recent blood glucose reading was 230 mg/dL c. A 60-yr-old patient with hyperosmolar hyperglycemic syndrome who has poor skin turgor and dry oral mucosa d. A 68-yr-old patient with type 2 diabetes who has severe peripheral neuropathy and complains of burning foot pain

ANS: C The patient's diagnosis of HHS and signs of dehydration indicate that the nurse should rapidly assess for signs of shock and determine whether increased fluid infusion is needed. The other patients also need assessment and intervention but do not have life-threatening complications. DIF: Cognitive Level: Analyze (analysis) REF: 1146 OBJ: Special Questions: Multiple Patients | Special Questions: Prioritization TOP: Nursing Process: Planning MSC: NCLEX: Safe and Effective Care Environment

A patient with rheumatoid arthritis (RA) complains to the clinic nurse about having chronically dry eyes. Which action by the nurse is appropriate? a. Ask the HCP about discontinuing methotrexate b. Remind the patient that RA is a chronic health condition. c. Suggest the patient use over-the-counter (OTC) artificial tears. d. Teach the patient about adverse effects of the RA medications.

ANS: C The patient's dry eyes are consistent with Sjögren's syndrome, a common extraarticular manifestation of RA. Symptomatic therapy such as OTC eye drops is recommended. Dry eyes are not a side effect of methotrexate. A focus on the prognosis for RA is not helpful. The dry eyes are not caused by RA treatment but by the disease itself. DIF: Cognitive Level: Apply (application) REF: 1546 TOP: Nursing Process: Implementation MSC: NCLEX: Physiological Integrity

Which information obtained during the nurse's assessment of a patient's nutritional-metabolic pattern may indicates increased risk for musculoskeletal problems? a. The patient takes a multivitamin daily. b. The patient dislikes fruits and vegetables. c. The patient is 5 ft, 2 in tall and weighs 180 lb. d. The patient prefers whole milk to nonfat milk.

ANS: C The patient's height and weight indicate obesity, which places stress on weight-bearing joints and predisposes the patient to osteoarthritis. The use of whole milk, avoidance of fruits and vegetables, and use of a daily multivitamin are not risk factors for musculoskeletal problems. DIF: Cognitive Level: Apply (application) REF: 1452 TOP: Nursing Process: Assessment MSC: NCLEX: Health Promotion and Maintenance

5. Which information obtained during the nurse's assessment of a 30-year-old patient's nutritional-metabolic pattern may indicate the risk for musculoskeletal problems? a. The patient takes a multivitamin daily. b. The patient dislikes fruits and vegetables. c. The patient is 5 ft 2 in and weighs 180 lb. d. The patient prefers whole milk to nonfat milk.

ANS: C The patient's height and weight indicate obesity, which places stress on weight-bearing joints. The use of whole milk, avoiding fruits and vegetables, and use of a daily multivitamin are not risk factors for musculoskeletal problems.

A patient screened for diabetes at a clinic has a fasting plasma glucose level of 120 mg/dL (6.7 mmol/L). The nurse will plan to teach the patient about a. self-monitoring of blood glucose. b. using low doses of regular insulin. c. lifestyle changes to lower blood glucose. d. effects of oral hypoglycemic medications.

ANS: C The patient's impaired fasting glucose indicates prediabetes, and the patient should be counseled about lifestyle changes to prevent the development of type 2 diabetes. The patient with prediabetes does not require insulin or oral hypoglycemics for glucose control and does not need to self-monitor blood glucose. DIF: Cognitive Level: Apply (application) REF: 1133 TOP: Nursing Process: Planning MSC: NCLEX: Physiological Integrity

The nurse assesses vital signs for a patient admitted 2 days ago with gram-negative sepsis: temperature of 101.2° F, blood pressure of 90/56 mm Hg, pulse of 92 beats/min, and respirations of 34 breaths/min. Which action should the nurse take next? a. Give the scheduled IV antibiotic. b. Give the PRN acetaminophen (Tylenol). c. Obtain oxygen saturation using pulse oximetry. d. Notify the health care provider of the patient's vital signs.

ANS: C The patient's increased respiratory rate in combination with the admission diagnosis of gram-negative sepsis indicates that acute respiratory distress syndrome (ARDS) may be developing. The nurse should check for hypoxemia, a hallmark of ARDS. The health care provider should be notified after further assessment of the patient. Giving the scheduled antibiotic and the PRN acetaminophen will also be done, but they are not the highest priority for a patient who may be developing ARDS. DIF: Cognitive Level: Analyze (analysis)

The nurse is caring for an older patient who was hospitalized 2 days earlier with community-acquired pneumonia. Which assessment information is most important to communicate to the health care provider? a. Persistent cough of blood-tinged sputum. b. Scattered crackles in the posterior lung bases. c. Oxygen saturation 90% on 100% O2 by nonrebreather mask. d. Temperature 101.5° F (38.6° C) after 2 days of IV antibiotics.

ANS: C The patient's low SpO2 despite receiving a high fraction of inspired oxygen (FIO2) indicates the possibility of acute respiratory distress syndrome (ARDS). The patient's blood-tinged sputum and scattered crackles are not unusual in a patient with pneumonia, although they do require continued monitoring. The continued temperature elevation indicates a possible need to change antibiotics, but this is not as urgent a concern as the progression toward hypoxemia despite an increase in O2 flow rate. DIF: Cognitive Level: Analyze (analysis)

After obtaining the information shown in the accompanying figure regarding a patient with Addison's disease, which prescribed action will the nurse take first? a. Give 4 oz of fruit juice orally. b. Recheck the blood glucose level. c. Infuse 5% dextrose and 0.9% saline. d. Administer O2 therapy as needed.

ANS: C The patient's poor skin turgor, hypotension, and hyponatremia indicate an Addisonian crisis. Immediate correction of the hypovolemia and hyponatremia is needed. The other actions may also be needed but are not the initial action for the patient. DIF: Cognitive Level: Analyze (analysis) REF: 1179 OBJ: Special Questions: Prioritization TOP: Nursing Process: Planning MSC: NCLEX: Physiological Integrity

After a patient experienced a brief episode of tinnitus, diplopia, and dysarthria with no residual effects, the nurse anticipates teaching the patient about a. cerebral aneurysm clipping. c. oral low-dose aspirin therapy. b. heparin intravenous infusion. d. tissue plasminogen activator (tPA).

ANS: C The patient's symptoms are consistent with transient ischemic attack (TIA), and drugs that inhibit platelet aggregation are prescribed after a TIA to prevent a stroke. Continuous heparin infusion is not routinely used after TIA or with acute ischemic stroke. The patient's symptoms are not consistent with a cerebral aneurysm. tPA is used only for acute ischemic stroke, not for TIA. DIF: Cognitive Level: Apply (application) REF: 1353 TOP: Nursing Process: Planning MSC: NCLEX: Physiological Integrity

Which assessment finding for a 33-yr-old female patient admitted with Graves' disease requires the most rapid intervention by the nurse? a. Heart rate 136 beats/min c. Temperature 103.8° F (40.4° C) b. Severe bilateral exophthalmos d. Blood pressure 166/100 mm Hg

ANS: C The patient's temperature indicates that the patient may have thyrotoxic crisis and that interventions to lower the temperature are needed immediately. The other findings also require intervention but do not indicate potentially life-threatening complications. DIF: Cognitive Level: Analyze (analysis) REF: 1165 OBJ: Special Questions: Prioritization TOP: Nursing Process: Assessment MSC: NCLEX: Physiological Integrity

The nurse performing a focused assessment of left posterior temporal lobe functions will assess the patient for a. sensation on the left side of the body. b. reasoning and problem-solving ability. c. ability to understand written and oral language. d. voluntary movements on the right side of the body.

ANS: C The posterior temporal lobe integrates the visual and auditory input for language comprehension. Reasoning and problem solving are functions of the anterior frontal lobe. Sensation on the left side of the body is located in the right postcentral gyrus. Voluntary movement on the right side is controlled in the left precentral gyrus. DIF: Cognitive Level: Apply (application) REF: 1298 TOP: Nursing Process: Assessment MSC: NCLEX: Physiological Integrity

The nurse should reposition the patient who has just had a laminectomy and diskectomy by a. instructing the patient to move the legs before turning the rest of the body. b. having the patient turn by grasping the side rails and pulling the shoulders over. c. placing a pillow between the patient's legs and turning the entire body as a unit. d. turning the patient's head and shoulders first, followed by the hips, legs, and feet.

ANS: C The spine should be kept in correct alignment after laminectomy. The other positions will create misalignment of the spine. DIF: Cognitive Level: Apply (application) REF: 1507 TOP: Nursing Process: Implementation MSC: NCLEX: Physiological Integrity

The home health nurse is caring for an 81-yr-old who had a stroke 2 months ago. Based on information shown in the accompanying figure from the history, physical assessment, and physical and occupational therapy, which problem is the highest priority? History Well controlled type 2 diabetes for 10 years Married 45 years; spouse has heart failure and chronic obstructive pulmonary disease Physical Assessment Oriented to time, place, person Speech clear Minimal left leg weakness Physical/Occupational Therapy Uses cane with walking Spouse does household cleaning and cooking and assists patient with bathing and dressing a. Risk for hypoglycemia c. Risk for caregiver role strain b. Impaired transfer ability d. Ineffective health maintenance

ANS: C The spouse's household and patient care responsibilities, in combination with chronic illnesses, indicate a high risk for caregiver role strain. The nurse should further assess the situation and take appropriate actions. The data about the control of the patient's diabetes indicates that ineffective health maintenance and risk for unstable blood glucose are not priority concerns at this time. Because the patient is able to ambulate with a cane, the nursing diagnosis of impaired transfer ability is not supported. DIF: Cognitive Level: Analyze (analysis) REF: 1362 OBJ: Special Questions: Prioritization TOP: Nursing Process: Diagnosis MSC: NCLEX: Psychosocial Integrity

To assess the functions of the trigeminal and facial nerves (CNs V and VII), the nurse should a. check for unilateral eyelid droop. b. shine a light into the patient's pupil. c. touch a cotton wisp strand to the cornea. d. have the patient read a magazine or book.

ANS: C The trigeminal and facial nerves are responsible for the corneal reflex. The optic nerve is tested by having the patient read a Snellen chart or a newspaper. Assessment of pupil response to light and ptosis are used to evaluate function of the oculomotor nerve. DIF: Cognitive Level: Understand (comprehension) REF: 1305 TOP: Nursing Process: Assessment MSC: NCLEX: Physiological Integrity

8. After completing the health history, the nurse assessing the musculoskeletal system will begin by a. having the patient move the extremities against resistance. b. feeling for the presence of crepitus during joint movement. c. observing the patient's body build and muscle configuration. d. checking active and passive range of motion for the extremities.

ANS: C The usual technique in the physical assessment is to begin with inspection. Abnormalities in muscle mass or configuration will allow the nurse to perform a more focused assessment of abnormal areas. The other assessments are also included in the assessment but are usually done after inspection.

After completing the health history, the nurse assessing the musculoskeletal system will begin by a. having the patient move the extremities against resistance. b. feeling for the presence of crepitus during joint movement. c. observing the patient's body build and muscle configuration. d. checking active and passive range of motion for the extremities.

ANS: C The usual technique in the physical assessment is to begin with inspection. Abnormalities in muscle mass or configuration will allow the nurse to perform a more focused assessment of affected areas. The other assessments are also included but are usually done after inspection. DIF: Cognitive Level: Understand (comprehension) REF: 1453 TOP: Nursing Process: Assessment MSC: NCLEX: Health Promotion and Maintenance

A 62-yr-old patient with hyperthyroidism is to be treated with radioactive iodine (RAI). The nurse instructs the patient a. about radioactive precautions to take with all body secretions. b. that symptoms of hyperthyroidism should be relieved in about a week. c. that symptoms of hypothyroidism may occur as the RAI therapy takes effect. d. to discontinue the antithyroid medications taken before the radioactive therapy.

ANS: C There is a high incidence of postradiation hypothyroidism after RAI, and the patient should be monitored for symptoms of hypothyroidism. RAI has a delayed response, with the maximum effect not seen for 2 to 3 months, and the patient will continue to take antithyroid medications during this time. The therapeutic dose of radioactive iodine is low enough that no radiation safety precautions are needed. DIF: Cognitive Level: Apply (application) REF: 1166 TOP: Nursing Process: Implementation MSC: NCLEX: Physiological Integrity

Which hospitalized patient will the nurse assign to the room closest to the nurses' station? a. Patient with Alzheimer's disease who has long-term memory deficit b. Patient with vascular dementia who takes medications for depression c. Patient with new-onset confusion, restlessness, and irritability after surgery d. Patient with dementia who has an abnormal Mini-Mental State Examination

ANS: C This patient's history and clinical manifestations are consistent with delirium. The patient is at risk for safety problems and should be placed near the nurses' station for ongoing observation. The other patients have chronic symptoms that are consistent with their diagnoses but are not at immediate risk for safety issues. DIF: Cognitive Level: Analyze (analysis) REF: 1415 OBJ: Special Questions: Multiple Patients TOP: Nursing Process: Planning

A 72-yr-old patient is brought to the clinic by the patient's spouse, who reports that the patient is unable to solve common problems around the house. To obtain information about the patient's current mental status, which question should the nurse ask the patient? a. "Are you sad right now?" c. "What did you eat for lunch?" b. "How is your self-image?" d. "Where were you were born?"

ANS: C This question tests the patient's short-term memory, which is decreased in the mild stage of Alzheimer's disease or dementia. Asking the patient about her birthplace tests for remote memory, which is intact in the early stages. Questions about the patient's emotions and self-image are helpful in assessing emotional status, but they are not as helpful in assessing mental state. DIF: Cognitive Level: Apply (application) REF: 1406 TOP: Nursing Process: Assessment MSC: NCLEX: Psychosocial Integrity

A patient is scheduled for transsphenoidal hypophysectomy to treat a pituitary adenoma. During preoperative teaching, the nurse instructs the patient about the need to a. cough and deep breathe every 2 hours postoperatively. b. remain on bed rest for the first 48 hours after the surgery. c. avoid brushing teeth for at least 10 days after the surgery. d. be positioned flat with sandbags at the head postoperatively.

ANS: C To avoid disruption of the suture line, the patient should avoid brushing the teeth for 10 days after surgery. It is not necessary to remain on bed rest after this surgery. Coughing is discouraged because it may cause leakage of cerebrospinal fluid (CSF) from the suture line. The head of the bed should be elevated 30 degrees to reduce pressure on the sella turcica and decrease the risk for headaches. DIF: Cognitive Level: Apply (application) REF: 1159 TOP: Nursing Process: Implementation MSC: NCLEX: Physiological Integrity

A patient who has type 2 diabetes is being prepared for an elective coronary angiogram. Which information would the nurse anticipate might lead to rescheduling the test? a. The patient's most recent A1C was 6.5%. b. The patient's blood glucose is 128 mg/dL. c. The patient took the prescribed metformin today. d. The patient took the prescribed captopril this morning.

ANS: C To avoid lactic acidosis, metformin should be discontinued a day or 2 before the coronary angiogram and should not be used for 48 hours after IV contrast media are administered. The other patient data will also be reported but do not indicate any need to reschedule the procedure. DIF: Cognitive Level: Apply (application) REF: 1130 TOP: Nursing Process: Assessment MSC: NCLEX: Physiological Integrity

Which prescribed intervention will the nurse implement first for a patient in the emergency department who is experiencing continuous tonic-clonic seizures? a. Give phenytoin (Dilantin) 100 mg IV. b. Monitor level of consciousness (LOC). c. Administer lorazepam (Ativan) 4 mg IV. d. Obtain computed tomography (CT) scan.

ANS: C To prevent ongoing seizures, the nurse should administer rapidly acting antiseizure medications such as the benzodiazepines. A CT scan is appropriate, but prevention of any seizure activity during the CT scan is necessary. Phenytoin will also be administered, but it is not rapidly acting. Patients who are experiencing tonic-clonic seizures are nonresponsive, although the nurse should assess LOC after the seizure. DIF: Cognitive Level: Analyze (analysis) REF: 1378 OBJ: Special Questions: Prioritization TOP: Nursing Process: Implementation MSC: NCLEX: Physiological Integrity

The nurse is caring for a patient receiving a continuous norepinephrine IV infusion. Which patient assessment finding indicates that the infusion rate may need to be adjusted? a. Heart rate is slow at 58 beats/min. b. Mean arterial pressure (MAP) is 56 mm Hg. c. Systemic vascular resistance (SVR) is elevated. d. Pulmonary artery wedge pressure (PAWP) is low.

ANS: C Vasoconstrictors such as norepinephrine will increase SVR, and this will increase the work of the heart and decrease peripheral perfusion. The infusion rate may need to be decreased. Bradycardia, hypotension (MAP of 56 mm Hg), and low PAWP are not associated with norepinephrine infusion. DIF: Cognitive Level: Apply (application)

During the change of shift report, a nurse is told that a patient has an occluded left posterior cerebral artery. The nurse will anticipate that the patient may have a. dysphasia. c. visual deficits. b. confusion. d. poor judgment.

ANS: C Visual disturbances are expected with posterior cerebral artery occlusion. Aphasia occurs with middle cerebral artery involvement. Cognitive deficits and changes in judgment are more typical of anterior cerebral artery occlusion. DIF: Cognitive Level: Apply (application) REF: 1350 TOP: Nursing Process: Assessment MSC: NCLEX: Physiological Integrity

A nurse is assessing a patient who is receiving a nitroprusside infusion to treat cardiogenic shock. Which finding indicates that the drug is effective? a. No new heart murmurs c. Warm, pink, and dry skin b. Decreased troponin level d. Blood pressure of 92/40 mm Hg

ANS: C Warm, pink, and dry skin indicates that perfusion to tissues is improved. Because nitroprusside is a vasodilator, the blood pressure may be low even if the drug is effective. Absence of a heart murmur and a decrease in troponin level are not indicators of improvement in shock. DIF: Cognitive Level: Apply (application)

The nurse is assessing a 22-yr-old patient experiencing the onset of symptoms of type 1 diabetes. To which question would the nurse anticipate a positive response? a. "Are you anorexic?" b. "Is your urine dark colored?" c. "Have you lost weight lately?" d. "Do you crave sugary drinks?"

ANS: C Weight loss occurs because the body is no longer able to absorb glucose and starts to break down protein and fat for energy. The patient is thirsty but does not necessarily crave sugar-containing fluids. Increased appetite is a classic symptom of type 1 diabetes. With the classic symptom of polyuria, urine will be very dilute. DIF: Cognitive Level: Apply (application) REF: 1121 TOP: Nursing Process: Assessment MSC: NCLEX: Physiological Integrity

A patient with diabetes who has bacterial pneumonia is being treated with IV gentamicin 60 mg IV BID. The nurse will monitor for adverse effects of the medication by evaluating the patient's a. blood glucose. b. urine osmolality. c. serum creatinine. d. serum potassium.

ANS: C When a patient at risk for chronic kidney disease (CKD) receives a potentially nephrotoxic medication, it is important to monitor renal function with BUN and creatinine levels. The other laboratory values would not be useful in assessing for the adverse effects of the gentamicin. DIF: Cognitive Level: Apply (application) REF: 1083 TOP: Nursing Process: Evaluation MSC: NCLEX: Physiological Integrity

After receiving 2 L of normal saline, the central venous pressure for a patient who has septic shock is 10 mm Hg, but the blood pressure is still 82/40 mm Hg. The nurse will anticipate an order for a. furosemide . c. norepinephrine . b. nitroglycerin . d. sodium nitroprusside .

ANS: C When fluid resuscitation is unsuccessful, vasopressor drugs are given to increase the systemic vascular resistance (SVR) and blood pressure and improve tissue perfusion. Furosemide would cause diuresis and further decrease the BP. Nitroglycerin would decrease the preload and further drop cardiac output and BP. Nitroprusside is an arterial vasodilator and would further decrease SVR. DIF: Cognitive Level: Apply (application)

The charge nurse is observing a new staff nurse who is assessing a patient with a traumatic spinal cord injury for sensation. Which action indicates a need for further teaching of the new nurse about neurologic assessment? a. The new nurse tests for light touch before testing for pain. b. The new nurse has the patient close the eyes during testing. c. The new nurse asks the patient if the instrument feels sharp. d. The new nurse uses an irregular pattern to test for intact touch.

ANS: C When performing a sensory assessment, the nurse should not provide verbal clues. The other actions by the new nurse are appropriate. DIF: Cognitive Level: Apply (application) REF: 1306 OBJ: Special Questions: Delegation TOP: Nursing Process: Evaluation MSC: NCLEX: Safe and Effective Care Environment

An 82-yr-old patient in a long-term care facility is newly diagnosed with hypothyroidism. The nurse will need to consult with the health care provider before administering the prescribed a. docusate (Colace). c. diazepam (Valium). b. ibuprofen (Motrin). d. cefoxitin (Mefoxin).

ANS: C Worsening of mental status and myxedema coma can be precipitated by the use of sedatives, especially in older adults. The nurse should discuss the use of diazepam with the health care provider before administration. The other medications may be given safely to the patient. DIF: Cognitive Level: Apply (application) REF: 1169 TOP: Nursing Process: Implementation MSC: NCLEX: Physiological Integrity

Which information will the nurse include when teaching a patient with acute low back pain (select all that apply)? a. Sleep in a prone position with the legs extended. b. Keep the knees straight when leaning forward to pick something up. c. Expect symptoms of acute low back pain to improve in a few weeks. d. Avoid activities that require twisting of the back or prolonged sitting. e. Use ibuprofen (Motrin, Advil) or acetaminophen (Tylenol) to relieve

ANS: C, D, E Acute back pain usually starts to improve within 2 weeks. In the meantime, the patient should use medications such as nonsteroidal antiinflammatory drugs (NSAIDs) or acetaminophen to manage pain and avoid activities that stress the back. Sleeping in a prone position and keeping the knees straight when leaning forward will place stress on the back and should be avoided. DIF: Cognitive Level: Apply (application) REF: 1503 TOP: Nursing Process: Planning MSC: NCLEX: Physiological Integrity

The emergency department (ED) nurse is starting therapeutic hypothermia in a patient who has been resuscitated after a cardiac arrest. Which actions in the hypothermia protocol can be delegated to an experienced licensed practical/vocational nurse (LPN/LVN) (select all that apply)? a. Continuously monitor heart rhythm. b. Assess neurologic status every 2 hours. c. Give acetaminophen (Tylenol) 650 mg. d. Place cooling blankets above and below patient. e. Attach rectal temperature probe to cooling blanket control panel.

ANS: C, D, E Experienced LPN/LVNs have the education and scope of practice to implement hypothermia measures (e.g., cooling blanket, temperature probe) and administer medications under the supervision of a registered nurse (RN). Assessment of neurologic status and monitoring the heart rhythm require RN-level education and scope of practice and should be done by the RN. DIF: Cognitive Level: Apply (application)

A patient with a left-brain stroke suddenly bursts into tears when family members visit. The nurse should a. use a calm voice to ask the patient to stop the crying behavior. b. explain to the family that depression is normal following a stroke. c. have the family members leave the patient alone for a few minutes. d. teach the family that emotional outbursts are common after strokes.

ANS: D Patients who have left-sided brain stroke are prone to emotional outbursts that are not necessarily related to the emotional state of the patient. Depression after a stroke is common, but the suddenness of the patient's outburst suggests that depression is not the major cause of the behavior. The family should stay with the patient. The crying is not within the patient's control, and asking the patient to stop will lead to embarrassment. DIF: Cognitive Level: Apply (application) REF: 1364 TOP: Nursing Process: Implementation MSC: NCLEX: Psychosocial Integrity

3. The nurse who notes that a 59-year-old female patient has lost 1 inch in height over the past 2 years will plan to teach the patient about a.discography studies. b.myelographic testing. c.magnetic resonance imaging (MRI). d.dual-energy x-ray absorptiometry (DXA).

ANS: D The decreased height and the patient's age suggest that the patient may have osteoporosis and that bone density testing is needed. Discography, MRI, and myelography are typically done for patients with current symptoms caused by musculoskeletal dysfunction and are not the initial diagnostic tests for osteoporosis.

A 44-yr-old female patient with Cushing syndrome is admitted for adrenalectomy. Which intervention by the nurse will be most helpful for the patient problem of disturbed body image related to changes in appearance? a. Reassure the patient that the physical changes are very common in patients with Cushing syndrome. b. Discuss the use of diet and exercise in controlling the weight gain associated with Cushing syndrome. c. Teach the patient that the metabolic impact of Cushing syndrome is of more importance than appearance. d. Remind the patient that most of the physical changes caused by Cushing syndrome will resolve after surgery.

ANS: D The most reassuring and accurate communication to the patient is that the physical and emotional changes caused by the Cushing syndrome will resolve after hormone levels return to normal postoperatively. Reassurance that the physical changes are expected or that there are more serious physiologic problems associated with Cushing syndrome are not therapeutic responses. The patient's physiological changes are caused by the high hormone levels, not by the patient's diet or exercise choices. DIF: Cognitive Level: Apply (application) REF: 1177 TOP: Nursing Process: Implementation MSC: NCLEX: Psychosocial Integrity

When rewarming a patient who arrived in the emergency department (ED) with a temperature of 87° F (30.6° C), which finding indicates that the nurse should discontinue active rewarming? a. The patient begins to shiver. b. The BP decreases to 86/42 mm Hg. c. The patient develops atrial fibrillation. d. The core temperature is 94° F (34.4° C).

ANS: D A core temperature of at least 89.6° F to 93.2° F (32° C to 34° C) indicates that sufficient rewarming has occurred. Dysrhythmias, hypotension, and shivering may occur during rewarming, and should be treated but are not an indication to stop rewarming the patient. DIF: Cognitive Level: Apply (application)

The nurse assessing the urinary system of a 45-yr-old patient would use palpation to a. determine kidney function. c. check for ureteral peristalsis. b. identify renal artery bruits. d. assess for bladder distention.

ANS: D A distended bladder may be palpable above the symphysis pubis. Palpation would not be helpful in assessing for the other listed urinary tract information. DIF: Cognitive Level: Understand (comprehension) REF: 1023 TOP: Nursing Process: Assessment MSC: NCLEX: Physiological Integrity

A patient with chronic obstructive pulmonary disease (COPD) arrives in the emergency department complaining of shortness of breath and dyspnea on minimal exertion. Which assessment finding by the nurse is most important to report to the health care provider? a. The patient has bibasilar lung crackles. b. The patient is sitting in the tripod position. c. The patient's pulse oximetry indicates a 91% O2 saturation. d. The patient's respirations have dropped to 10 breaths/minute.

ANS: D A drop in respiratory rate in a patient with respiratory distress suggests the onset of fatigue and a high risk for respiratory arrest. Therefore immediate action such as positive-pressure ventilation is needed. Patients who are experiencing respiratory distress frequently sit in the tripod position because it decreases the work of breathing. Crackles in the lung bases may be the baseline for a patient with COPD. An O2 saturation of 91% is common in patients with COPD and will provide adequate gas exchange and tissue oxygenation.

Which finding for a 77-yr-old patient seen in the outpatient clinic requires further nursing assessment and intervention? a. Symmetric joint swelling of fingers b. Decreased right knee range of motion c. Report of left hip aching when jogging d. History of recent loss of balance and fall

ANS: D A history of falls requires further assessment and development of fall prevention strategies. The other changes are more typical of bone and joint changes associated with normal aging. DIF: Cognitive Level: Apply (application) REF: 1450 TOP: Nursing Process: Assessment MSC: NCLEX: Physiological Integrity

13. Which finding is of highest priority when the nurse is planning care for a 77-year-old patient seen in the outpatient clinic? a. Symmetric joint swelling of fingers b. Decreased right knee range of motion c. Report of left hip aching when jogging d. History of recent loss of balance and fall

ANS: D A history of falls requires further assessment and development of fall prevention strategies. The other changes are more typical of bone and joint changes associated with normal aging.

7. The nurse finds that a patient can flex the arms when no resistance is applied but is unable to flex when the nurse applies light resistance. The nurse should document the patient's muscle strength as level a.0. b.1. c.2. d.3.

ANS: D A level 3 indicates that the patient is unable to move against resistance but can move against gravity. Level 1 indicates minimal muscle contraction, level 2 indicates that the arm can move when gravity is eliminated, and level 4 indicates active movement with some resistance.

After change-of-shift report on the Alzheimer's disease/dementia unit, which patient will the nurse assess first? a. Patient who has not had a bowel movement for 5 days b. Patient who has a stage II pressure ulcer on the coccyx c. Patient who is refusing to take the prescribed medications d. Patient who developed a new cough after eating breakfast

ANS: D A new cough after a meal in a patient with dementia suggests possible aspiration, and the patient should be assessed immediately. The other patients also require assessment and intervention but not as urgently as a patient with possible aspiration or pneumonia. DIF: Cognitive Level: Analyze (analysis) REF: 1413 OBJ: Special Questions: Prioritization | Special Questions: Multiple Patients TOP: Nursing Process: Assessment MSC: NCLEX: Safe and Effective Care Environment

The emergency department (ED) nurse receives report that a seriously injured patient involved in a motor vehicle crash is being transported to the facility with an estimated arrival in 5 minutes. In preparation for the patient's arrival, the nurse will obtain a. a dopamine infusion. c. lactated Ringer's solution. b. a hypothermia blanket. d. two 16-gauge IV catheters.

ANS: D A patient with multiple trauma may require fluid resuscitation to prevent or treat hypovolemic shock, so the nurse will anticipate the need for 2 large-bore IV lines to administer normal saline. Lactated Ringer's solution should be used cautiously and will not be ordered until the patient has been assessed for possible liver abnormalities. Vasopressor infusion is not used as the initial therapy for hypovolemic shock. Patients in shock need to be kept warm not cool. DIF: Cognitive Level: Apply (application)

Which information is most important for the nurse to communicate rapidly to the health care provider about a patient admitted with possible syndrome of inappropriate antidiuretic hormone (SIADH)? a. The patient has a recent weight gain of 9 lb. b. The patient complains of dyspnea with activity. c. The patient has a urine specific gravity of 1.025. d. The patient has a serum sodium level of 118 mEq/L.

ANS: D A serum sodium of less than 120 mEq/L increases the risk for complications such as seizures and needs rapid correction. The other data are not unusual for a patient with SIADH and do not indicate the need for rapid action. DIF: Cognitive Level: Analyze (analysis) REF: 1160 OBJ: Special Questions: Prioritization TOP: Nursing Process: Assessment MSC: NCLEX: Physiological Integrity

Which assessment finding for a patient who has had surgical reduction of an open fracture of the right radius requires notification of the health care provider? a. Serous wound drainage c. Right arm pain with movement b. Right arm muscle spasms d. Temperature 101.4° F (38.6° C)

ANS: D An elevated temperature suggests possible osteomyelitis. The other clinical manifestations are typical after a repair of an open fracture. DIF: Cognitive Level: Apply (application) REF: 1499 OBJ: Special Questions: Prioritization TOP: Nursing Process: Assessment MSC: NCLEX: Physiological Integrity

A patient being admitted with bacterial meningitis has a temperature of 102.5° F (39.2° C) and a severe headache. Which order should the nurse implement first? a. Administer ceftizoxime (Cefizox) 1 g IV. b. Give acetaminophen (Tylenol) 650 mg PO. c. Use a cooling blanket to lower temperature. d. Swab the nasopharyngeal mucosa for cultures.

ANS: D Antibiotic therapy should be instituted rapidly in bacterial meningitis, but cultures must be done before antibiotics are started. As soon as the cultures are done, the antibiotic should be started. Hypothermia therapy and acetaminophen administration are appropriate but can be started after the other actions are implemented. DIF: Cognitive Level: Analyze (analysis)

Which medication taken by a patient with restless legs syndrome should the nurse discuss with the patient? a. Ibuprofen b. Multivitamin c. Acetaminophen d. Diphenhydramine

ANS: D Antihistamines can aggravate restless legs syndrome. The other medications will not contribute to restless legs syndrome. DIF: Cognitive Level: Apply (application) REF: 1383 TOP: Nursing Process: Implementation MSC: NCLEX: Physiological Integrity

Which of these nursing actions for a patient with Guillain-Barré syndrome is appropriate for the nurse to delegate to experienced unlicensed assistive personnel (UAP)? a. Nasogastric tube feeding q4hr b. Artificial tear administration q2hr c. Assessment for bladder distention q2hr d. Passive range of motion to extremities q4hr

ANS: D Assisting a patient with movement is included in UAP education and scope of practice. Administration of tube feedings, administration of ordered medications, and assessment are skills requiring more education and expanded scope of practice, and the RN should perform these skills. DIF: Cognitive Level: Apply (application)

Which is the correct point on the accompanying figure where the nurse will assess for ecchymosis when admitting a patient with a basilar skull fracture? a. A(eye) b. B(upper lip) c. C(ear) d. D(base of skull)

ANS: D Battle's sign (postauricular ecchymosis) and periorbital ecchymoses are associated with basilar skull fracture. DIF: Cognitive Level: Understand (comprehension)

When assessing a patient with chronic obstructive pulmonary disease (COPD), the nurse finds a new onset of agitation and confusion. Which action should the nurse take first? a. Observe for facial symmetry. b. Notify the health care provider. c. Attempt to calm and reorient the patient. d. Assess oxygenation using pulse oximetry.

ANS: D Because agitation and confusion are frequently the initial indicators of hypoxemia, the nurse's initial action should be to assess O2 saturation. The other actions are also appropriate, but assessment of oxygenation takes priority over other assessments and notification of the health care provider. DIF: Cognitive Level: Analyze (analysis)

A patient with acute respiratory distress syndrome (ARDS) who is intubated and receiving mechanical ventilation develops a right pneumothorax. Which collaborative action will the nurse anticipate next? a. Increase the tidal volume and respiratory rate. b. Decrease the fraction of inspired oxygen (FIO2). c. Perform endotracheal suctioning more frequently. d. Lower the positive end-expiratory pressure (PEEP).

ANS: D Because barotrauma is associated with high airway pressures, the level of PEEP should be decreased. The other actions will not decrease the risk for another pneumothorax. DIF: Cognitive Level: Apply (application)

A hospitalized patient with possible renal insufficiency after coronary artery bypass surgery is scheduled for a creatinine clearance test. Which item will the nurse need to obtain? a. Urinary catheter c. Cleansing towelettes b. Sterile specimen cup d. Large urine container

ANS: D Because creatinine clearance testing involves a 24-hour urine specimen, the nurse should obtain a large container for the urine collection. Catheterization, cleaning of the perineum with antiseptic towelettes, and a sterile specimen cup are not needed for this test. DIF: Cognitive Level: Understand (comprehension) REF: 1031 TOP: Nursing Process: Implementation MSC: NCLEX: Physiological Integrity

Which finding for a patient admitted with glomerulonephritis indicates to the nurse that treatment has been effective? a. The patient denies pain with voiding. b. The urine dipstick is negative for nitrites. c. The antistreptolysin-O (ASO) titer has decreased. d. The periorbital and peripheral edema are resolved.

ANS: D Because edema is a common clinical manifestation of glomerulonephritis, resolution of the edema indicates that the prescribed therapies have been effective. Nitrites will be negative and the patient will not experience dysuria because the patient does not have a urinary tract infection. Antibodies to streptococcus will persist after a streptococcal infection.

Which assessment information is most important for the nurse to obtain when evaluating whether treatment of a patient with anaphylactic shock has been effective? a. Heart rate c. Blood pressure b. Orientation d. Oxygen saturation

ANS: D Because the airway edema that is associated with anaphylaxis can affect airway and breathing, the O2 saturation is the most critical assessment. Improvements in the other assessments will also be expected with effective treatment of anaphylactic shock. DIF: Cognitive Level: Analyze (analysis)

The nurse evaluating effectiveness of prescribed calcitonin and ibandronate (Boniva) for a patient with Paget's disease will consider the patient's a. oral intake. c. grip strength. b. daily weight. d. pain intensity.

ANS: D Bone pain is a common early manifestation of Paget's disease, and the nurse should assess the pain intensity to determine if treatment is effective. The other information will also be collected by the nurse but will not be used in evaluating the effectiveness of the therapy. DIF: Cognitive Level: Apply (application) REF: 1514 TOP: Nursing Process: Evaluation MSC: NCLEX: Physiological Integrity

A hospitalized diabetic patient received 38 U of NPH insulin at 7:00 AM. At 1:00 PM, the patient has been away from the nursing unit for 2 hours, missing the lunch delivery while awaiting a chest x-ray. To prevent hypoglycemia, the best action by the nurse is to a. save the lunch tray for the patient's later return to the unit. b. ask that diagnostic testing area staff to start a 5% dextrose IV. c. send a glass of milk or orange juice to the patient in the diagnostic testing area. d. request that if testing is further delayed, the patient be returned to the unit to eat.

ANS: D Consistency for mealtimes assists with regulation of blood glucose, so the best option is for the patient to have lunch at the usual time. Waiting to eat until after the procedure is likely to cause hypoglycemia. Administration of an IV solution is unnecessarily invasive for the patient. A glass of milk or juice will keep the patient from becoming hypoglycemic but will cause a rapid rise in blood glucose because of the rapid absorption of the simple carbohydrate in these items. DIF: Cognitive Level: Analyze (analysis) REF: 1127 TOP: Nursing Process: Implementation MSC: NCLEX: Physiological Integrity

Which finding by the nurse will be most helpful in determining whether a 67-yr-old patient with benign prostatic hyperplasia has an upper urinary tract infection (UTI)? a. Bladder distention b. Foul-smelling urine c. Suprapubic discomfort d. Costovertebral tenderness

ANS: D Costovertebral tenderness is characteristic of pyelonephritis. Bladder distention, foul-smelling urine, and suprapubic discomfort are characteristic of a lower UTI and are likely to be present if the patient also has an upper UTI.

Which statement by a 50-yr-old female patient indicates to the nurse that further assessment of thyroid function may be necessary? a. "I notice my breasts are tender lately." b. "I am so thirsty that I drink all day long." c. "I get up several times at night to urinate." d. "I feel a lump in my throat when I swallow."

ANS: D Difficulty in swallowing can occur with a goiter. Nocturia is associated with diseases such as diabetes mellitus, diabetes insipidus, or chronic kidney disease. Breast tenderness would occur with excessive gonadal hormone levels. Thirst is a sign of disease such as diabetes. DIF: Cognitive Level: Apply (application) REF: 1109 TOP: Nursing Process: Assessment MSC: NCLEX: Physiological Integrity

A patient is being treated with carbidopa/levodopa (Sinemet) for Parkinson's disease. Which information indicates a need for change in the medication or dosage? a. Shuffling gait c. Cogwheel rigidity of limbs b. Tremor at rest d. Uncontrolled head movement

ANS: D Dyskinesia is an adverse effect of the Sinemet, indicating a need for a change in medication or decrease in dose. The other findings are typical with Parkinson's disease. DIF: Cognitive Level: Apply (application) REF: 1390 TOP: Nursing Process: Planning MSC: NCLEX: Physiological Integrity

Which nursing action is needed when preparing to assist with the insertion of a pulmonary artery catheter? a. Determine if the cardiac troponin level is elevated. b. Auscultate heart sounds before and during insertion. c. Place the patient on NPO status before the procedure. d. Attach cardiac monitoring leads before the procedure.

ANS: D Dysrhythmias can occur as the catheter is floated through the right atrium and ventricle, and it is important for the nurse to monitor for these during insertion. Pulmonary artery catheter insertion does not require anesthesia, and the patient will not need to be NPO. Changes in cardiac troponin or heart and breath sounds are not expected during pulmonary artery catheter insertion. DIF: Cognitive Level: Apply (application)

The nurse notes premature ventricular contractions (PVCs) while suctioning a patient's endotracheal tube. Which next action by the nurse is indicated? a. Plan to suction the patient more frequently. b. Decrease the suction pressure to 80 mm Hg. c. Give antidysrhythmic medications per protocol. d. Stop and ventilate the patient with 100% oxygen.

ANS: D Dysrhythmias during suctioning may indicate hypoxemia or sympathetic nervous system stimulation. The nurse should stop suctioning and ventilate the patient with 100% O2. There is no indication that more frequent suctioning is needed. Lowering the suction pressure will decrease the effectiveness of suctioning without improving the hypoxemia. Because the PVCs occurred during suctioning, there is no need for antidysrhythmic medications (which may have adverse effects) unless they recur when the suctioning is stopped and patient is well oxygenated. DIF: Cognitive Level: Apply (application)

To assess whether there is any improvement in a patient's dysuria, which question will the nurse ask? a. "Do you have to urinate at night?" b. "Do you have blood in your urine?" c. "Do you have to urinate frequently?" d. "Do you have pain when you urinate?"

ANS: D Dysuria is painful urination. The alternate responses are used to assess other urinary tract symptoms: hematuria, nocturia, and frequency. DIF: Cognitive Level: Understand (comprehension) REF: 1025 TOP: Nursing Process: Assessment MSC: NCLEX: Physiological Integrity

A young adult who is employed as a hairdresser and has a 15 pack-year history of cigarette smoking is scheduled for an annual physical examination. The nurse will plan to teach the patient about the increased risk for a. renal failure. c. pyelonephritis. b. kidney stones. d. bladder cancer.

ANS: D Exposure to the chemicals involved with working as a hairdresser and in smoking both increase the risk of bladder cancer, and the nurse should assess whether the patient understands this risk. The patient is not at increased risk for renal failure, pyelonephritis, or kidney stones. DIF: Cognitive Level: Apply (application) REF: 1021 TOP: Nursing Process: Planning MSC: NCLEX: Health Promotion and Maintenance

A patient with suspected meningitis is scheduled for a lumbar puncture. Before the procedure, the nurse will plan to a. enforce NPO status for 4 hours. b. transfer the patient to radiology. c. administer a sedative medication. d. help the patient to a lateral position.

ANS: D For a lumbar puncture, the patient lies in the lateral recumbent position. The procedure does not usually require a sedative, is done in the patient room, and has no risk for aspiration. DIF: Cognitive Level: Apply (application) REF: 1310 TOP: Nursing Process: Planning MSC: NCLEX: Physiological Integrity

A 19-yr-old patient is brought to the emergency department (ED) with multiple lacerations and tissue avulsion of the left hand. When asked about tetanus immunization, the patient denies having any previous vaccinations. The nurse will anticipate giving a. tetanus immunoglobulin (TIG) only. b. TIG and tetanus-diphtheria toxoid (Td). c. tetanus-diphtheria toxoid and pertussis vaccine (Tdap) only. d. TIG and tetanus-diphtheria toxoid and pertussis vaccine (Tdap).

ANS: D For an adult with no previous tetanus immunizations, TIG and Tdap are recommended. The other immunizations are not sufficient for this patient. DIF: Cognitive Level: Apply (application)

An 18-yr-old male patient with small stature is scheduled for a growth hormone stimulation test. In preparation for the test, the nurse will obtain a. ice in a basin. c. a cardiac monitor. b. glargine insulin. d. 50% dextrose solution.

ANS: D Hypoglycemia is induced during the growth hormone stimulation test, and the nurse should be ready to administer 50% dextrose immediately. Regular insulin is used to induce hypoglycemia (glargine is never given IV). The patient does not require cardiac monitoring during the test. Although blood samples for some tests must be kept on ice, this is not true for the growth hormone stimulation test. DIF: Cognitive Level: Apply (application) REF: 1115 TOP: Nursing Process: Planning MSC: NCLEX: Physiological Integrity

A 62-yr-old patient who has Parkinson's disease is taking bromocriptine (Parlodel). Which information obtained by the nurse may indicate a need for a decrease in the dosage? a. The patient has a chronic dry cough. b. The patient has four loose stools in a day. c. The patient develops a deep vein thrombosis. d. The patient's blood pressure is 92/52 mm Hg.

ANS: D Hypotension is an adverse effect of bromocriptine, and the nurse should check with the health care provider before giving the medication. Diarrhea, cough, and deep vein thrombosis are not associated with bromocriptine use. DIF: Cognitive Level: Apply (application) REF: 1390 TOP: Nursing Process: Evaluation MSC: NCLEX: Physiological Integrity

The nurse is caring for a patient with a subarachnoid hemorrhage who is intubated and placed on a mechanical ventilator with 10 cm H2O of peak end-expiratory pressure (PEEP). When monitoring the patient, the nurse will need to notify the health care provider immediately if the patient develops a. O2 saturation of 93%. b. green nasogastric tube drainage. c. respirations of 20 breaths/minute. d. increased jugular venous distention.

ANS: D Increases in jugular venous distention in a patient with a subarachnoid hemorrhage may indicate an increase in intracranial pressure (ICP) and that the PEEP setting is too high for this patient. A respiratory rate of 20, O2 saturation of 93%, and green nasogastric tube drainage are within normal limits. DIF: Cognitive Level: Apply (application)

Which nursing interventions included in the care of a mechanically ventilated patient with acute respiratory failure can the registered nurse (RN) delegate to an experienced licensed practical/vocational nurse (LPN/LVN) working in the intensive care unit? a. Assess breath sounds every hour. b. Monitor central venous pressures. c. Place patient in the prone position. d. Insert an indwelling urinary catheter.

ANS: D Insertion of indwelling urinary catheters is included in LPN/LVN education and scope of practice and can be safely delegated to an LPN/LVN who is experienced in caring for critically ill patients. Placing a patient who is on a ventilator in the prone position requires multiple staff, and should be supervised by an RN. Assessment of breath sounds and obtaining central venous pressures require advanced assessment skills and should be done by the RN caring for a critically ill patient. DIF: Cognitive Level: Apply (application)

When working in the urology/nephrology clinic, which patient could the nurse delegate to an experienced licensed practical/vocational nurse (LPN/LVN)? a. Patient who is scheduled for a renal biopsy after a recent kidney transplant b. Patient who will need monitoring for several hours after a renal arteriogram c. Patient who requires teaching about possible post-cystoscopy complications d. Patient who will have catheterization to check for residual urine after voiding

ANS: D LPN/LVN education includes common procedures such as catheterization of stable patients. The other patients require more complex assessments or patient teaching that are included in registered nurse (RN) education and scope of practice. DIF: Cognitive Level: Apply (application) REF: 1026 OBJ: Special Questions: Delegation | Special Questions: Multiple Patients TOP: Nursing Process: Planning MSC: NCLEX: Safe and Effective Care Environment

A patient has been admitted with meningococcal meningitis. Which observation by the nurse requires action? a. The patient receives a regular diet tray. b. The bedrails on both sides of the bed are elevated. c. Staff have turned off the lights in the patient's room. d. Staff have entered the patient's room without a mask.

ANS: D Meningococcal meningitis is spread by respiratory secretions, so it is important to maintain respiratory isolation as well as standard precautions. Because the patient may be confused and weak, bedrails should be elevated at both the foot and head of the bed. Low light levels in the room decrease pain caused by photophobia. Nutrition is an important aspect of care in a patient with meningitis. DIF: Cognitive Level: Apply (application)

The emergency department (ED) triage nurse is assessing four victims involved in a motor vehicle collision. Which patient has the highest priority for treatment? a. A patient with no pedal pulses b. A patient with an open femur fracture c. A patient with bleeding facial lacerations d. A patient with paradoxical chest movement

ANS: D Most immediate deaths from trauma occur because of problems with ventilation, so the patient with paradoxical chest movements should be treated first. Face and head fractures can obstruct the airway, but the patient with facial injuries only has lacerations. The other two patients also need rapid intervention but do not have airway or breathing problems. DIF: Cognitive Level: Analyze (analysis)

A patient complains of leg cramps during hemodialysis. The nurse should a. massage the patient's legs. b. reposition the patient supine. c. give acetaminophen (Tylenol). d. infuse a bolus of normal saline.

ANS: D Muscle cramps during dialysis are caused by rapid removal of sodium and water. Treatment includes infusion of normal saline. The other actions do not address the reason for the cramps. DIF: Cognitive Level: Apply (application) REF: 1091 TOP: Nursing Process: Implementation MSC: NCLEX: Physiological Integrity

The nurse finds that a patient can flex the arms when no resistance is applied but is unable to flex when the nurse applies light resistance. The nurse should document the patient's muscle strength as level a. 0. c. 2. b. 1. d. 3.

ANS: D Muscle strength of 3 indicates the patient is unable to move against resistance but can move against gravity. Level 1 indicates minimal muscle contraction, level 2 indicates the arm can move when gravity is eliminated, and level 4 indicates active movement with some resistance. DIF: Cognitive Level: Understand (comprehension) REF: 1455 TOP: Nursing Process: Assessment MSC: NCLEX: Health Promotion and Maintenance

Which finding by the nurse when assessing a patient with a large pituitary adenoma is most important to report to the health care provider? a. Changes in visual field c. Blood glucose 150 mg/dL b. Milk leaking from breasts d. Nausea and projectile vomiting

ANS: D Nausea and projectile vomiting may indicate increased intracranial pressure, which will require rapid actions for diagnosis and treatment. Changes in the visual field, elevated blood glucose, and galactorrhea are common with pituitary adenoma, but these do not require rapid action to prevent life-threatening complications. DIF: Cognitive Level: Analyze (analysis) REF: 1157 OBJ: Special Questions: Prioritization TOP: Nursing Process: Assessment MSC: NCLEX: Physiological Integrity

The nurse assesses a 78-yr-old who uses naproxen (Aleve) daily for hand and knee osteoarthritis management. Which information requires a discussion with the health care provider about an urgent change in the treatment plan? a. Knee crepitation is noted with normal knee range of motion. b. Patient reports embarrassment about having Heberden's nodes. c. Patient's knee pain while golfing has increased over the last year. d. Laboratory results indicate blood urea nitrogen (BUN) is elevated.

ANS: D Older patients are at increased risk for renal toxicity caused by nonsteroidal antiinflammatory drugs (NSAIDs) such as naproxen. The other information will also be reported to the health care provider but is consistent with the patient's diagnosis of osteoarthritis and will not require an immediate change in the patient's treatment plan. DIF: Cognitive Level: Apply (application) REF: 1523 TOP: Nursing Process: Assessment MSC: NCLEX: Physiological Integrity

When monitoring the effectiveness of treatment for a patient with a large anterior wall myocardial infarction, the most pertinent measurement for the nurse to obtain is a. central venous pressure (CVP). b. systemic vascular resistance (SVR). c. pulmonary vascular resistance (PVR). d. pulmonary artery wedge pressure (PAWP).

ANS: D PAWP reflects left ventricular end diastolic pressure (or left ventricular preload) and is a sensitive indicator of cardiac function. Because the patient is high risk for left ventricular failure, the PAWP must be monitored. An increase will indicate left ventricular failure. The other values would also provide useful information, but the most definitive measurement of changes in cardiac function is the PAWP. DIF: Cognitive Level: Apply (application)

The nurse identifies a patient with type 1 diabetes and a history of herpes simplex infection as being at risk for Bell's palsy. Which information should the nurse include in teaching the patient? a. "You may be able to prevent Bell's palsy by doing facial exercises regularly." b. "Prophylactic treatment of herpes with antiviral agents prevents Bell's palsy." c. "Medications to treat Bell's palsy work only if started before paralysis onset." d. "Call the doctor if you experience pain or develop herpes lesions near the ear."

ANS: D Pain or herpes lesions near the ear may indicate the onset of Bell's palsy, and rapid corticosteroid treatment may reduce the duration of Bell's palsy symptoms. Antiviral therapy for herpes simplex does not reduce the risk for Bell's palsy. Corticosteroid therapy will be most effective in reducing symptoms if started before paralysis is complete but will still be somewhat effective when started later. Facial exercises do not prevent Bell's palsy. DIF: Cognitive Level: Apply (application)

Which assessment data reported by a patient is consistent with a lower urinary tract infection (UTI)? a. Low urine output b. Bilateral flank pain c. Nausea and vomiting d. Burning on urination

ANS: D Pain with urination is a common symptom of a lower UTI. Urine output does not decrease, but frequency may be experienced. Flank pain and nausea are associated with an upper UTI.

A 61-yr-old female patient admitted with pneumonia has a total serum calcium level of 13.3 mg/dL (3.3 mmol/L). The nurse will anticipate the need to teach the patient about testing for _____ levels. a. calcitonin b. catecholamine c. thyroid hormone d. parathyroid hormone

ANS: D Parathyroid hormone (PTH) is the major controller of blood calcium levels. Although calcitonin secretion is a countermechanism to PTH, it does not play a major role in calcium balance. Catecholamine and thyroid hormone levels do not affect serum calcium level. DIF: Cognitive Level: Apply (application) REF: 1103 TOP: Nursing Process: Planning MSC: NCLEX: Physiological Integrity

The nurse is caring for a patient who was admitted the previous day with a basilar skull fracture after a motor vehicle crash. Which assessment finding indicates a possible complication that should be reported to the health care provider? a. Complaint of severe headache b. Large contusion behind left ear c. Bilateral periorbital ecchymosis d. Temperature of 101.4° F (38.6° C)

ANS: D Patients who have basilar skull fractures are at risk for meningitis, so the elevated temperature should be reported to the health care provider. The other findings are typical of a patient with a basilar skull fracture. DIF: Cognitive Level: Apply (application)

A 54-yr-old woman who recently reached menopause and has a family history of osteoporosis is diagnosed with osteopenia following densitometry testing. In teaching the woman, the nurse explains that a. with a family history of osteoporosis, there is no way to prevent or slow bone resorption. b. estrogen replacement therapy must be started to prevent rapid progression to osteoporosis. c. continuous, low-dose corticosteroid treatment is effective in stopping the course of osteoporosis. d. calcium loss from bones can be slowed by increasing calcium intake and weight-bearing exercise.

ANS: D Progression of osteoporosis can be slowed by increasing calcium intake and weight-bearing exercise. Estrogen replacement therapy is no longer routinely given to prevent osteoporosis because of increased risk of heart disease as well as breast and uterine cancer. Corticosteroid therapy increases the risk for osteoporosis. DIF: Cognitive Level: Apply (application) REF: 1512 TOP: Nursing Process: Implementation MSC: NCLEX: Physiological Integrity

Nurses in change-of-shift report are discussing the care of a patient with a stroke who has progressively increasing weakness and decreasing level of consciousness. Which patient problem do they determine has the highest priority for the patient? a. Impaired physical mobility related to weakness b. Disturbed sensory perception related to brain injury c. Risk for impaired skin integrity related to immobility d. Risk for aspiration related to inability to protect airway

ANS: D Protection of the airway is the priority of nursing care for a patient having an acute stroke. The other diagnoses are also appropriate, but interventions to prevent aspiration are the priority at this time. DIF: Cognitive Level: Analyze (analysis) REF: 1354 OBJ: Special Questions: Prioritization TOP: Nursing Process: Analysis MSC: NCLEX: Physiological Integrity

A patient develops increasing dyspnea and hypoxemia 2 days after heart surgery. To determine whether the patient has acute respiratory distress syndrome (ARDS) or pulmonary edema caused by heart failure, the nurse will plan to assist with a. obtaining a ventilation-perfusion scan. b. drawing blood for arterial blood gases. c. positioning the patient for a chest x-ray. d. insertion of a pulmonary artery catheter.

ANS: D Pulmonary artery wedge pressures are normal in the patient with ARDS because the fluid in the alveoli is caused by increased permeability of the alveolar-capillary membrane rather than by the backup of fluid from the lungs (as occurs in cardiogenic pulmonary edema). The other tests will not help in differentiating cardiogenic from noncardiogenic pulmonary edema. DIF: Cognitive Level: Apply (application)

After having a craniectomy and left anterior fossae incision, a 64-yr-old patient has impaired physical mobility related to decreased level of consciousness and weakness. An appropriate nursing intervention is to a. cluster nursing activities to allow longer rest periods. b. turn and reposition the patient side to side every 2 hours. c. position the bed flat and log roll to reposition the patient. d. perform range-of-motion (ROM) exercises every 4 hours.

ANS: D ROM exercises will help prevent the complications of immobility. Patients with anterior craniotomies are positioned with the head elevated. The patient with a craniectomy should not be turned to the operative side. When the patient is weak, clustering nursing activities may lead to more fatigue and weakness. DIF: Cognitive Level: Apply (application)

A patient reporting painful urination and knee pain is diagnosed with reactive arthritis. The nurse will plan to teach the patient about the need for several months of therapy with a. methotrexate b. anakinra (Kineret). c. etanercept (Enbrel). d. doxycycline (Vibramycin).

ANS: D Reactive arthritis associated with urethritis is usually caused by infection with Chlamydia trachomatis and requires 3 months of treatment with doxycycline. The other medications are used for chronic inflammatory problems such as rheumatoid arthritis. DIF: Cognitive Level: Apply (application) REF: 1538 TOP: Nursing Process: Planning MSC: NCLEX: Physiological Integrity

A patient with a stroke experiences facial drooping on the right side and right-sided arm and leg paralysis. When admitting the patient, which clinical manifestation will the nurse expect to find? a. Impulsive behavior b. Right-sided neglect c. Hyperactive left-sided tendon reflexes d. Difficulty comprehending instructions

ANS: D Right-sided paralysis indicates a left-brain stroke, which will lead to difficulty with comprehension and use of language. The left-side reflexes are likely to be intact. Impulsive behavior and neglect are more likely with a right-side stroke. DIF: Cognitive Level: Apply (application) REF: 1350 TOP: Nursing Process: Assessment MSC: NCLEX: Physiological Integrity

The nurse is interviewing a new patient with diabetes who takes rosiglitazone (Avandia). Which information would the nurse anticipate resulting in the health care provider discontinuing the medication? a. The patient's blood pressure is 154/92. b. The patient's blood glucose is 86 mg/dL. c. The patient reports a history of emphysema. d. The patient has chest pressure when walking.

ANS: D Rosiglitazone can cause myocardial ischemia. The nurse should immediately notify the health care provider and expect orders to discontinue the medication. A blood glucose level of 86 mg/dL indicates a positive effect from the medication. Hypertension and a history of emphysema do not contraindicate this medication. DIF: Cognitive Level: Apply (application) REF: 1130 TOP: Nursing Process: Assessment MSC: NCLEX: Physiological Integrity

The nurse will anticipate teaching a patient with a possible seizure disorder about which test? a. Cerebral angiography b. Evoked potential studies c. Electromyography (EMG) d. Electroencephalography (EEG)

ANS: D Seizure disorders are usually assessed using EEG testing. Evoked potential is used to diagnose problems with the visual or auditory systems. Cerebral angiography is used to diagnose vascular problems. EMG is used to evaluate electrical innervation to skeletal muscle. DIF: Cognitive Level: Understand (comprehension) REF: 1311 TOP: Nursing Process: Planning MSC: NCLEX: Physiological Integrity

When assessing a 53-yr-old patient with bacterial meningitis, the nurse obtains the following data. Which finding requires the most immediate intervention? a. The patient exhibits nuchal rigidity. b. The patient has a positive Kernig's sign. c. The patient's temperature is 101° F (38.3° C). d. The patient's blood pressure is 88/42 mm Hg.

ANS: D Shock is a serious complication of meningitis, and the patient's low blood pressure indicates the need for interventions such as fluids or vasopressors. Nuchal rigidity and a positive Kernig's sign are expected with bacterial meningitis. The nurse should intervene to lower the temperature, but this is not as life threatening as the hypotension. DIF: Cognitive Level: Analyze (analysis)

Which menu choice by a patient with osteoporosis indicates the nurse's teaching about appropriate diet has been effective? a. Pancakes with syrup and bacon b. Whole wheat toast and fresh fruit c. Egg-white omelet and a half grapefruit d. Oatmeal with skim milk and fruit yogurt

ANS: D Skim milk and yogurt are high in calcium. The other choices do not contain any high-calcium foods. DIF: Cognitive Level: Apply (application) REF: 1512 TOP: Nursing Process: Evaluation MSC: NCLEX: Physiological Integrity

Which finding indicates a need to contact the health care provider before the nurse administers metformin (Glucophage)? a. The patient's blood glucose level is 174 mg/dL. b. The patient is scheduled for a chest x-ray in an hour. c. The patient has gained 2 lb (0.9 kg) in the past 24 hours. d. The patient's blood urea nitrogen (BUN) level is 52 mg/dL.

ANS: D The BUN indicates possible renal failure, and metformin should not be used in patients with renal failure. The other findings are not contraindications to the use of metformin. DIF: Cognitive Level: Apply (application) REF: 1130 TOP: Nursing Process: Assessment MSC: NCLEX: Physiological Integrity

During the primary survey of a patient with severe leg trauma, the nurse observes that the patient's left pedal and posterior tibial pulses are absent and the entire leg is swollen. Which action will the nurse take next? a. Send blood to the lab for a complete blood count. b. Assess further for a cause of the decreased circulation. c. Finish the airway, breathing, circulation, disability survey. d. Start normal saline fluid infusion with a large-bore IV line.

ANS: D The assessment data indicate that the patient may have arterial trauma and hemorrhage. When a possibly life-threatening injury is found during the primary survey, the nurse should immediately start interventions before proceeding with the survey. Although a complete blood count is indicated, administration of IV fluids should be started first. Completion of the primary survey and further assessment should be completed after the IV fluids are initiated. DIF: Cognitive Level: Analyze (analysis)

Which nursing diagnosis is of highest priority for a patient with Parkinson's disease who is unable to move the facial muscles? a. Activity intolerance b. Self-care deficit: toileting c. Ineffective self-health management d. Imbalanced nutrition: less than body requirements

ANS: D The data about the patient indicate that poor nutrition will be a concern because of decreased swallowing. The other diagnoses may also be appropriate for a patient with Parkinson's disease, but the data do not indicate that they are current problems for this patient. DIF: Cognitive Level: Analyze (analysis) REF: 1391 OBJ: Special Questions: Prioritization TOP: Nursing Process: Analysis MSC: NCLEX: Physiological Integrity

Which assessment finding of a 42-yr-old patient who had a bilateral adrenalectomy requires the most rapid action by the nurse? a. The blood glucose is 192 mg/dL. b. The lungs have bibasilar crackles. c. The patient reports 6/10 incisional pain. d. The blood pressure (BP) is 88/50 mm Hg.

ANS: D The decreased BP indicates possible adrenal insufficiency. The nurse should immediately notify the health care provider so that corticosteroid medications can be administered. The nurse should also address the elevated glucose, incisional pain, and crackles with appropriate collaborative or nursing actions, but prevention and treatment of acute adrenal insufficiency are the priorities after adrenalectomy. DIF: Cognitive Level: Analyze (analysis) REF: 1176 OBJ: Special Questions: Prioritization TOP: Nursing Process: Assessment MSC: NCLEX: Physiological Integrity

The nurse who notes that a 59-yr-old female patient has lost 1 inch in height over the past 2 years will plan to teach the patient about a. discography studies. b. myelographic testing. c. magnetic resonance imaging (MRI). d. dual-energy x-ray absorptiometry (DXA).

ANS: D The decreased height and the patient's age suggest that the patient may have osteoporosis, and bone density testing is needed. Discography, MRI, and myelography are typically done for patients with current symptoms caused by musculoskeletal dysfunction and are not the initial diagnostic tests for osteoporosis. DIF: Cognitive Level: Apply (application) REF: 1458 TOP: Nursing Process: Planning MSC: NCLEX: Health Promotion and Maintenance

After the emergency department nurse has received a status report on the following patients who have been admitted with head injuries, which patient should the nurse assess first? a. A 20-yr-old patient whose cranial x-ray shows a linear skull fracture b. A 50-yr-old patient who has an initial Glasgow Coma Scale score of 13 c. A 30-yr-old patient who lost consciousness for a few seconds after a fall d. A 40-yr-old patient whose right pupil is 10 mm and unresponsive to light

ANS: D The dilated and nonresponsive pupil may indicate an intracerebral hemorrhage and increased intracranial pressure. The other patients are not at immediate risk for complications such as herniation. DIF: Cognitive Level: Analyze (analysis)

After change-of-shift report, which patient should the progressive care nurse assess first? a. Patient who was extubated this morning and has a temperature of 101.4°F (38.6°C) b. Patient with bilevel positive airway pressure (BiPAP) for obstructive sleep apnea and a respiratory rate of 16 c. Patient with arterial pressure monitoring who is 2 hours post-percutaneous coronary intervention and needs to void d. Patient who is receiving IV heparin for a venous thromboembolism and has a partial thromboplastin time (PTT) of 101 sec

ANS: D The findings for this patient indicate high risk for bleeding from an elevated (nontherapeutic) PTT. The nurse needs to adjust the rate of the infusion (dose) per the health care provider's parameters. The patient with BiPAP for sleep apnea has a normal respiratory rate. The patient recovering from the percutaneous coronary intervention will need to be assisted with voiding and this task could be delegated to unlicensed assistive personnel. The patient with a fever may be developing ventilator-associated pneumonia, but addressing the bleeding risk is a higher priority. DIF: Cognitive Level: Analyze (analysis)

The nurse educator is evaluating the care that a new registered nurse (RN) provides to a patient receiving mechanical ventilation. Which action by the new RN indicates the need for more education? a. The RN increases the FIO2 to 100% before suctioning. b. The RN secures a bite block in place using adhesive tape. c. The RN asks for assistance to resecure the endotracheal tube. d. The RN positions the patient with the head of bed at 10 degrees.

ANS: D The head of the patient's bed should be positioned at 30 to 45 degrees to prevent ventilator-associated pneumonia. The other actions by the new RN are appropriate. DIF: Cognitive Level: Apply (application)

A patient with acute kidney injury (AKI) has longer QRS intervals on the electrocardiogram (ECG) than were noted on the previous shift. Which action should the nurse take first? a. Notify the patient's health care provider. b. Document the QRS interval measurement. c. Review the chart for the patient's current creatinine level. d. Check the medical record for the most recent potassium level.

ANS: D The increasing QRS interval is suggestive of hyperkalemia, so the nurse should check the most recent potassium and then notify the patient's health care provider. The BUN and creatinine will be elevated in a patient with AKI, but they would not directly affect the electrocardiogram (ECG). Documentation of the QRS interval is also appropriate, but interventions to decrease the potassium level are needed to prevent life-threatening dysrhythmias. DIF: Cognitive Level: Analyze (analysis) REF: 1072 OBJ: Special Questions: Prioritization TOP: Nursing Process: Implementation MSC: NCLEX: Physiological Integrity

A patient arrives in the emergency department (ED) after topical exposure to powdered lime at work. Which action should the nurse take first? a. Obtain the patient's vital signs. b. Obtain a baseline complete blood count. c. Decontaminate the patient by showering with water. d. Brush off any visible powder on the skin and clothing.

ANS: D The initial action should be to protect staff members and decrease the patient's exposure to the toxin by decontamination. Patients exposed to powdered lime should not be showered; instead, any and all visible powder should be brushed off. The other actions can be done after the decontamination is completed. DIF: Cognitive Level: Analyze (analysis)

While waiting for heart transplantation, a patient with severe cardiomyopathy has a ventricular assist device (VAD) implanted. When planning care for this patient, the nurse should anticipate a. preparing the patient for a permanent VAD. b. administering immunosuppressive medications. c. teaching the patient the reason for complete bed rest. d. monitoring the surgical incision for signs of infection.

ANS: D The insertion site for the VAD provides a source for transmission of infection to the circulatory system and requires frequent monitoring. Patients with VADs are able to have some mobility and may not be on bed rest. The VAD is a bridge to transplantation, not a permanent device. Immunosuppression is not necessary for nonbiologic devices such as the VAD. DIF: Cognitive Level: Apply (application)

The home health nurse is making a follow-up visit to a patient with recently diagnosed rheumatoid arthritis (RA). Which assessment made by the nurse indicates more patient teaching is needed? a. The patient takes a 2-hour nap each day. b. The patient has been taking 16 aspirins each day. c. The patient sits on a stool while preparing meals. d. The patient sleeps with two pillows under the head.

ANS: D The joints should be maintained in an extended position to avoid contractures, so patients should use a small, flat pillow for sleeping. Rest, aspirin, and energy management are appropriate for a patient with RA and indicate teaching has been effective. DIF: Cognitive Level: Apply (application) REF: 1531 TOP: Nursing Process: Evaluation MSC: NCLEX: Physiological Integrity

. The nurse responds to a ventilator alarm and finds the patient lying in bed gasping and holding the endotracheal tube (ET) in her hand. Which action should the nurse take next? a. Activate the rapid response team. b. Provide reassurance to the patient. c. Call the health care provider to reinsert the tube. d. Manually ventilate the patient with 100% oxygen.

ANS: D The nurse should ensure maximal patient oxygenation by manually ventilating with a bag-valve-mask system. Offering reassurance to the patient, notifying the health care provider about the need to reinsert the tube, and activating the rapid response team are also appropriate after the nurse has stabilized the patient's oxygenation. DIF: Cognitive Level: Analyze (analysis)

The nurse will determine more teaching is needed if a patient with discomfort from a bunion says, "I will a. give away my high-heeled shoes." b. take ibuprofen (Motrin) if I need it." c. use the bunion pad to cushion the area." d. only wear sandals, no closed-toe shoes."

ANS: D The patient can wear shoes that have a wide forefoot (toe box). The other patient statements indicate the teaching has been effective. DIF: Cognitive Level: Apply (application) REF: 1509 TOP: Nursing Process: Evaluation MSC: NCLEX: Physiological Integrity

The nurse has been teaching a patient with type 2 diabetes about managing blood glucose levels and taking glipizide (Glucotrol). Which patient statement indicates a need for additional teaching? a. "If I overeat at a meal, I will still take the usual dose of medication." b. "Other medications besides the Glucotrol may affect my blood sugar." c. "When I am ill, I may have to take insulin to control my blood sugar." d. "My diabetes won't cause complications because I don't need insulin."

ANS: D The patient should understand that type 2 diabetes places the patient at risk for many complications and that good glucose control is as important when taking oral agents as when using insulin. The other statements are accurate and indicate good understanding of the use of glipizide. DIF: Cognitive Level: Apply (application) REF: 1130 TOP: Nursing Process: Evaluation MSC: NCLEX: Physiological Integrity

After receiving change-of-shift report, which patient should the nurse assess first? a. Patient who is scheduled for the drain phase of a peritoneal dialysis exchange b. Patient with stage 4 chronic kidney disease who has an elevated phosphate level c. Patient with stage 5 chronic kidney disease who has a potassium level of 3.4 mEq/L d. Patient who has just returned from having hemodialysis and has a heart rate of 124/min

ANS: D The patient who has tachycardia after hemodialysis may be bleeding or excessively hypovolemic and should be assessed immediately for these complications. The other patients also need assessments or interventions but are not at risk for life-threatening complications. DIF: Cognitive Level: Analyze (analysis) REF: 1091 OBJ: Special Questions: Prioritization | Special Questions: Multiple Patients TOP: Nursing Process: Assessment MSC: NCLEX: Safe and Effective Care Environment

Four hours after mechanical ventilation is initiated, a patient's arterial blood gas (ABG) results include a pH of 7.51, PaO2 of 82 mm Hg, PaCO2 of 26 mm Hg, and HCO3- of 23 mEq/L (23 mmol/L). The nurse will anticipate the need to a. increase the FIO2. c. increase the respiratory rate. b. increase the tidal volume. d. decrease the respiratory rate.

ANS: D The patient's PaCO2 and pH indicate respiratory alkalosis caused by too high a respiratory rate. The PaO2 is appropriate for a patient with COPD and increasing the respiratory rate and tidal volume would further lower the PaCO2. DIF: Cognitive Level: Analyze (analysis)

Which statement by a patient who had a cystoscopy the previous day should be reported immediately to the health care provider? a. "My urine looks pink." c. "My sleep was restless." b. "My IV site is bruised." d. "My temperature is 101."

ANS: D The patient's elevated temperature may indicate a bladder infection, a possible complication of cystoscopy. The health care provider should be notified so that antibiotic therapy can be started. Pink-tinged urine is expected after a cystoscopy. The insomnia and bruising should be discussed further with the patient but do not indicate a need to notify the health care provider. DIF: Cognitive Level: Analyze (analysis) REF: 1028 OBJ: Special Questions: Prioritization TOP: Nursing Process: Assessment MSC: NCLEX: Physiological Integrity

A patient arrives in the emergency department with hemiparesis and dysarthria that started 2 hours previously, and health records show a history of several transient ischemic attacks (TIAs). The nurse anticipates preparing the patient for a. surgical endarterectomy. b. transluminal angioplasty. c. intravenous heparin drip administration. d. tissue plasminogen activator (tPA) infusion.

ANS: D The patient's history and clinical manifestations suggest an acute ischemic stroke, and a patient who is seen within 4.5 hours of stroke onset is likely to receive tPA (after screening with a CT scan). Heparin administration in the emergency phase is not indicated. Emergent carotid transluminal angioplasty or endarterectomy is not indicated for the patient who is having an acute ischemic stroke. DIF: Cognitive Level: Apply (application) REF: 1355 TOP: Nursing Process: Planning MSC: NCLEX: Physiological Integrity

The nurse is concerned about a postoperative patient's risk for injury during an episode of delirium. The most appropriate action by the nurse is to a. secure the patient in bed using a soft chest restraint. b. ask the health care provider to order an antipsychotic drug. c. instruct family members to remain at the patient's bedside and prevent injury. d. assign unlicensed assistive personnel (UAP) to stay with and reorient the patient.

ANS: D The priority goal is to protect the patient from harm. Having a UAP stay with the patient will ensure the patient's safety. Visits by family members are helpful in reorienting the patient, but families should not be responsible for protecting patients from injury. Antipsychotic medications may be ordered, but only if other measures are not effective because these medications have many side effects. Restraints are not recommended because they can increase the patient's agitation and disorientation. DIF: Cognitive Level: Analyze (analysis) Apply (application) REF: 1412 TOP: Nursing Process: Implementation MSC: NCLEX: Safe and Effective Care Environment

A patient in the emergency department with sudden-onset right-sided weakness is diagnosed with an intracerebral hemorrhage. Which information about the patient is most important to communicate to the health care provider? a. The patient's speech is difficult to understand. b. The patient's blood pressure (BP) is 144/90 mm Hg. c. The patient takes a diuretic because of a history of hypertension. d. The patient has atrial fibrillation and takes warfarin (Coumadin).

ANS: D The use of warfarin probably contributed to the intracerebral bleeding and remains a risk factor for further bleeding. Administration of vitamin K is needed to reverse the effects of the warfarin, especially if the patient is to have surgery to correct the bleeding. The history of hypertension is a risk factor for the patient but has no immediate effect on the patient's care. The BP of 144/90 indicates the need for ongoing monitoring but not for any immediate change in therapy. Slurred speech is consistent with a left-sided stroke, and no change in therapy is indicated. DIF: Cognitive Level: Analyze (analysis) REF: 1349 OBJ: Special Questions: Prioritization TOP: Nursing Process: Assessment MSC: NCLEX: Physiological Integrity

After receiving change-of-shift report on a medical unit, which patient should the nurse assess first? a. A patient with cystic fibrosis who has thick, green-colored sputum b. A patient with pneumonia who has crackles bilaterally in the lung bases c. A patient with emphysema who has an oxygen saturation of 90% to 92% d. A patient with septicemia who has intercostal and suprasternal retractions

ANS: D This patient's history of septicemia and labored breathing suggest the onset of ARDS, which will require rapid interventions such as administration of O2 and use of positive-pressure ventilation. The other patients should also be assessed, but their assessment data are typical of their disease processes and do not suggest deterioration in their status. DIF: Cognitive Level: Analyze (analysis)

A 28-yr-old male patient with type 1 diabetes reports how he manages his exercise and glucose control. Which behavior indicates that the nurse should implement additional teaching? a. The patient always carries hard candies when engaging in exercise. b. The patient goes for a vigorous walk when his glucose is 200 mg/dL. c. The patient has a peanut butter sandwich before going for a bicycle ride. d. The patient increases daily exercise when ketones are present in the urine.

ANS: D When the patient is ketotic, exercise may result in an increase in blood glucose level. Patients with type 1 diabetes should be taught to avoid exercise when ketosis is present. The other statements are correct. DIF: Cognitive Level: Apply (application) REF: 1134 TOP: Nursing Process: Assessment MSC: NCLEX: Physiological Integrity

Which prescribed medication should the nurse expect will have rapid effects on a patient admitted to the emergency department in thyroid storm? a. Iodine c. Propylthiouracil b. Methimazole d. Propranolol (Inderal)

ANS: D b-Adrenergic blockers work rapidly to decrease the cardiovascular manifestations of thyroid storm. The other medications take days to weeks to have an impact on thyroid function. DIF: Cognitive Level: Apply (application) REF: 1165 TOP: Nursing Process: Implementation MSC: NCLEX: Physiological Integrity

ANS: A Tobacco use is a modifiable risk factor for AD. The patient will not be able to modify the increased risk associated with family history of AD and past head injury. While the total cholesterol is borderline high, the high HDL indicates that no change is needed in cholesterol management. DIF: Cognitive Level: Analyze (analysis) REF: 1402 OBJ: Special Questions: Prioritization TOP: Nursing Process: Assessment MSC: NCLEX: Health Promotion and Maintenance

After reviewing the health record shown in the accompanying figure for a patient who has multiple risk factors for Alzheimer's disease (AD), which topic will be most important for the nurse to discuss with the patient? a. Tobacco use c. Cholesterol level b. Family history d. Head injury history

A 42-year-old man who is scheduled for an arthrocentesis arrives at the outpatient surgery unit and states, "I do not want this procedure done today." Which response by the nurse is most appropriate? A "When would you like to reschedule the procedure?" B "Tell me what your concerns are about this procedure." C "The procedure is safe, so why should you be worried?" D "The procedure is not painful because an anesthetic is used."

B "Tell me what your concerns are about this procedure." The nurse should use therapeutic communication to determine the patient's concern about the procedure. The nurse should not provide false reassurance. It is not appropriate for the nurse to assume the patient is concerned about pain or to assume the patient is asking to reschedule the procedure.

A 57-year-old postmenopausal woman is scheduled for dual-energy x-ray absorptiometry (DXA). Which statement, if made by the patient to the nurse, indicates understanding of the procedure? A "The bone density in my heel will be measured." B "This procedure will not cause any pain or discomfort." C "I will not be exposed to any radiation during the procedure." D "I will need to remove my hearing aids before the procedure."

B "This procedure will not cause any pain or discomfort." Dual-energy x-ray absorptiometry (DXA) is painless and measures the bone mass of spine, femur, forearm, and total body with minimal radiation exposure. A quantitative ultrasound (QUS) evaluates density, elasticity, and strength of bone using ultrasound of the calcaneus (heel). Magnetic resonance imaging would require removal of objects such as hearing aids that have metal parts.

A female patient with a long-standing history of rheumatoid arthritis has sought care because of increasing stiffness in her right knee that has culminated in complete fixation of the joint. The nurse would document the presence of which problem? A Atrophy B Ankylosis C Crepitation D Contracture

B Ankylosis Ankylosis is stiffness or fixation of a joint, whereas contracture is reduced movement as a consequence of fibrosis of soft tissue (muscles, ligaments, or tendons). Atrophy is a flabby appearance of muscle leading to decreased function and tone. Crepitation is a grating or crackling sound that accompanies movement.

The home care nurse visits an 84-year-old woman with pneumonia after her discharge from the hospital. Which assessment finding would the nurse expect because of age-related changes in the musculoskeletal system? A Positive straight-leg-raising test B Muscle strength is scale grade 3/5 C Lateral S-shaped curvature of the spine D Fingers drift to the ulnar side of the forearm

B Muscle strength is scale grade 3/5 Decreased muscle strength is an age-related change of the musculoskeletal system caused by decreased number and size of the muscle cells. The other assessment findings indicate musculoskeletal abnormalities. A positive straight-leg-raising test indicates nerve root irritation from intervertebral disk prolapse and herniation. An ulnar deviation or drift indicates rheumatoid arthritis due to tendon contracture. Scoliosis is a lateral curvature of the spine.

In reviewing bone remodeling, what should the nurse know about the involvement of bone cells? A Osteoclasts add canaliculi. B Osteoblasts deposit new bone. C Osteocytes are mature bone cells. D Osteons create a dense bone structure.

B Osteoblasts deposit new bone. Bone remodeling is achieved when osteoclasts remove old bone and osteoblasts deposit new bone. Osteocytes are mature bone cells, and osteons or Haversian systems create a dense bone structure, but these are not involved with bone remodeling.

A 54-year-old patient admitted with cellulitis and probable osteomyelitis received an injection of radioisotope at 9:00 AM before a bone scan. The nurse should plan to send the patient for the bone scan at what time? A 9:30 PM B 10:00 AM C 11:00 AM D 1:00 PM

C 11:00 AM A technician usually administers a calculated dose of a radioisotope 2 hours before a bone scan. If the patient was injected at 9:00 AM, the procedure should be done at 11:00 AM. 10:00 AM would be too early; 1:00 PM and 9:30 PM would be too late.

A 63-year-old woman has been taking prednisone (Deltasone) daily for several years after a kidney transplant to prevent organ rejection. What is most important for the nurse to assess? A Staggering gait Incorrect B Ruptured tendon C Back or neck pain D Tardive dyskinesia

C Back or neck pain Osteoporosis with resultant fractures is a frequent and serious complication of systemic corticosteroid therapy. The ribs and vertebrae are affected the most, and patients should be observed for signs of compression fractures (back and neck pain). Phenytoin (Dilantin) is an antiseizure medication. An adverse effect of phenytoin is an ataxic (or staggering) gait. A rare adverse effect of ciprofloxacin (Cipro) and other fluoroquinolones is tendon rupture, usually of the Achilles tendon. The highest risk is in people age 60 and older and in people taking corticosteroids. Antipsychotics and antidepressants may cause tardive dyskinesia, which is characterized by involuntary movements of the tongue and face.

An 82-year-old patient is frustrated by her flabby belly and rigid hips. What should the nurse tell the patient about these frustrations? A "You should go on a diet and exercise more to feel better about yourself." B "Something must be wrong with you because you should not have these problems." C "You have arthritis and need to go on nonsteroidal antiinflammatory drugs (NSAIDs)." D "Decreased muscle mass and strength and increased hip rigidity are normal changes of aging."

D "Decreased muscle mass and strength and increased hip rigidity are normal changes of aging." The musculoskeletal system's normal changes of aging include decreased muscle mass and strength; increased rigidity in the hips, neck, shoulders, back, and knees; decreased fine motor dexterity; and slowed reaction times. Going on a diet and exercising will help but not stop these changes. Telling the patient "Something must be wrong with you..." will not be helpful to the patient's frustrations.

A 54-year-old patient is about to have a bone scan. In teaching the patient about this procedure, the nurse should include what information? A Two additional follow-up scans will be required. B There will be only mild pain associated with the procedure. C The procedure takes approximately 15 to 30 minutes to complete. D The patient will be asked to drink increased fluids after the procedure.

D The patient will be asked to drink increased fluids after the procedure. Patients are asked to drink increased fluids after a bone scan to aid in excretion of the radioisotope, if not contraindicated by another condition. No follow-up scans and no pain are associated with bone scans that take 1 hour of lying supine.

ANS: B Costovertebral angle (CVA) tenderness with percussion suggests pyelonephritis or polycystic kidney disease. DIF: Cognitive Level: Understand (comprehension) REF: 1024 OBJ: Special Questions: Alternate item format: Hot spot TOP: Nursing Process: Assessment MSC: NCLEX: Health Promotion and Maintenance

When assessing a patient with a urinary tract infection, indicate on the accompanying figure where the nurse will percuss to assess for possible pyelonephritis. a. 1 b. 2 c. 3 d. 4

ANS: D Swan neck deformity involves distal interphalangeal joint hyperflexion and proximal interphalangeal joint hyperextension of the hands. The other deformities are also associated with rheumatoid arthritis: ulnar drift, boutonniere deformity, and hallux vagus. DIF: Cognitive Level: Understand (comprehension) REF: 1527 TOP: Nursing Process: Assessment MSC: NCLEX: Physiological Integrity

When reviewing the health record for a new patient with rheumatoid arthritis, the nurse reads that the patient has swan neck deformities. Which deformity will the nurse expect to observe when assessing the patient? a. A b. B c. C d. D

2. Drugs or diseases that impair the function of the extrapyramidal system may cause loss of a. sensations of pain and temperature. b. regulation of the autonomic nervous system. c. integration of somatic and special sensory inputs. d. automatic movements associated with skeletal muscle activity.

2. d

3. During the admitting neurologic examination, the nurse determines the patient has speech difficulties as well as weakness of the right arm and lower face. The nurse would expect a CT scan to show pathology in the distribution of the a. basilar artery. b. left middle cerebral artery. c. right anterior cerebral artery. d. left posterior communicating artery.

3. b,

4. A patient is seen in the emergency department after diving into the pool and hitting the bottom with a blow to the face that hyperextended the neck and scraped the skin off the nose. The patient also described "having double vision" when looking down. During the neurologic exam, the nurse finds the patient is unable to abduct either eye. The nurse recognizes this finding is related to a. a basal skull fracture. b. a stretch injury to bilateral CN VI. c. a stiff neck from the hyperextension injury. d. facial swelling from the scrape on the bottom of the pool.

4. b,

5. Stimulation of the parasympathetic nervous system results in (select all that apply) a. constriction of the bronchi. b. dilation of skin blood vessels. c. increased secretion of insulin. d. increased blood glucose levels. e. relaxation of the urinary sphincters.

5. a, b, c, e,

6. The nurse is assessing the muscle strength of an older adult patient. The nurse knows the findings cannot be compared with those of a younger adult because a. nutritional status is better in young adults. b. muscle bulk and strength decrease in older adults. c. muscle strength should be the same for all adults. d. most young adults exercise more than older adults.

6. b,

7. A patient is admitted with a headache, fever, and general malaise. The HCP has asked that the patient be prepared for a lumbar puncture. What is a priority nursing action to avoid complications? a. Ensure that CT scan is performed prior to lumbar puncture. b. Assess laboratory results for changes in the white cell count. c. Provide acetaminophen for the headache and fever before the procedure. d. Administer antibiotics before the procedure to treat the potential meningitis.

7. a,

8. During neurologic testing, the patient is able to perceive pain elicited by pinprick. Based on this finding, the nurse may omit testing for a. position sense. b. patellar reflexes. c. temperature perception. d. heel-to-shin movements.

8. c,

9. A patient's eyes jerk while the patient looks to the left. The nurse will record this finding as a. nystagmus. b. CN VI palsy. c. ophthalmic dyskinesia. d. oculocephalic response.

9. a,

The nurse admits a 55-year-old female with multiple sclerosis to a long-term care facility. Which finding is of most immediate concern to the nurse? A Ataxic gait B Radicular pain C Severe fatigue D Urinary retention

A Ataxic gait An ataxic gait is a staggering, uncoordinated gait. Fall risk is the highest in individuals with gait instability or visual or cognitive impairments. The other signs and symptoms (e.g., fatigue, urinary retention, radicular pain) may also occur in the patient with multiple sclerosis and need to be managed, but are not the priority.

A 50-year-old patient is reporting a sore shoulder after raking the yard. The nurse should suspect which problem? A Bursitis B Fasciitis C Sprained ligament D Achilles tendonitis

A Bursitis Bursitis is common in adults over age 40 and with repetitive motion, such as raking. Plantar fasciitis frequently occurs as a stabbing pain at the heel caused by straining the ligament that supports the arch. Achilles tendonitis is an inflammation of the tendon that attaches the calf muscle to the heel bone, not the shoulder, and causes pain with walking or running. A sprained ligament occurs when a ligament is stretched or torn from a direct injury or sudden twisting of the joint, not repetitive motion.

Musculoskeletal assessment is an important component of care for patients on what type of long-term therapy? A Corticosteroids B β-Adrenergic blockers C Antiplatelet aggregators D Calcium-channel blockers

A Corticosteroids Corticosteroids are associated with avascular necrosis and decreased bone and muscle mass. β-blockers, calcium-channel blockers, and antiplatelet aggregators are not commonly associated with damage to the musculoskeletal system.

When working with patients, the nurse knows that patients have the most difficulties with diarthrodial joints. Which joints are included in this group of joints (select all that apply)? A Hinge joint of the knee B Ligaments joining the vertebrae C Fibrous connective tissue of the skull D Ball and socket joint of the shoulder or hip E Cartilaginous connective tissue of the pubis joint

A Hinge joint of the knee D Ball and socket joint of the shoulder or hip The diarthrodial joints include the hinge joint of the knee and elbow, the ball and socket joint of the shoulder and hip, the pivot joint of the radioulnar joint, and the condyloid, saddle, and gliding joints of the wrist and hand. The ligaments and cartilaginous connective tissue joining the vertebrae and pubis joint and the fibrous connective tissue of the skull are synarthrotic joints.

The nurse is performing a musculoskeletal assessment of an 81-year-old female patient whose mobility has been progressively decreasing in recent months. How should the nurse best assess the patient's range of motion (ROM) in the affected leg? A Observe the patient's unassisted ROM in the affected leg. B Perform passive ROM, asking the patient to report any pain. C Ask the patient to lift progressive weights with the affected leg. D Move both of the patient's legs from a supine position to full flexion.

A Observe the patient's unassisted ROM in the affected leg. Passive ROM should be performed with extreme caution and may be best avoided when assessing older patients. Observing the patient's active ROM is more accurate and safe than asking the patient to lift weights with her legs.

A patient is to receive methylprednisolone (Solu-Medrol) 100 mg. The label on the medication states: methylprednisolone 125 mg in 2 mL. How many milliliters will the nurse administer?

ANS: 1.6 A concentration of 125 mg in 2 mL will result in 100 mg in 1.6 mL. DIF: Cognitive Level: Apply (application) REF: 1179 TOP: Nursing Process: Implementation MSC: NCLEX: Physiological Integrity

A patient with osteomyelitis is to receive vancomycin (Vancocin) 500 mg IV every 6 hours. The vancomycin is diluted in 100 mL of normal saline and needs to be administered over 1 hour. The nurse will set the IV pump for how many milliliters per minute? (Round to the nearest hundredth.)

ANS: 1.67 To administer 100 mL in 60 minutes, the IV pump will need to provide 1.67 mL/min. DIF: Cognitive Level: Understand (comprehension) REF: 1497 TOP: Nursing Process: Implementation MSC: NCLEX: Physiological Integrity

A patient who is having an acute exacerbation of multiple sclerosis has a prescription for methylprednisolone (Solu-Medrol) 150 mg IV. The label on the vial reads: methylprednisolone 125 mg in 2 mL. How many mL will the nurse administer?

ANS: 2.4 With a concentration of 125 mg/2 mL, the nurse will need to administer 2.4 mL to obtain 150 mg of methylprednisolone. DIF: Cognitive Level: Understand (comprehension) REF: 1386 TOP: Nursing Process: Implementation MSC: NCLEX: Physiological Integrity

A patient in the oliguric phase after an acute kidney injury has had a 250-mL urine output and an emesis of 100 mL in the past 24 hours. What is the patient's fluid restriction for the next 24 hours?

ANS: 950 mL The general rule for calculating fluid restrictions is to add all fluid losses for the previous 24 hours, plus 600 mL for insensible losses: (250 + 100 + 600 = 950 mL). DIF: Cognitive Level: Understand (comprehension) REF: 1073 TOP: Nursing Process: Implementation MSC: NCLEX: Physiological Integrity

In which order will the nurse implement these interprofessional interventions prescribed for a patient admitted with acute osteomyelitis with a temperature of 101.2° F? (Put a comma and a space between each answer choice [A, B, C, D].) a. Obtain blood cultures from two sites. b. Administer dose of gentamicin 60 mg IV. c. Send to radiology for computed tomography (CT) scan of right leg. d. Administer acetaminophen (Tylenol) now and every 4 hours PRN for fever.

ANS: A, B, D, C The highest treatment priority for possible osteomyelitis is initiation of antibiotic therapy, but cultures should be obtained before administration of antibiotics. Addressing the discomfort of the fever is the next highest priority. Because the purpose of the CT scan is to determine the extent of the infection, it can be done last. DIF: Cognitive Level: Analyze (analysis) REF: 1497 OBJ: Special Questions: Prioritization TOP: Nursing Process: Planning MSC: NCLEX: Physiological Integrity

In which order will the nurse take these steps to prepare NPH 20 units and regular insulin 2 units using the same syringe? a. Rotate NPH vial. b. Withdraw regular insulin. c. Withdraw 20 units of NPH. d. Inject 20 units of air into NPH vial. e. Inject 2 units of air into regular insulin vial.

ANS: A, D, E, B, C

A 63-yr-old patient who began experiencing right arm and leg weakness is admitted to the emergency department. In which order will the nurse implement these actions included in the stroke protocol? (Put a comma and a space between each answer choice [A, B, C, D].) a. Obtain CT scan without contrast. b. Infuse tissue plasminogen activator (tPA). c. Administer oxygen to keep O2 saturation >95%. d. Use National Institute of Health Stroke Scale to assess patient.

ANS: C, D, A, B The initial actions should be those that help with airway, breathing, and circulation. Baseline neurologic assessments should be done next. A CT scan will be needed to rule out hemorrhagic stroke before tPA can be administered. DIF: Cognitive Level: Analyze (analysis) REF: 1354 OBJ: Special Questions: Prioritization TOP: Nursing Process: Implementation MSC: NCLEX: Physiological Integrity

A patient with neurogenic shock after a spinal cord injury is to receive lactated Ringer's solution 400 mL over 20 minutes. When setting the IV pump to deliver the IV fluid, the nurse will set the rate at how many milliliters per hour?

ANS: 1200 To administer 400 mL in 20 minutes, the nurse will need to set the pump to run at 1200 mL/hour. DIF: Cognitive Level: Understand (comprehension)

A 198-lb patient is to receive a dobutamine infusion at 5 mcg/kg/min. The label on the infusion bag states: dobutamine 250 mg in 250 mL of normal saline. When setting the infusion pump, the nurse will set the infusion rate at how many milliliters per hour?

ANS: 27 To administer the dobutamine at the prescribed rate of 5 mcg/kg/min from a concentration of 250 mg in 250 mL, the nurse will need to infuse 27 mL/hr. DIF: Cognitive Level: Apply (application)

1. A patient's vital signs are pulse 90, respirations 24, and BP 128/64 mm Hg, and cardiac output is 4.7 L/min. The patient's stroke volume is _____ mL. (Round to the nearest whole number.)

ANS: 52 Stroke volume = Cardiac output/heart rate 52 mL = (4.7 L x 1000 mL/L)/90

An unconscious patient with a traumatic head injury has a blood pressure of 130/76 mm Hg and an intracranial pressure (ICP) of 20 mm Hg. The nurse will calculate the cerebral perfusion pressure (CPP) as ____ mm Hg.

ANS: 74 Calculate the CPP: (CPP = Mean arterial pressure [MAP] - ICP). MAP = DBP + 1/3 (Systolic blood pressure [SBP] - Diastolic blood pressure [DBP]). The MAP is 94. The CPP is 74. DIF: Cognitive Level: Apply (application)

The nurse is caring for a patient who has an intraaortic balloon pump (IABP) after a massive heart attack. When assessing the patient, the nurse notices blood backing up into the IABP catheter. In which order should the nurse take the following actions? (Put a comma and a space between each answer choice [A, B, C, D].) a. Confirm that the IABP console has turned off. b. Assess the patient's vital signs and orientation. c. Obtain supplies for insertion of a new IABP catheter. d. Notify the health care provider of the IABP malfunction.

ANS: A, B, D, C Blood in the IABP catheter indicates a possible tear in the balloon. The console should shut off automatically to prevent complications such as air embolism. Next, the nurse will assess the patient and communicate with the health care provider about the patient's assessment and the IABP problem. Finally, supplies for insertion of a new IABP catheter may be needed based on the patient assessment and the decision of the health care provider. DIF: Cognitive Level: Analyze (analysis)

The following four patients arrive in the emergency department (ED) after a motor vehicle collision. In which order should the nurse assess them? (Put a comma and a space between each answer choice [A, B, C, D, E].) a. A 74-yr-old patient with palpitations and chest pain b. A 43-yr-old patient complaining of 7/10 abdominal pain c. A 21-yr-old patient with multiple fractures of the face and jaw d. A 37-yr-old patient with a misaligned lower left leg with intact pulses

ANS: C, A, B, D The highest priority is to assess the 21-yr-old patient for airway obstruction, which is the most life-threatening injury. The 74-yr-old patient may have chest pain from cardiac ischemia and should be assessed and have diagnostic testing for this pain. The 43-yr-old patient may have abdominal trauma or bleeding and should be seen next to assess circulatory status. The 37-yr-old patient appears to have a possible fracture of the left leg and should be seen soon, but this patient has the least life-threatening injury. DIF: Cognitive Level: Analyze (analysis)

In which order will the nurse perform the following actions when caring for a patient with possible C5 spinal cord trauma who is admitted to the emergency department? (Put a comma and a space between each answer choice [A, B, C, D, E].) a. Infuse normal saline at 150 mL/hr. b. Monitor cardiac rhythm and blood pressure. c. Administer O2 using a nonrebreather mask. d. Immobilize the patient's head, neck, and spine. e. Transfer the patient to radiology for spinal computed tomography (CT).

ANS: D, C, B, A, E The first action should be to prevent further injury by stabilizing the patient's spinal cord if the patient does not have penetrating trauma. Maintenance of oxygenation by administration of 100% O2 is the second priority. Because neurogenic shock is a possible complication, monitoring of heart rhythm and BP are indicated followed by infusing normal saline for volume replacement. A CT scan to determine the extent and level of injury is needed once initial assessment and stabilization are accomplished. DIF: Cognitive Level: Analyze (analysis)

When assisting with oral intubation of a patient who is having respiratory distress, in which order will the nurse take these actions? (Put a comma and a space between each answer choice [A, B, C, D, E].) a. Obtain a portable chest-x-ray. b. Position the patient in the supine position. c. Inflate the cuff of the endotracheal tube after insertion. d. Attach an end-tidal CO2 detector to the endotracheal tube. e. Oxygenate the patient with a bag-valve-mask device for several minutes.

ANS: E, B, C, D, A The patient is pre-oxygenated with a bag-valve-mask system for 3 to 5 minutes before intubation and then placed in a supine position. After the intubation, the cuff on the endotracheal tube is inflated to occlude and protect the airway. Tube placement is assessed first with an end-tidal CO2 sensor and then with chest x-ray examination. DIF: Cognitive Level: Analyze (analysis)

The health care provider orders the following interventions for a 67-kg patient who has septic shock with a blood pressure of 70/42 mm Hg and O2 saturation of 90% on room air. In which order will the nurse implement the actions? (Put a comma and a space between each answer choice [A, B, C, D, E].) a. Give vancomycin 1 g IV. b. Obtain blood and urine cultures c. Start norepinephrine 0.5 mcg/min. d. Infuse normal saline 2000 mL over 30 minutes. e. Titrate oxygen administration to keep O2 saturation above 95%.

ANS: E, D, C, B, A The initial action for this hypotensive and hypoxemic patient should be to improve the O2 saturation, followed by infusion of IV fluids and vasopressors to improve perfusion. Cultures should be obtained before giving antibiotics. DIF: Cognitive Level: Analyze (analysis)

Which action will the nurse include in the plan of care for a patient with newly diagnosed ankylosing spondylitis? a. Advise the patient to sleep on the back with a flat pillow. b. Emphasize that application of heat may worsen symptoms. c. Schedule annual laboratory assessment for the HLA-B27 antigen. d. Assist patient to choose physical activities that involve spinal flexion.

ANS: A Because ankylosing spondylitis results in flexion deformity of the spine, postures that extend the spine (e.g., sleeping on the back and with a flat pillow) are recommended. HLA-B27 antigen is assessed for initial diagnosis but is not needed annually. To counteract the development of flexion deformities, the patient should choose activities that extend the spine, such as swimming. Heat application is used to decrease localized pain. DIF: Cognitive Level: Apply (application) REF: 1537 TOP: Nursing Process: Planning MSC: NCLEX: Physiological Integrity

The nurse is caring for a 63-yr-old with a possible pituitary tumor who is scheduled for a computed tomography scan with contrast. Which information about the patient is important to discuss with the health care provider before the test? a. History of renal insufficiency b. Complains of chronic headache c. Recent bilateral visual field loss d. Blood glucose level of 134 mg/dL

ANS: A Because contrast media may cause acute kidney injury in patients with poor renal function, the health care provider will need to prescribe therapies such as IV fluids to prevent this complication. The other findings are consistent with the patient's diagnosis of a pituitary tumor. DIF: Cognitive Level: Apply (application) REF: 1115 TOP: Nursing Process: Assessment MSC: NCLEX: Physiological Integrity

A patient with severe kyphosis is scheduled for dual-energy x-ray absorptiometry (DXA) testing. The nurse will plan to a. explain the procedure. b. start an IV line for contrast medium injection. c. give an oral sedative 60 to 90 minutes before the procedure. d. screen the patient for allergies to shellfish or iodine products.

ANS: A DXA testing is painless and noninvasive. No IV access is necessary. Contrast medium is not used. Shellfish or iodine allergies are not a concern with DXA testing. Because the procedure is painless, no antianxiety medications are required. DIF: Cognitive Level: Apply (application) REF: 1458 TOP: Nursing Process: Implementation MSC: NCLEX: Physiological Integrity

A patient who has elevated blood urea nitrogen (BUN) and serum creatinine levels is scheduled for a renal arteriogram. Which bowel preparation order would the nurse question for this patient? a. Fleet enema c. Senna/docusate (Senokot-S) b. Tap-water enema d. Bisacodyl (Dulcolax) tablets

ANS: A High-phosphate enemas, such as Fleet enemas, should be avoided in patients with elevated BUN and creatinine because phosphate cannot be excreted by patients with renal failure. The other medications for bowel evacuation are more appropriate. DIF: Cognitive Level: Apply (application) REF: 1024 TOP: Nursing Process: Implementation MSC: NCLEX: Physiological Integrity

Which patient action indicates a good understanding of the nurse's teaching about the use of an insulin pump? a. The patient programs the pump for an insulin bolus after eating. b. The patient changes the location of the insertion site every week. c. The patient takes the pump off at bedtime and starts it again each morning. d. The patient plans a diet with more calories than usual when using the pump.

ANS: A In addition to the basal rate of insulin infusion, the patient will adjust the pump to administer a bolus after each meal, with the dosage depending on the oral intake. The insertion site should be changed every 2 or 3 days. There is more flexibility in diet and exercise when an insulin pump is used, but it does not provide for consuming a higher calorie diet. The pump will deliver a basal insulin rate 24 hours a day. DIF: Cognitive Level: Apply (application) REF: 1129 TOP: Nursing Process: Evaluation MSC: NCLEX: Health Promotion and Maintenance

The nurse teaching a support group of women with rheumatoid arthritis (RA) about how to manage activities of daily living suggests they should a. avoid activities requiring repetitive use of the same muscles and joints. b. protect the knee joints by sleeping with a small pillow under the knees. c. stand rather than sit when performing daily household and yard chores. d. strengthen small hand muscles by wringing out sponges or washcloths.

ANS: A Patients are advised to avoid repetitious movements. Sitting during household chores is recommended to decrease stress on joints. Wringing water out of sponges would increase joint stress. Patients are encouraged to position joints in the extended (neutral) position. Sleeping with a pillow behind the knees would decrease the ability of the knee to extend and also decrease knee range of motion. DIF: Cognitive Level: Apply (application) REF: 1524 TOP: Nursing Process: Implementation MSC: NCLEX: Physiological Integrity

Sodium polystyrene sulfonate (Kayexalate) is ordered for a patient with hyperkalemia. Before administering the medication, the nurse should assess the a. bowel sounds. b. blood glucose. c. blood urea nitrogen (BUN). d. level of consciousness (LOC).

ANS: A Sodium polystyrene sulfonate (Kayexalate) should not be given to a patient with a paralytic ileus (as indicated by absent bowel sounds) because bowel necrosis can occur. The BUN and creatinine, blood glucose, and LOC would not affect the nurse's decision to give the medication. DIF: Cognitive Level: Apply (application) REF: 1080 TOP: Nursing Process: Assessment MSC: NCLEX: Physiological Integrity

The nurse has administered 4 oz of orange juice to an alert patient whose blood glucose was 62 mg/dL. Fifteen minutes later, the blood glucose is 67 mg/dL. Which action should the nurse take next? a. Give the patient 4 to 6 oz more orange juice. b. Administer the PRN glucagon (Glucagon) 1 mg IM. c. Have the patient eat some peanut butter with crackers. d. Notify the health care provider about the hypoglycemia.

ANS: A The "rule of 15" indicates that administration of quickly acting carbohydrates should be done two or three times for a conscious patient whose glucose remains less than 70 mg/dL before notifying the health care provider. More complex carbohydrates and fats may be used after the glucose has stabilized. Glucagon should be used if the patient's level of consciousness decreases so that oral carbohydrates can no longer be given. DIF: Cognitive Level: Analyze (analysis) REF: 1146 OBJ: Special Questions: Prioritization TOP: Nursing Process: Implementation MSC: NCLEX: Physiological Integrity

The nurse observes a patient ambulating in the hospital hall when the patient's arms and legs suddenly jerk and the patient falls to the floor. The nurse will first a. assess the patient for a possible injury. b. give the scheduled divalproex (Depakote). c. document the timing and description of the seizure. d. notify the patient's health care provider about the seizure.

ANS: A The patient who has had a myoclonic seizure and fall is at risk for head injury and should first be evaluated and treated for this possible complication. Documentation of the seizure, notification of the health care provider, and administration of antiseizure medications are also appropriate actions, but the initial action should be assessment for injury. DIF: Cognitive Level: Analyze (analysis) REF: 1376 OBJ: Special Questions: Prioritization TOP: Nursing Process: Implementation MSC: NCLEX: Physiological Integrity

A 72-yr-old patient with a history of benign prostatic hyperplasia (BPH) is admitted with acute urinary retention and elevated blood urea nitrogen (BUN) and creatinine levels. Which prescribed therapy should the nurse implement first? a. Insert urethral catheter. b. Obtain renal ultrasound. c. Draw a complete blood count. d. Infuse normal saline at 50 mL/hour.

ANS: A The patient's elevation in BUN is most likely associated with hydronephrosis caused by the acute urinary retention, so the insertion of a retention catheter is the first action to prevent ongoing postrenal failure for this patient. The other actions also are appropriate but should be implemented after the retention catheter. DIF: Cognitive Level: Analyze (analysis) REF: 1071 OBJ: Special Questions: Prioritization TOP: Nursing Process: Implementation MSC: NCLEX: Physiological Integrity

A 68-yr-old male patient is brought to the emergency department (ED) by ambulance after being found unconscious on the bathroom floor by his spouse. Which action will the nurse take first? a. Check oxygen saturation. b. Assess pupil reaction to light. c. Palpate the head for injuries d. Verify Glasgow Coma Scale (GCS) score.

ANS: A Airway patency and breathing are the most vital functions and should be assessed first. The neurologic assessments should be accomplished next and additional assessment after that. DIF: Cognitive Level: Analyze (analysis)

Which patient is most appropriate for the intensive care unit (ICU) charge nurse to assign to a registered nurse (RN) who has floated from the medical unit? a. A 45-yr-old patient receiving IV antibiotics for meningococcal meningitis b. A 35-yr-old patient with intracranial pressure (ICP) monitoring after a head injury c. A 25-yr-old patient admitted with a skull fracture and craniotomy the previous day d. A 55-yr-old patient who has increased intracranial pressure (ICP) and is receiving hyperventilation therapy

ANS: A An RN who works on a medical unit will be familiar with administration of IV antibiotics and with meningitis. The patient recovering from a craniotomy, the patient with an ICP monitor, and the patient on a ventilator should be assigned to an RN familiar with the care of critically ill patients. DIF: Cognitive Level: Analyze (analysis)

Which finding for a patient who has hypothyroidism and hypertension indicates that the nurse should contact the health care provider before administering levothyroxine (Synthroid)? a. Increased thyroxine (T4) level b. Blood pressure 112/62 mm Hg c. Distant and difficult to hear heart sounds d. Elevated thyroid stimulating hormone level

ANS: A An increased thyroxine level indicates the levothyroxine dose needs to be decreased. The other data are consistent with hypothyroidism and the nurse should administer the levothyroxine. DIF: Cognitive Level: Apply (application) REF: 1169 TOP: Nursing Process: Implementation MSC: NCLEX: Physiological Integrity

A patient will need vascular access for hemodialysis. Which statement by the nurse accurately describes an advantage of a fistula over a graft? a. A fistula is much less likely to clot. b. A fistula increases patient mobility. c. A fistula can accommodate larger needles. d. A fistula can be used sooner after surgery.

ANS: A Arteriovenous (AV) fistulas are much less likely to clot than grafts, although it takes longer for them to mature to the point where they can be used for dialysis. The choice of an AV fistula or a graft does not have an impact on needle size or patient mobility. DIF: Cognitive Level: Understand (comprehension) REF: 1088 TOP: Nursing Process: Implementation MSC: NCLEX: Physiological Integrity

Which question during the assessment of a patient who has diabetes will help the nurse identify autonomic neuropathy? a. "Do you feel bloated after eating?" b. "Have you seen any skin changes?" c. "Do you need to increase your insulin dosage when you are stressed?" d. "Have you noticed any painful new ulcerations or sores on your feet?"

ANS: A Autonomic neuropathy can cause delayed gastric emptying, which results in a bloated feeling for the patient. The other questions are also appropriate to ask but would not help in identifying autonomic neuropathy. DIF: Cognitive Level: Apply (application) REF: 1150 TOP: Nursing Process: Assessment MSC: NCLEX: Physiological Integrity

A patient hospitalized with a new diagnosis of Guillain-Barré syndrome has numbness and weakness of both feet. The nurse will anticipate teaching the patient about a. infusion of immunoglobulin b. intubation and mechanical ventilation. c. administration of corticosteroid drugs. d. insertion of a nasogastric (NG) feeding tube.

ANS: A Because Guillain-Barré syndrome is in the earliest stages (as evidenced by the symptoms), use of high-dose immunoglobulin is appropriate to reduce the extent and length of symptoms. Mechanical ventilation and tube feedings may be used later in the progression of the syndrome but are not needed now. Corticosteroid use is not helpful in reducing the duration or symptoms of the syndrome. DIF: Cognitive Level: Apply (application)

To assist an older patient with diabetes to engage in moderate daily exercise, which action is most important for the nurse to take? a. Determine what types of activities the patient enjoys. b. Remind the patient that exercise improves self-esteem. c. Teach the patient about the effects of exercise on glucose level. d. Give the patient a list of activities that are moderate in intensity.

ANS: A Because consistency with exercise is important, assessment for the types of exercise that the patient finds enjoyable is the most important action by the nurse in ensuring adherence to an exercise program. The other actions may be helpful but are not the most important in improving compliance. DIF: Cognitive Level: Analyze (analysis) REF: 1134 OBJ: Special Questions: Prioritization TOP: Nursing Process: Implementation MSC: NCLEX: Physiological Integrity

A patient has a tumor in the cerebellum. The nurse will plan interventions to a. prevent falls. c. avoid aspiration. b. stabilize mood. d. improve memory.

ANS: A Because functions of the cerebellum include coordination and balance, the patient with dysfunction is at risk for falls. The cerebellum does not affect memory, mood, or swallowing ability. DIF: Cognitive Level: Apply (application) REF: 1298 TOP: Nursing Process: Planning MSC: NCLEX: Physiological Integrity

After change-of-shift report, which patient should the nurse assess first? a. Patient with myasthenia gravis who is reporting increased muscle weakness b. Patient with a bilateral headache described as "like a band around my head" c. Patient with seizures who is scheduled to receive a dose of phenytoin (Dilantin) d. Patient with Parkinson's disease who has developed cogwheel rigidity of the arms

ANS: A Because increased muscle weakness may indicate the onset of a myasthenic crisis, the nurse should assess this patient first. The other patients should also be assessed but do not appear to need immediate nursing assessments or actions to prevent life-threatening complications. DIF: Cognitive Level: Analyze (analysis) REF: 1393 OBJ: Special Questions: Prioritization | Special Questions: Multiple Patients TOP: Nursing Process: Planning MSC: NCLEX: Safe and Effective Care Environment

Which information about a patient who is receiving cisatracurium (Nimbex) to prevent asynchronous breathing with the positive pressure ventilator requires action by the nurse? a. No sedative has been ordered for the patient. b. The patient does not respond to verbal stimulation. c. There is no cough or gag reflex when the patient is suctioned. d. The patient's oxygen saturation remains between 90% to 93%.

ANS: A Because neuromuscular blockade is extremely anxiety provoking, it is essential that patients who are receiving neuromuscular blockade receive concurrent sedation and analgesia. Absence of response to stimuli is expected in patients receiving neuromuscular blockade. The O2 saturation is adequate. DIF: Cognitive Level: Apply (application)

The urgent care center protocol for tick bites includes the following actions. Which action will the nurse take first when caring for a patient with a tick bite? a. Use tweezers to remove any remaining ticks. b. Check the vital signs, including temperature. c. Give doxycycline (Vibramycin) 100 mg orally. d. Obtain information about recent outdoor activities.

ANS: A Because neurotoxic venom is released as long as the tick is attached to the patient, the initial action should be to remove any ticks using tweezers or forceps. The other actions are also appropriate, but the priority is to minimize venom release. DIF: Cognitive Level: Analyze (analysis)

A patient with septic shock has a BP of 70/46 mm Hg, pulse of 136 beats/min, respirations of 32 breaths/min, temperature of 104°F, and blood glucose of 246 mg/dL. Which intervention ordered by the health care provider should the nurse implement first? a. Give normal saline IV at 500 mL/hr. b. Give acetaminophen (Tylenol) 650 mg rectally. c. Start insulin drip to maintain blood glucose at 110 to 150 mg/dL. d. Start norepinephrine to keep systolic blood pressure above 90 mm Hg.

ANS: A Because of the decreased preload associated with septic shock, fluid resuscitation is the initial therapy. The other actions also are appropriate, and should be initiated quickly as well. DIF: Cognitive Level: Analyze (analysis)

The nurse is caring for a patient who has septic shock. Which assessment finding is most important for the nurse to report to the health care provider? a. Skin cool and clammy c. Blood pressure of 92/56 mm Hg b. Heart rate of 118 beats/min d. O2 saturation of 93% on room air

ANS: A Because patients in the early stage of septic shock have warm and dry skin, the patient's cool and clammy skin indicates that shock is progressing. The other information will also be reported, but does not indicate deterioration of the patient's status. DIF: Cognitive Level: Analyze (analysis)

A patient is admitted to the emergency department with possible renal trauma after an automobile accident. Which prescribed intervention will the nurse implement first? a. Check blood pressure and heart rate. b. Administer morphine sulfate 4 mg IV. c. Transport to radiology for an intravenous pyelogram. d. Insert a urethral catheter and obtain a urine specimen.

ANS: A Because the kidney is very vascular, the initial action with renal trauma will be assessment for bleeding and shock. The other actions are also important after the patient's cardiovascular status has been determined and stabilized.

A female patient being admitted with pneumonia has a history of neurogenic bladder as a result of a spinal cord injury. Which action will the nurse plan to take first? a. Ask about the usual urinary pattern and any measures used for bladder control. b. Assist the patient to the toilet at scheduled times to help ensure bladder emptying. c. Check the patient for urinary incontinence every 2 hours to maintain skin integrity. d. Use intermittent catheterization on a regular schedule to avoid the risk of infection.

ANS: A Before planning any interventions, the nurse should complete the assessment and determine the patient's normal bladder pattern and the usual measures used by the patient at home. All the other responses may be appropriate, but until the assessment is complete, an individualized plan for the patient cannot be developed. DIF: Cognitive Level: Analyze (analysis) REF: 1021 OBJ: Special Questions: Prioritization TOP: Nursing Process: Planning MSC: NCLEX: Physiological Integrity

A patient with left knee pain is diagnosed with bursitis. The nurse will explain that bursitis is an inflammation of a. a fluid-filled sac found at some joints. b. a synovial membrane that lines the joint. c. the connective tissue joining bones within a joint. d. the fibrocartilage that acts as a shock absorber in the knee.

ANS: A Bursae are fluid-filled sacs that cushion joints and bony prominences. Fibrocartilage is a solid tissue that cushions some joints. Ligaments are connective tissue joining bones within a joint The synovial membrane lines many joints but is not a bursa. DIF: Cognitive Level: Understand (comprehension) REF: 1450 TOP: Nursing Process: Implementation MSC: NCLEX: Physiological Integrity

Which finding by the nurse when assessing a patient with Hashimoto's thyroiditis and a goiter will require the most immediate action? a. New-onset changes in the patient's voice b. Elevation in the patient's T3 and T4 levels c. Resting apical pulse rate 112 beats/minute d. Bruit audible bilaterally over the thyroid gland

ANS: A Changes in the patient's voice indicate that the goiter is compressing the laryngeal nerve and may lead to airway compression. The other findings will also be reported but are expected with Hashimoto's thyroiditis and do not require immediate action. DIF: Cognitive Level: Analyze (analysis) REF: 1163 OBJ: Special Questions: Prioritization TOP: Nursing Process: Assessment MSC: NCLEX: Physiological Integrity

Which nursing action can the nurse delegate to experienced unlicensed assistive personnel (UAP) who are working in the diabetic clinic? a. Measure the ankle-brachial index. b. Check for changes in skin pigmentation. c. Assess for unilateral or bilateral foot drop. d. Ask the patient about symptoms of depression. sing

ANS: A Checking systolic pressure at the ankle and brachial areas and calculating the ankle-brachial index is a procedure that can be done by UAP who have been trained in the procedure. The other assessments require more education and critical thinking and should be done by the registered nurse (RN). DIF: Cognitive Level: Apply (application) REF: 1152 OBJ: Special Questions: Delegation TOP: Nur

Which finding indicates to the nurse that the current therapies are effective for a patient with acute adrenal insufficiency? a. Increasing serum sodium levels b. Decreasing blood glucose levels c. Decreasing serum chloride levels d. Increasing serum potassium levels

ANS: A Clinical manifestations of Addison's disease include hyponatremia and an increase in sodium level indicates improvement. The other values indicate that treatment has not been effective. DIF: Cognitive Level: Apply (application) REF: 1178 TOP: Nursing Process: Evaluation MSC: NCLEX: Physiological Integrit

The nurse determines that colchicine has been effective for a patient with an acute attack of gout upon finding a. reduced joint pain. . b. increased urine output. c. elevated serum uric acid d. increased white blood cells (WBC).

ANS: A Colchicine reduces joint pain in 24 to 48 hours by decreasing inflammation. The recommended increase in fluid intake of 2 to 3 L/day during acute gout would increase urine output but would not indicate the effectiveness of colchicine. Elevated serum uric acid would result in increased symptoms. The WBC count might decrease with decreased inflammation but would not increase. DIF: Cognitive Level: Understand (comprehension) REF: 1533 TOP: Nursing Process: Evaluation MSC: NCLEX: Physiological Integrity

A female patient who had a stroke 24 hours ago has expressive aphasia. An appropriate nursing intervention to help the patient communicate is to a. ask questions that the patient can answer with "yes" or "no." b. develop a list of words that the patient can read and practice reciting. c. have the patient practice her facial and tongue exercises with a mirror. d. prevent embarrassing the patient by answering for her if she does not respond.

ANS: A Communication will be facilitated and less frustrating to the patient when questions that require a "yes" or "no" response are used. When the language areas of the brain are injured, the patient might not be able to read or recite words, which will frustrate the patient without improving communication. Expressive aphasia is caused by damage to the language areas of the brain, not by the areas that control the motor aspects of speech. The nurse should allow time for the patient to respond. DIF: Cognitive Level: Apply (application) REF: 1361 TOP: Nursing Process: Implementation MSC: NCLEX: Physiological Integrity

A patient with dermatomyositis is receiving long-term prednisone therapy. Which assessment finding by the nurse is important to report to the health care provider? a. The patient has painful hematuria. b. Acne is noted on the patient's face. c. Fasting blood glucose is 112 mg/dL. d. The patient has an increased appetite.

ANS: A Corticosteroid use is associated with an increased risk for infection, so the nurse should report the urinary tract symptoms immediately to the health care provider. The increase in blood glucose, increased appetite, and acne are also adverse effects of corticosteroid use but do not need diagnosis and treatment as rapidly as the probable urinary tract infection. DIF: Cognitive Level: Apply (application) REF: 1545 TOP: Nursing Process: Planning MSC: NCLEX: Physiological Integrity

10. A 72-year-old patient with kyphosis is scheduled for dual-energy x-ray absorptiometry (DXA) testing. The nurse will plan to a. explain the procedure. b. start an IV line for contrast medium injection. c. give an oral sedative 60 to 90 minutes before the procedure. d. screen the patient for allergies to shellfish or iodine products.

ANS: A DXA testing is painless and noninvasive. No IV access is necessary. Contrast medium is not used. Because the procedure is painless, no antianxiety medications are required.

A college athlete is seen in the clinic 6 weeks after a concussion. Which assessment information will the nurse collect to determine whether the patient is developing postconcussion syndrome? a. Short-term memory b. Muscle coordination c. Glasgow Coma Scale d. Pupil reaction to light

ANS: A Decreased short-term memory is one indication of postconcussion syndrome. The other data may be assessed but are not indications of postconcussion syndrome. DIF: Cognitive Level: Apply (application)

After the nurse has taught a 28-yr-old with fibromyalgia, which statement by the patient indicates a good understanding of effective self-management? a. "I will need to stop drinking so much coffee and soda." b. "I am going to join a soccer team to get more exercise." c. "I will call the doctor every time my symptoms get worse." d. "I should avoid using over-the-counter medications for pain."

ANS: A Dietitians frequently suggest patients with fibromyalgia limit their intake of caffeine and sugar because these substances are muscle irritants. Mild exercise such as walking is recommended for patients with fibromyalgia, but vigorous exercise is likely to make symptoms worse. Because symptoms may fluctuate from day to day, the patient should be able to adapt the regimen independently rather than calling the provider whenever symptoms get worse. Over-the-counter medications such as ibuprofen and acetaminophen are frequently used for symptom management. DIF: Cognitive Level: Apply (application) REF: 1548 TOP: Nursing Process: Evaluation MSC: NCLEX: Physiological Integrity

A patient who takes multiple medications develops acute gouty arthritis. The nurse will consult with the health care provider before giving the prescribed dose of a. sertraline (Zoloft). b. famotidine (Pepcid). c. hydrochlorothiazide. d. oxycodone (Roxicodone).

ANS: A Diuretic use increases uric acid levels and can precipitate gout attacks. The other medications are safe to administer. DIF: Cognitive Level: Apply (application) REF: 1532 TOP: Nursing Process: Implementation MSC: NCLEX: Physiological Integrity

A woman who has multiple sclerosis (MS) asks the nurse about risks associated with pregnancy. Which response by the nurse is accurate? a. "MS symptoms may be worse after the pregnancy." b. "Women with MS frequently have premature labor." c. "MS is associated with an increased risk for congenital defects." d. "Symptoms of MS are likely to become worse during pregnancy."

ANS: A During the postpartum period, women with MS are at greater risk for exacerbation of symptoms. There is no increased risk for congenital defects in infants born of mothers with MS. Symptoms of MS may improve during pregnancy. Onset of labor is not affected by MS. DIF: Cognitive Level: Understand (comprehension) REF: 1384 TOP: Nursing Process: Implementation MSC: NCLEX: Health Promotion and Maintenance

A nurse is caring for a patient who is orally intubated and receiving mechanical ventilation. To decrease the risk for ventilator-associated pneumonia, which action will the nurse include in the plan of care? a. Elevate head of bed to 30 to 45 degrees. b. Give enteral feedings at no more than 10 mL/hr. c. Suction the endotracheal tube every 2 to 4 hours. d. Limit the use of positive end-expiratory pressure.

ANS: A Elevation of the head decreases the risk for aspiration. Positive end-expiratory pressure is frequently needed to improve oxygenation in patients receiving mechanical ventilation. Suctioning should be done only when the patient assessment indicates that it is necessary. Enteral feedings should provide adequate calories for the patient's high energy needs. DIF: Cognitive Level: Apply (application)

The following interventions are ordered by the health care provider for a patient who has respiratory distress and syncope after eating strawberries. Which will the nurse complete first? a. Give epinephrine. b. Administer diphenhydramine. c. Start continuous ECG monitoring. d. Draw blood for complete blood count (CBC)

ANS: A Epinephrine rapidly causes peripheral vasoconstriction, dilates the bronchi, and blocks the effects of histamine and reverses the vasodilation, bronchoconstriction, and histamine release that cause the symptoms of anaphylaxis. The other interventions are also appropriate but would not be the first ones completed. DIF: Cognitive Level: Analyze (analysis)

A 78-kg patient with septic shock has a pulse rate of 120 beats/min with low central venous pressure and pulmonary artery wedge pressure. Urine output has been 30 mL/hr for the past 3 hours. Which order by the health care provider should the nurse question? a. Administer furosemide (Lasix) 40 mg IV. b. Increase normal saline infusion to 250 mL/hr. c. Give hydrocortisone (Solu-Cortef) 100 mg IV. d. Titrate norepinephrine to keep systolic blood pressure (BP) above 90 mm Hg.

ANS: A Furosemide will lower the filling pressures and renal perfusion further for the patient with septic shock. Patients in septic shock require large amounts of fluid replacement. If the patient remains hypotensive after initial volume resuscitation with minimally 30 mL/kg, vasopressors such as norepinephrine may be added. IV corticosteroids may be considered for patients in septic shock who cannot maintain an adequate BP with vasopressor therapy despite fluid resuscitation. DIF: Cognitive Level: Apply (application)

A patient with acute respiratory distress syndrome (ARDS) and acute kidney injury has the following drugs ordered. Which drug should the nurse discuss with the health care provider before giving? a. gentamicin 60 mg IV b. pantoprazole (Protonix) 40 mg IV c. sucralfate (Carafate) 1 g per nasogastric tube d. methylprednisolone (Solu-Medrol) 60 mg IV

ANS: A Gentamicin, which is one of the aminoglycoside antibiotics, is potentially nephrotoxic, and the nurse should clarify the drug and dosage with the health care provider before administration. The other drugs are appropriate for the patient with ARDS. DIF: Cognitive Level: Apply (application)

The nurse providing care for a patient who has an adrenocortical adenoma causing hyperaldosteronism should a. monitor the blood pressure every 4 hours. b. elevate the patient's legs to relieve edema. c. monitor blood glucose level every 4 hours. d. order the patient a potassium-restricted diet.

ANS: A Hypertension caused by sodium retention is a common complication of hyperaldosteronism. Hyperaldosteronism does not cause an elevation in blood glucose. The patient will be hypokalemic and require potassium supplementation before surgery. Edema does not usually occur with hyperaldosteronism. DIF: Cognitive Level: Apply (application) REF: 1180 TOP: Nursing Process: Implementation MSC: NCLEX: Physiological Integrity

A 27-yr-old patient admitted with diabetic ketoacidosis (DKA) has a serum glucose level of 732 mg/dL and serum potassium level of 3.1 mEq/L. Which action prescribed by the health care provider should the nurse take first? a. Place the patient on a cardiac monitor. b. Administer IV potassium supplements. c. Ask the patient about home insulin doses. d. Start an insulin infusion at 0.1 units/kg/hr. MSC: NCLEX: Physiological Integrity

ANS: A Hypokalemia can lead to potentially fatal dysrhythmias such as ventricular tachycardia and ventricular fibrillation, which would be detected with electrocardiogram (ECG) monitoring. Because potassium must be infused over at least 1 hour, the nurse should initiate cardiac monitoring before infusion of potassium. Insulin should not be administered without cardiac monitoring because insulin infusion will further decrease potassium levels. Discussion of home insulin and possible causes can wait until the patient is stabilized. DIF: Cognitive Level: Analyze (analysis) REF: 1146 OBJ: Special Questions: Prioritization TOP: Nursing Process: Implementation

A patient with carotid atherosclerosis asks the nurse to describe a carotid endarterectomy. Which response by the nurse is accurate? a. "The obstructing plaque is surgically removed from inside an artery in the neck." b. "The diseased portion of the artery in the brain is replaced with a synthetic graft." c. "A wire is threaded through an artery in the leg to the clots in the carotid artery, and the clots are removed." d. "A catheter with a deflated balloon is positioned at the narrow area, and the balloon is inflated to flatten the plaque."

ANS: A In a carotid endarterectomy, the carotid artery is incised, and the plaque is removed. The response beginning, "The diseased portion of the artery in the brain is replaced" describes an arterial graft procedure. The answer beginning, "A catheter with a deflated balloon is positioned at the narrow area" describes an angioplasty. The final response beginning, "A wire is threaded through the artery" describes the mechanical embolus removal in cerebral ischemia (MERCI) procedure. DIF: Cognitive Level: Understand (comprehension) REF: 1353 TOP: Nursing Process: Implementation MSC: NCLEX: Physiological Integrity

A young adult patient is being seen in the clinic with increased secretion of the anterior pituitary hormones. The nurse would expect the laboratory test results to show a. increased urinary cortisol. . b. decreased serum thyroxine. c. elevated serum aldosterone levels d. low urinary catecholamines excretion.

ANS: A Increased secretion of adrenocorticotropic hormone (ACTH) by the anterior pituitary gland will lead to an increase in serum and urinary cortisol levels. An increase, rather than a decrease, in thyroxine level would be expected with increased secretion of thyroid-stimulating hormone (TSH) by the anterior pituitary. Aldosterone and catecholamine levels are not controlled by the anterior pituitary. DIF: Cognitive Level: Understand (comprehension) REF: 1107 TOP: Nursing Process: Assessment MSC: NCLEX: Physiological Integrity

When admitting a patient with possible respiratory failure and a high PaCO2, which assessment information should be immediately reported to the health care provider? a. The patient is very somnolent. b. The patient complains of weakness. c. The patient's blood pressure is 164/98. d. The patient's oxygen saturation is 90%.

ANS: A Increasing somnolence will decrease the patient's respiratory rate and further increase the PaCO2 and respiratory failure. Rapid action is needed to prevent respiratory arrest. An SpO2 of 90%, weakness, and elevated blood pressure all require ongoing monitoring but are not indicators of possible impending respiratory arrest. DIF: Cognitive Level: Analyze (analysis)

The nurse advises a patient with myasthenia gravis (MG) to a. perform physically demanding activities early in the day. b. anticipate the need for weekly plasmapheresis treatments. c. do frequent weight-bearing exercise to prevent muscle atrophy. d. protect the extremities from injury due to poor sensory perception.

ANS: A Muscles are generally strongest in the morning, and activities involving muscle activity should be scheduled then. Plasmapheresis is not routinely scheduled but is used for myasthenia crisis or for situations in which corticosteroid therapy must be avoided. There is no decrease in sensation with MG, and muscle atrophy does not occur because although there is muscle weakness, they are still used. DIF: Cognitive Level: Apply (application) REF: 1393 TOP: Nursing Process: Implementation MSC: NCLEX: Physiological Integrity

A patient who is receiving positive pressure ventilation is scheduled for a spontaneous breathing trial (SBT). Which finding by the nurse is most likely to result in postponing the SBT? a. New ST segment elevation is noted on the cardiac monitor. b. Enteral feedings are being given through an orogastric tube. c. Scattered rhonchi are heard when auscultating breath sounds. d. hydromorphone (Dilaudid) is being used to treat postoperative pain.

ANS: A Myocardial ischemia is a contraindication for ventilator weaning. The ST segment elevation is an indication that weaning should be postponed until further investigation and/or treatment for myocardial ischemia can be done. Ventilator weaning can proceed when opioids are used for pain management, abnormal lung sounds are present, or enteral feedings are being used. DIF: Cognitive Level: Apply (application)

A female patient is scheduled for an oral glucose tolerance test. Which information from the patient's health history is important for the nurse to communicate to the health care provider regarding this test? a. The patient uses oral contraceptives. b. The patient runs several days a week. c. The patient has been pregnant three times. d. The patient has a family history of diabetes. ty

ANS: A Oral contraceptive use may falsely elevate oral glucose tolerance test (OGTT) values. Exercise and a family history of diabetes both can affect blood glucose but will not lead to misleading information from the OGTT. History of previous pregnancies may provide informational about gestational glucose tolerance but will not lead to misleading information from the OGTT. DIF: Cognitive Level: Apply (application) REF: 1124 TOP: Nursing Process: Assessment MSC: NCLEX: Physiological Integri

Which action will the nurse take when caring for a patient with osteomalacia? a. Teach about the use of vitamin D supplements. b. Educate about the need for weight-bearing exercise. c. Discuss the use of medications such as bisphosphonates. d. Emphasize the importance of sunscreen use when outside.

ANS: A Osteomalacia is caused by inadequate intake or absorption of vitamin D. Weight-bearing exercise and bisphosphonate administration may be used for osteoporosis but will not be beneficial for osteomalacia. Because ultraviolet light is needed for the body to synthesize vitamin D, the patient might be taught that 20 minutes a day of sun exposure is beneficial. DIF: Cognitive Level: Apply (application) REF: 1510 TOP: Nursing Process: Implementation MSC: NCLEX: Physiological Integrity

The nurse determines that additional instruction is needed for a patient with chronic syndrome of inappropriate antidiuretic hormone (SIADH) when the patient makes which statement? a. "I need to shop for foods low in sodium and avoid adding salt to food." b. "I should weigh myself daily and report any sudden weight loss or gain." c. "I need to limit my fluid intake to no more than 1 quart of liquids a day." d. "I should eat foods high in potassium because diuretics cause potassium loss."

ANS: A Patients with SIADH are at risk for hyponatremia, and a sodium supplement may be prescribed. The other patient statements are correct and indicate successful teaching has occurred. DIF: Cognitive Level: Apply (application) REF: 1160 TOP: Nursing Process: Evaluation MSC: NCLEX: Physiological Integrity

Which action will the nurse include in the plan of care for a patient who has a cauda equina spinal cord injury? a. Catheterize patient every 3 to 4 hours. b. Assist patient to ambulate 4 times daily. c. Administer medications to reduce bladder spasm. d. Stabilize the neck when repositioning the patient.

ANS: A Patients with cauda equina syndrome have areflexic bladder, and intermittent catheterization will be used for emptying the bladder. Because the bladder is flaccid, antispasmodic medications will not be used. The legs are flaccid with cauda equina syndrome, and the patient will be unable to ambulate. The head and neck will not need to be stabilized after a cauda equina injury, which affects the lumbar and sacral nerve roots. DIF: Cognitive Level: Apply (application)

The nurse is caring for a hospitalized patient with a decreased glomerular filtration rate who is scheduled for an intravenous pyelogram (IVP). Which action will be included in the plan of care? a. Monitor the urine output after the procedure. b. Assist with monitored anesthesia care (MAC). c. Give oral contrast solution before the procedure. d. Insert a large size urinary catheter before the IVP.

ANS: A Patients with impaired renal function are at risk for decreased renal function after IVP because the contrast medium used is nephrotoxic, so the nurse should monitor the patient's urine output. MAC sedation and retention catheterization are not required for the procedure. The contrast medium is given IV, not orally. DIF: Cognitive Level: Apply (application) REF: 1027 TOP: Nursing Process: Planning MSC: NCLEX: Physiological Integrity

A patient who has bacterial meningitis is disoriented and anxious. Which nursing action will be included in the plan of care? a. Encourage family members to remain at the bedside. b. Apply soft restraints to protect the patient from injury. c. Keep the room well-lighted to improve patient orientation. d. Minimize contact with the patient to decrease sensory input.

ANS: A Patients with meningitis and disorientation will be calmed by the presence of someone familiar at the bedside. Restraints should be avoided because they increase agitation and anxiety. The patient requires frequent assessment for complications. The use of touch and a soothing voice will decrease anxiety for most patients. The patient will have photophobia, so the light should be dim. DIF: Cognitive Level: Apply (application)

11. A patient has a new order for magnetic resonance imaging (MRI) to evaluate for left femur osteomyelitis after a hip replacement surgery. Which information indicates that the nurse should consult with the health care provider before scheduling the MRI? a. The patient has a pacemaker. b. The patient is claustrophobic. c. The patient wears a hearing aid. d. The patient is allergic to shellfish.

ANS: A Patients with permanent pacemakers cannot have MRI because of the force exerted by the magnetic field on metal objects. An open MRI will not cause claustrophobia. The patient will need to be instructed to remove the hearing aid before the MRI, but this does not require consultation with the health care provider. Because contrast medium will not be used, shellfish allergy is not a contraindication to MRI.

A patient has a new order for magnetic resonance imaging (MRI) to evaluate possible left femur osteomyelitis after hip arthroplasty surgery. Which information indicates the nurse should consult with the health care provider before scheduling the MRI? a. The patient has a pacemaker. c. The patient wears a hearing aid. b. The patient is claustrophobic. d. The patient is allergic to shellfish.

ANS: A Patients with permanent pacemakers cannot have an MRI because of the force exerted by the magnetic field on metal objects. An open MRI will not cause claustrophobia. The patient will need to be instructed to remove the hearing aid before the MRI, but this does not require consultation with the health care provider. Because contrast medium will not be used, shellfish allergy is not a contraindication to MRI. DIF: Cognitive Level: Apply (application) REF: 1457 TOP: Nursing Process: Assessment MSC: NCLEX: Physiological Integrity

A patient has been taking phenytoin (Dilantin) for 2 years. Which action will the nurse take when evaluating for adverse effects of the medication? a. Inspect the oral mucosa. c. Auscultate the bowel sounds. b. Listen to the lung sounds. d. Check pupil reaction to light.

ANS: A Phenytoin can cause gingival hyperplasia, but does not affect bowel sounds, lung sounds, or pupil reaction to light. DIF: Cognitive Level: Apply (application) REF: 1379 TOP: Nursing Process: Evaluation MSC: NCLEX: Physiological Integrity

Which finding for a patient who is taking hydroxychloroquine (Plaquenil) to treat rheumatoid arthritis is likely to be an adverse effect of the medication? a. Blurred vision c. Abdominal cramping b. Joint tenderness d. Elevated blood pressure

ANS: A Plaquenil can cause retinopathy. The medication should be stopped. Other findings are not related to the medication although they will also be reported. DIF: Cognitive Level: Apply (application) REF: 1528 TOP: Nursing Process: Evaluation MSC: NCLEX: Physiological Integrity

A patient with diabetes is starting on intensive insulin therapy. Which type of insulin will the nurse discuss using for mealtime coverage? a. Lispro (Humalog) mir) b. Glargine (Lantus) c. Detemir (Leve d. NPH (Humulin N)

ANS: A Rapid- or short-acting insulin is used for mealtime coverage for patients receiving intensive insulin therapy. NPH, glargine, or detemir will be used as the basal insulin. DIF: Cognitive Level: Apply (application) REF: 1125 TOP: Nursing Process: Planning MSC: NCLEX: Physiological Integrity

Which statement by the patient indicates a need for additional instruction in administering insulin? a. "I need to rotate injection sites among my arms, legs, and abdomen each day." b. "I can buy the 0.5-mL syringes because the line markings will be easier to see." c. "I do not need to aspirate the plunger to check for blood before injecting insulin." d. "I should draw up the regular insulin first, after injecting air into the NPH bottle."

ANS: A Rotating sites is no longer recommended because there is more consistent insulin absorption when the same site is used consistently. The other patient statements are accurate and indicate that no additional instruction is needed. DIF: Cognitive Level: Apply (application) REF: 1128 TOP: Nursing Process: Evaluation MSC: NCLEX: Health Promotion and Maintenance

Admission vital signs for a brain-injured patient are blood pressure of 128/68 mm Hg, pulse of 110 beats/min, and of respirations 26 breaths/min. Which set of vital signs, if taken 1 hour later, will be of most concern to the nurse? a. Blood pressure of 154/68 mm Hg, pulse of 56 beats/min, respirations of 12 breaths/min b. Blood pressure of 134/72 mm Hg, pulse of 90 beats/min, respirations of 32 breaths/min c. Blood pressure of 148/78 mm Hg, pulse of 112 beats/min, respirations of 28 breaths/min d. Blood pressure of 110/70 mm Hg, pulse of 120 beats/min, respirations of 30 breaths/min

ANS: A Systolic hypertension with widening pulse pressure, bradycardia, and respiratory changes represent Cushing's triad. These findings indicate that the intracranial pressure (ICP) has increased, and brain herniation may be imminent unless immediate action is taken to reduce ICP. The other vital signs may indicate the need for changes in treatment, but they are not indicative of an immediately life-threatening process. DIF: Cognitive Level: Apply (application)


Ensembles d'études connexes

Scaled Agile 4.6 SAFE Release Train Engineer Certification Course

View Set

Chapter 12 PHR and Patient Portals

View Set